NCLEX PN Adult Health

Pataasin ang iyong marka sa homework at exams ngayon gamit ang Quizwiz!

Assessment of a client with a history of stroke reveals that the client understands and follows commands but answers questions with incorrect word choices. The nurse documents the presence of which communication deficit? A. Aphasia B. Apraxia C. Dysarthria D. Dysphagia

Correct Answer: A. Aphasia Aphasia refers to impaired communication due to a neurological condition (eg, stroke, traumatic brain injury). The term aphasia is interchangeable with dysphasia, although aphasia is used more commonly. Receptive aphasia refers to impaired comprehension of speech and writing. A client with receptive aphasia may speak full sentences, but the words do not make sense. The nurse should speak clearly, ask simple "yes" or "no" questions, and use gestures and pictures to increase understanding. Expressive aphasia refers to impaired speech and writing. A client with expressive aphasia may be able to speak short phrases but will have difficulty with word choice. The nurse should listen without interrupting and give the client time to form words. A client may have one type of aphasia or a combination of both, and the severity will vary with the individual. Incorrect Answers: [B. Apraxia] Apraxia refers to loss of the ability to perform a learned movement (eg, whistling, clapping, dressing) due to neurological impairment. [C. Dysarthria] Dysarthria is weakness of the muscles used for speech. Pronunciation and articulation are affected. Comprehension and the meaning of words are intact, but speech is difficult to understand (eg, mumble, lisp). [D. Dysphagia] Dysphagia refers to difficulty swallowing. The term dysphagia is often confused with dysphasia. Clients with motor deficits after a stroke may have dysphagia, which requires swallowing precautions to prevent aspiration. Educational objective:Aphasia refers to a neurological impairment of communication. Clients may have impaired speech and writing, impaired comprehension of words, or a combination of both.

The nurse is reinforcing teaching about ulcer prevention with a client newly diagnosed with peptic ulcer disease. Which of the following client statements indicate appropriate understanding of teaching? Select all that apply. A. "I need to avoid taking medicines like ibuprofen without a prescription." B. "I should avoid drinking excess coffee or cola." C. "I should enroll in a smoking cessation program." D. "I should reduce or eliminate my intake of alcoholic beverages." E. "I will eliminate whole wheat foods, like breads and cereals, from my diet."

Correct Answer: A, B, C, and D Peptic ulcer disease (PUD) is characterized by ulceration of the protective layers (ie, mucosa) of the esophagus, stomach, and/or duodenum. Mucosal "breaks" allow digestive enzymes and stomach acid to digest underlying tissues, leading to potential gastrointestinal bleeding and perforation. Risk factors for PUD include gastrointestinal Helicobacter pylori infections, genetic predisposition, chronic NSAID (eg, aspirin, ibuprofen, naproxen) use, stress, and diet and lifestyle choices. Nurses educating clients with PUD about ulcer prevention should focus on modifiable risk factors: · NSAIDs: Chronic use of NSAIDs can damage the gastric mucosa and delay ulcer healing. · Caffeine: Cola, tea, and coffee should be avoided as they stimulate stomach acid secretion. · Smoking: Tobacco increases secretion of stomach acid and delays ulcer healing. · Alcohol: Alcohol should be avoided as it stimulates stomach acid secretion and impairs ulcer healing. · Meal timing: Eating multiple small meals throughout the day or eating shortly before sleeping may actually worsen PUD by increasing stomach acid secretion. Incorrect Answers: [E. "I will eliminate whole wheat foods, like breads and cereals, from my diet."] Evidence does not support the standard elimination of specific foods from the diet in clients with PUD. However, clients should avoid foods that exacerbate their symptoms. Educational objective:Peptic ulcer disease (PUD) is a gastrointestinal illness caused by breaks in the gastrointestinal mucosa, leading to ulcer formation. To reduce ulcer formation risk, clients with PUD should be instructed to stop smoking; avoid chronic NSAID use; avoid meals or snacks before sleeping; and limit alcohol and caffeine consumption.

A client with aortic stenosis is scheduled for surgery in 2 weeks. The client reports episodes of angina and passing out twice at home. Which would be the best response by the nurse to explain the appropriate activity for this client at this time? A. "Avoid strenuous activity before the surgery." B. "Continue to exercise, even if angina occurs. It will strengthen your heart muscles." C. "Take short walks 3 times a day." D. "There are no activity restrictions unless angina occurs."

Correct Answer: A. "Avoid strenuous activity before the surgery." Aortic stenosis is the narrowing of the orifice between the left ventricle and aorta. Many clients with aortic stenosis are asymptomatic. Symptoms usually develop with exertion as the left ventricle cannot pump enough blood to meet the body's demands due to aortic obstruction (stenosis). These include dyspnea, angina, and, in severe cases, syncope (reduced blood flow to the brain). Clients usually do not experience symptoms at rest. Incorrect Answers: [B. "Continue to exercise, even if angina occurs. It will strengthen your heart muscles."] This client already developed syncope and angina (exertional chest pain) and is at high risk for sudden death with exertion. [C. "Take short walks 3 times a day."] The client should restrict activity. The incidence of sudden death is high in this population, and it is therefore prudent to decrease the strain on the heart while awaiting surgery. [D. "There are no activity restrictions unless angina occurs."] This client already developed syncope and angina (exertional chest pain) and is at high risk for sudden death with exertion. Educational objective:Clients with severe aortic stenosis are at risk for developing syncope and sudden death with exertion. The left ventricle cannot push enough blood into the aorta to meet the body's demands due to the valve stenosis.

The daughter of an 80-year-old client recently diagnosed with Alzheimer disease (AD) says to the nurse, "I guess I can anticipate getting this disease myself at some point." What is an appropriate response by the nurse? A. "Engaging in regular exercise decreases the risk of AD." B. "Having a family history of AD is not a risk factor." C. "Try not to worry about this now as you can't do anything to prevent AD." D. "You should avoid aluminum cans and cookware to prevent AD."

Correct Answer: A. "Engaging in regular exercise decreases the risk of AD." The development of Alzheimer disease (AD) is related to a combination of genetic, lifestyle, and environmental factors. Clients with AD are usually diagnosed at age ≥65. Early-onset AD is a rare form of the disease that develops before age 60 and is strongly related to genetics. Children of clients with early-onset AD have a 50% chance of developing the disease. Incorrect Answers: [B. "Having a family history of AD is not a risk factor."] For late-onset AD, the strongest known risk factor is advancing age. Having a first-degree relative (eg, parent, sibling) with late-onset AD also increases the risk of developing AD. Trauma to the brain has been associated with the development of AD in the future. Brain trauma may be prevented by wearing seat belts and sports helmets and taking measures to prevent falls. [C. "Try not to worry about this now as you can't do anything to prevent AD."] Research suggests that healthy lifestyle choices (eg, smoking cessation, avoiding excessive alcohol intake, exercising regularly, participating in mentally challenging activities) reduce the risk for developing AD [D. "You should avoid aluminum cans and cookware to prevent AD."] Research has failed to confirm that exposure to aluminum products (eg, cans, cookware, antiperspirant deodorant) is related to the development of AD. Educational objective:Research suggests that healthy lifestyle choices (eg, smoking cessation, avoiding excessive alcohol intake, exercising regularly, participating in mentally challenging activities) reduce the risk for developing Alzheimer disease.

The nurse cares for an elderly client with type II diabetes who was diagnosed with diabetic retinopathy. Which statement by the client requires the most immediate action by the nurse? A. "Half of my vision looks like it's being blocked by a curtain." B. "I have to use reading glasses to see small print." C. "My vision seems cloudy and I notice a lot of glare." D. "The colors don't seem as bright as they used to."

Correct Answer: A. "Half of my vision looks like it's being blocked by a curtain." Chronic hyperglycemia can cause microvascular damage in the retina, leading to diabetic retinopathy, the most common cause of new blindness in adults. Option 1 indicates a retinal detachment requiring emergency management. A partial retinal detachment may be painless and cause symptoms such as a curtain blocking part of the visual field, floaters or lines, and sudden flashes of light. An unrepaired complete retinal detachment can cause blindness. Incorrect Answers: [B. "I have to use reading glasses to see small print."] The need for reading glasses is associated with presbyopia and is a common, nonemergency, age-related visual disorder. [C. "My vision seems cloudy and I notice a lot of glare."] Cloudy vision with a glare is associated with a cataract, a nonemergency, age-related visual disorder. [D. "The colors don't seem as bright as they used to."] Although decreased vibrancy of colors is a sign of diabetic retinopathy and requires intervention, it is not indicative of a partial or complete retinal detachment; therefore, it is not an emergency. Educational objective:Chronic hyperglycemia can cause microvascular damage in the retina, leading to diabetic retinopathy, the most common cause of new blindness in adults. A partial retinal detachment may be painless and cause symptoms such as a curtain blocking part of the visual field, floaters or lines, and sudden flashes of light

The nurse reinforces teaching to a client with HIV during a follow-up clinic visit after being on antiretroviral drugs for the past 2 months. Which statement by the client indicates a need for further instruction? A. "I can stop taking these HIV drugs once my viral levels are undetectable." B. "I need to get tested regularly for sexually transmitted infections because I'm sexually active." C. "I should use latex condoms and barriers when having anal, vaginal, or oral sex." D. "I won't stop injecting drugs, but I will use a needle exchange program."

Correct Answer: A. "I can stop taking these HIV drugs once my viral levels are undetectable." Antiretroviral therapy (ART) is a medication regimen consisting of multiple drugs for managing and preventing progression of HIV infections. ART impairs viral replication at multiple points, which leads to decreased viral loads and increased CD4+ (ie, helper T) cell counts. When educating clients about ART, it is critical to explain that treatment is lifelong and requires strict adherence (Option 1). Even clients with undetectable viral loads remain infected with HIV. The discontinuation of, or poor adherence to, ART results in the progression of HIV (which may lead to AIDS) and promotes viral drug resistance. Incorrect Answers: [B. "I need to get tested regularly for sexually transmitted infections because I'm sexually active."] Clients with HIV who are sexually active are at increased risk for sexually transmitted infections (STIs). Regular testing (≥1 time annually) and treatment for STIs are recommended. [C. "I should use latex condoms and barriers when having anal, vaginal, or oral sex."] Latex or polyurethane barriers should be used during sex to prevent STI transmission, as nonbarrier contraception and natural skin condoms (eg, lambskin) offer poor protection against HIV and STI transmission. [D. "I won't stop injecting drugs, but I will use a needle exchange program."] IV drug use is a common source of HIV infection. Although abstinence from IV drugs is preferred, clients who continue to use them should be instructed to avoid sharing needles and receive information about needle and syringe exchange programs. Educational objective:Clients with HIV must be educated to strictly adhere to prescribed antiretroviral therapy to prevent disease progression. Clients with HIV who are sexually active should seek testing for sexually transmitted infections and use latex condoms/barriers during sex. Clients with HIV should use a needle exchange program if using IV drugs.

The nurse is caring for a client receiving treatment for benign prostatic hyperplasia. Which client statement requires further investigation? A. "I have a burning sensation when I urinate." B. "I have been having some dribbling after I finish urinating." C. "I missed 3 days of finasteride while on a trip last week." D. "I was awakened 3 times last night by the need to urinate."

Correct Answer: A. "I have a burning sensation when I urinate." Benign prostatic hyperplasia (BPH) is an abnormal prostate enlargement that most commonly affects male clients age >50. The prostate gradually enlarges and compresses the urethra, causing voiding problems. Symptoms include urinary urgency, frequency, and hesitancy, dribbling urine after voiding, nighttime frequency (nocturia), and urinary retention. Treatment includes lifestyle changes and medications that shrink or slow growth of the prostate, and symptom management interventions (eg, voiding schedule, avoidance of caffeine and antihistamines). Surgical prostate resection may be required. Clients with BPH have increased risk for urinary tract infection (UTI) because of incomplete bladder emptying and urine retention. Symptoms of UTI are often similar to those of BPH; however, burning sensation with urination and cloudy/foul-smelling urine are specific UTI symptoms that require further assessment and treatment. Incorrect Answers: [B. "I have been having some dribbling after I finish urinating."] Dribbling after urination and nocturia are expected findings with BPH. [C. "I missed 3 days of finasteride while on a trip last week."] Dribbling after urination and nocturia are expected findings with BPH. [D. "I was awakened 3 times last night by the need to urinate."] Finasteride (Proscar) is a medication that inhibits further growth of the prostate. Appreciable differences in prostate size are noticed on Educational objective:Clients with benign prostatic hyperplasia (BPH) have increased risk for urinary tract infections (UTI) due to incomplete bladder emptying and urine retention. Symptoms of UTI that differ from those of BPH include burning sensation with urination and cloudy/foul-smelling urine.

Which statement made by the client demonstrates a correct understanding of the home care of an ascending colostomy? A. "I will avoid eating foods such as broccoli and cauliflower." B. "I will empty the pouch when it is one-half full of stool." C. "I will irrigate the colostomy to promote regular bowel movements." D. "I will restrict my fluid intake to 2,000 milliliters of fluid a day."

Correct Answer: A. "I will avoid eating foods such as broccoli and cauliflower." A colostomy is a surgical procedure that creates an opening (stoma) in the abdominal wall for the passage of stool to bypass an obstructed or diseased portion of the colon. Stool drains through the intestinal stoma into a pouch device secured to the skin. Colostomies can be performed on any part of the colon (ascending, transverse, descending, and sigmoid). Depending on the location of the colostomy, characteristics of the stool will vary, with the stool becoming more solid as it passes through the colon. Proper care of the stoma and pouch appliance that should be taught to the client or caregiver includes the following: · Ensure sufficient fluid intake (at least 3,000 mL/day unless contraindicated) to prevent dehydration; identify times to increase fluid requirements (hot weather, increased perspiration, diarrhea). · Identify and eliminate foods that cause gas and odor (broccoli, cauliflower, dried beans, brussels sprouts). · Empty the pouch when it becomes one-third full to prevent leaks due to increasing pouch weight. Incorrect Answers: [C. "I will irrigate the colostomy to promote regular bowel movements."] Stool produced in the ascending and transverse colon is semiliquid, which eliminates the need for irrigation. Irrigation to promote a bowel regimen may be useful for descending or sigmoid colostomies as the stool is more formed. Educational objective:The stool changes from liquid to more solid as it passes through the colon. Proper care of the ostomy and pouching device in clients with a colostomy includes ensuring sufficient fluid intake, preventing gas and odor, and changing the pouching system when it becomes one-third full to prevent leaks.

The nurse is caring for a client with absence seizures. The unlicensed assistive personnel (UAP) asks if the client will "shake and jerk" when having a seizure. Which response from the nurse is the most helpful? A. "No, absence seizures can look like daydreaming or staring off into space." B. "No, you are wrong. Don't worry about that." C. "Yes, so please let me know if you see the client do that." D. "You don't have to monitor the client for seizures."

Correct Answer: A. "No, absence seizures can look like daydreaming or staring off into space." Absence seizures typically occur in children. The presentation is classic and includes: 1. Daydreaming episodes or brief (<10 seconds) staring spells 2. Absence of warning and postictal phases 3. Absence of other forms of epileptic activity (no myoclonus or tonic-clonic activity) 4. Unresponsiveness during the seizure 5. No memory of the seizure Incorrect Answers: [B. "No, you are wrong. Don't worry about that."] The most helpful response by the nurse is one that corrects while educating the UAP. The UAP may be present when a client has a seizure, and understanding of what to expect will aid client care. [C. "Yes, so please let me know if you see the client do that."] Seizures may include tonic (body stiffening), clonic (muscle jerking), atonic (loss of muscle tone or "drop attack"), myoclonic (brief muscle jerk), or tonic-clonic (alternating stiffening and jerking) body motions. Absence seizures do not typically involve these body motions. [D. "You don't have to monitor the client for seizures."] Although it is not the responsibility of the UAP to monitor the client, the UAP may witness a seizure and call for help if needed. Educational objective:Absence seizures are brief periods of staring; there is no evidence of tonic-clonic activity or postictal confusion. The UAP should be educated about absence seizures when involved in the care of such clients.

The nurse reviews prescriptions for assigned adult clients. Which prescription should the nurse question? A. 0.45% sodium chloride (NaCl) solution prescribed for a client with syndrome of inappropriate antidiuretic hormone secretion who has a sodium level of 120 mEq/L (120 mmol/L) B. 0.9% NaCl solution prescribed for a client with gastrointestinal bleeding who has a hemoglobin level of 8.9 g/dL (89 g/L) C. 1,000 mL bolus of 0.9% NaCl solution prescribed for a client with septic shock who has a white blood cell count of 18,000/mm3 (18.0 × 109/L) D. Lactated Ringer's solution prescribed for a male client with hypovolemic shock and a thermal burn who has a hematocrit level of 56% (0.56)

Correct Answer: A. 0.45% sodium chloride (NaCl) solution prescribed for a client with syndrome of inappropriate antidiuretic hormone secretion who has a sodium level of 120 mEq/L (120 mmol/L) Syndrome of inappropriate antidiuretic hormone secretion (SIADH) is associated with increased water reabsorption and excessive intra- and extracellular fluid, which result in hypervolemia from fluid retention and dilutional hyponatremia. In the setting of SIADH, the nurse should question a prescription for a hypotonic solution (eg, 0.45% NaCl; or dextrose water) as it would worsen the fluid and electrolyte imbalance. A prescription for fluid restriction and a hypertonic IV solution (eg, 3% NaCl) administered in small quantities would be appropriate to shift fluid back into the vascular compartment and correct hyponatremia. Incorrect Answers: [B. 0.9% NaCl solution prescribed for a client with gastrointestinal bleeding who has a hemoglobin level of 8.9 g/dL (89 g/L) ] Isotonic fluids (eg, normal saline) are appropriate for clients with volume deficit such as those with gastrointestinal bleeding. [C. 1,000 mL bolus of 0.9% NaCl solution prescribed for a client with septic shock who has a white blood cell count of 18,000/mm3 (18.0 × 109/L) ] Septic shock involves an inflammatory response to pathogens that leads to massive vasodilation and increased capillary permeability, resulting in intravascular hypovolemia and severe hypotension. An isotonic solution (eg, 0.9% NaCl) bolus is prescribed to expand intravascular volume and increase blood pressure. [D. Lactated Ringer's solution prescribed for a male client with hypovolemic shock and a thermal burn who has a hematocrit level of 56% (0.56) ] A burn injury causes tissue damage and increased capillary permeability; this leads to fluid and electrolyte losses related to evaporation and intravascular fluid shifts into the interstitial tissue, which result in hypovolemia, hemoconcentration (eg, hematocrit >53% [0.53]), and hypotension. An isotonic solution (eg, lactated Ringer's) is prescribed to replace fluid and electrolyte losses. Educational objective:Syndrome of inappropriate antidiuretic hormone secretion is associated with hypervolemia and dilutional hyponatremia. Fluid restriction and hypertonic IV solutions (eg, 3% saline) are prescribed to correct hyponatremia.

The nurse is caring for 4 clients. Which client should the nurse see first? A. 2 days post abdominal aortic aneurysm repair with weak pedal pulses and mottled skin on the legs B. 2 days post coronary bypass graft surgery with a white blood cell count of 18,000/mm*3 (18.0 × 10*9/L) D. Chronic heart failure with peripheral edema and shortness of breath on exertion C. Pneumothorax with a chest tube to negative suction and subcutaneous emphysema

Correct Answer: A. 2 days post abdominal aortic aneurysm repair with weak pedal pulses and mottled skin on the legs The nurse should palpate pulses (eg, femoral, posterior tibial, dorsalis pedis), observe skin color, and feel the temperature of the lower extremities in the client with abdominal aortic aneurysm (AAA) repair first. Pulses can be absent for 4-12 hours after surgery due to vasospasm. However, a weak pedal pulse compared to the client's baseline or an absent pulse with a painful, cool, or mottled extremity 2 days post surgery can indicate an arterial or graft occlusion. Unaddressed occlusion may cause life- or limb-threatening ischemia (eg, lower extremities, intestines, kidneys). Incorrect Answers: [B. 2 days post coronary bypass graft surgery with a white blood cell count of 18,000/mm3 (18.0 × 109/L)] An elevated white blood cell count (>11,000/mm3 [11.0 x 109/L]) could be caused by an underlying infection or the stress of surgery. Potential infection does not take priority over graft occlusion. [D. Chronic heart failure with peripheral edema and shortness of breath on exertion] Edema, shortness of breath, and activity intolerance are chronic, expected manifestations of congestive heart failure. [C. Pneumothorax with a chest tube to negative suction and subcutaneous emphysema] Subcutaneous emphysema is air in the tissue surrounding the chest tube insertion site and can occur in a client with a pneumothorax. The nurse should auscultate lung sounds and palpate to determine the degree of emphysema; however, this expected finding does not take priority over graft occlusion. Educational objective:A weak or absent pedal pulse and a cool or mottled extremity in a client who is post abdominal aortic aneurysm repair can indicate an arterial or graft occlusion, leading to possible life- or limb-threatening ischemia.

Which client is at greatest risk for pulmonary embolism? A. A client 6 hours postoperative cesarean section. B. A client in atrial fibrillation. C. A client with a subdural hematoma. D. A client with pneumonia

Correct Answer: A. A client 6 hours postoperative cesarean section Death from pulmonary embolism is often attributed to a missed diagnosis. Early identification of risk factors (eg, stasis of venous blood, hypercoagulability of blood, venous injury) can have a positive effect on client outcome. This postoperative client is at greatest risk due to the presence of the following 4 risk factors: — Abdominal cesarean section surgery (venous injury) — Engorged pelvic vessels from pregnancy (stasis of venous blood, hypercoagulability of blood) — Inactivity/immobility ≥6 hours related to positioning during surgery and the immediate postoperative period and epidural anesthesia (stasis of venous blood) — Postpartum state (hypercoagulability of blood) Incorrect Answers: [B. A client in atrial fibrillation.] In atrial fibrillation, stasis and turbulence of blood increases risk of thrombus formation. Once mobilized, emboli can get trapped in blood vessels causing ischemia. Smaller vasculature and increased blood flow in the brain increases the probability of a stroke, rather than PE. This client has 1 risk factor and is not at greatest risk for PE. [C. A client with a subdural hematoma.] The presence of a subdural hematoma does not pose a significant risk for PE unless the client has been immobile. Many clients with subdural hematomas are asymptomatic and walking. This client is not at greatest risk for PE. [D. A client with pneumonia] Any acute medical illness (eg, pneumonia) can predispose a client to PE from inflammation and the client's relative immobility. However, this risk is lower than the risk for PE from major surgery. Educational objective:Death from pulmonary embolism (PE) is often attributed to a missed diagnosis. Nurses must recognize any condition or situation that predisposes a client to stasis of venous blood, hypercoagulability of blood, and venous injury, as these factors increase the risk for PE.

The practical nurse is collecting data on a client with acute diverticulitis. Which finding will the nurse report immediately to the registered nurse? A. Abdominal pain has progressed to the left upper quadrant B. Hemoglobin is 11.2 g/dL (112 g/L) C. Lying on side with knees drawn up to abdomen and trunk flexed D. White blood cell count is 12,000/mm3 (12 × 10*9/L)

Correct Answer: A. Abdominal pain has progressed to the left upper quadrant Diverticula are saclike protrusions or outpouchings of the intestinal mucosa of the large intestine caused by increased intraluminal pressure (chronic constipation). The left (descending, sigmoid) colon is the most common area for diverticula to develop. When these diverticula become inflamed (diverticulitis), the client may experience acute pain (usually in the left lower quadrant) and systemic signs of infection (eg, fever, tachycardia, nausea, leukocytosis). Potential complications of diverticulitis include abscess formation (continuous fever despite antibiotics and palpable mass) and intestinal perforation resulting in diffuse peritonitis (progressive pain in other quadrants of the abdomen, rigidity, guarding, rebound tenderness). Peritonitis is a potentially lethal complication that should be reported immediately. Incorrect Answers: [B. Hemoglobin is 11.2 g/dL (112 g/L)] Bleeding is expected and obvious in clients with acute diverticulitis, often with a large amount of bright red blood seen in the stool. This client's mild anemia is nonspecific and should not be given priority over the peritoneal signs, which should be reported immediately. [C. Lying on side with knees drawn up to abdomen and trunk flexed] The fetal position is an expected manifestation of abdominal pain. The client with peritonitis prefers to lie still and take shallow breaths to avoid stretching the inflamed peritoneum. The expected pain in diverticulitis should not take priority over the peritoneal signs. [D. White blood cell count is 12,000/mm3 (12 × 10*9/L)] Leukocytosis is expected with acute diverticulitis. However, this client's white blood cell count is only minimally elevated (upper limit of normal is 11,000/mm3 [11 × 109/L]) and is not a priority over peritoneal signs. Educational objective: Diverticulitis is caused by inflammation of diverticula (outpouchings) of the large intestine that can lead to abscess, perforation, peritonitis, and/or bleeding. Peritonitis is a potentially fatal complication that should be reported to the registered nurse.

A client with chronic kidney disease has blood laboratory results as shown:Sodium - 150 mEq/LPotassium - 6.0 mEq/LChloride - 100 mEq/LCalcium - 9.0 mg/dLMagnesium - 2.0 mg/dLPhosphorus - 5.8 mg/dL What is the best afternoon snack to provide this client? A. Apple slices with caramel dip. B. Chips and avocado dip. C. Nonfat yogurt with orange slices. D. Vanilla pudding with strawberries.

Correct Answer: A. Apple slices with caramel dip. Clients with chronic kidney disease (CKD) have decreased glomerular filtration, resulting in retention of fluid, potassium, and phosphorus. Fluid retention is initially treated with sodium restriction and diuretic therapy. Dietary adjustments should also be made to reduce serum potassium and phosphorus. Laboratory values are key to determining allowable foods. Dairy products (eg, milk, yogurt) and certain fruits (eg, bananas, oranges, coconuts, watermelons, and avocados) contain high potassium levels. Dairy products also contain high phosphorus levels. Examples of allowable foods for CKD clients include apples, pears, grapes, pineapple, blackberries, blueberries, and plums. Incorrect Answers: [B. Chips and avocado dip.] Avocados are high in potassium; the chips may be high in sodium. [C. Nonfat yogurt with orange slices.] Pudding and yogurt contain dairy products and are high in phosphorous and potassium. Oranges are high in potassium. [D. Vanilla pudding with strawberries.] Pudding and yogurt contain dairy products and are high in phosphorous and potassium. Oranges are high in potassium. Educational objective:The diet for a client with chronic kidney disease may need to be restricted in fluids, sodium, potassium, and phosphorus. Dairy products (eg, milk, yogurt) and certain fruits (eg, bananas, oranges, coconuts, watermelons, and avocados) contain high potassium levels. Dairy products are also high in phosphorus.

The nurse is assisting with care of a client with blunt head injury admitted for observation, including hourly neurologic checks. At 1:00 AM, the client reports a headache; the neurologic check is normal, and the nurse administers acetaminophen prn. At 2:00 AM, the client appears to be sleeping. What action does the nurse anticipate taking? A. Arouse the client and ask what the current month is B. Awaken the client and check for paresthesia C. Document "relief apparently obtained" and recheck at 3:00 AM D. Let the client sleep but verify respiratory rate

Correct Answer: A. Arouse the client and ask what the current month is. Serial neurologic assessments are important as neurologic abnormalities are often initially subtle, making it important to note any trend in the client's neurologic status. Interventions for neurologic issues are most effective when made early. A neurologic assessment includes the following: 1. Glasgow Coma Scale (GCS): Best eye, verbal, and motor responses. Best verbal response assesses orientation to person, place, and time (time is the most sensitive). 2. Pupils: Equal, round, response to light, and accommodation (PERRLA) 3. Motor: Strength and movement in all four extremities 4. Vital signs: Especially any signs of Cushing's triad of bradycardia, bradypnea/abnormal breathing pattern, and widening pulse pressure (the difference between systolic and diastolic blood pressure readings). The nurse is monitoring for signs of increased intracranial pressure (ICP). This client was not admitted to the hospital to get a good night's sleep, but instead due to the need for serial neurologic assessments by a nurse; assessment is the priority. Incorrect Answers: [B. Awaken the client and check for paresthesia] It is good to awaken the client, but paresthesia is part of a neurovascular assessment. Neurovascular assessment, commonly known as the 5 Ps, consists of assessment for paresthesia, pain, pallor, paralysis, and pulselessness. A neurovascular assessment is used when circulation is a primary concern. It is not the primary neurologic assessment needed in a closed head injury. [C. Document "relief apparently obtained" and recheck at 3:00 AM] Although pain relief has probably been achieved, this option does not reflect any neurologic assessment. One of the early signs of increased ICP is change in level of consciousness, and this option does not assess the client's ability to arouse. [D. Let the client sleep but verify respiratory rate] Checking the respiratory rate and characteristics is part of the neurologic assessment. However, this alone is insufficient for assessing neurologic status after a blunt head injury. Educational objective:Clients must be awakened for a prescribed, necessary neurologic assessment. A neurologic assessment consists of the Glasgow Coma Scale, pupillary checks, movement and strength of the extremities, and vital signs.

A client is brought to the emergency department following a motor vehicle collision. The client's admission vital signs are blood pressure 70/50 mm Hg, pulse 123/min, and respirations 8/min. The nurse anticipates the results of which diagnostic test to best evaluate the client's oxygenation and ventilation status? A. Arterial blood gases B. Chest x-ray C. Hematocrit and hemoglobin levels D. Serum lactate level

Correct Answer: A. Arterial blood gases Normal Adult ABG values at sea level pH — 7.35 - 7.45 PaO2 — 80 - 100 mm Hg PaCO2 — 35 - 45 mm Hg HCO3 — 22 - 26 mEq/L Saturation (SaO2) - 95% - 99% Arterial blood gas (ABG) assessment parameters provide objective data about the efficiency of gas exchange in the lungs and effectively evaluate the following: — Acid-base balance (pH, HCO3) — Oxygenation status (PaO2, partial pressure of oxygen in the arterial blood) — Tissue oxygenation (SaO2, percentage of available hemoglobin saturated with oxygen) — Ventilation (PaCO2, partial pressure of carbon dioxide in the arterial blood) Respiratory failure can occur when oxygenation is inadequate (hypoxemic failure) and/or when ventilation is inadequate (hypercapnic failure). The adequacy of oxygenation and ventilation in a client with respiratory failure is best evaluated through ABG analysis. Incorrect Answers: [B. Chest x-ray ] Chest x-ray is used to determine structural abnormality (eg, enlarged heart, fractured ribs), presence of air, fluid, infiltrates, lesions, and response to treatment. It does not provide objective data about a client's gas exchange, oxygenation, and ventilation status. [C. Hematocrit and hemoglobin levels ] Decreased serum hematocrit and hemoglobin levels can affect the carrying capacity and delivery of oxygen to the tissues. They do not provide objective data about a client's gas exchange, oxygenation, and ventilation status. [D. Serum lactate level] The serum lactate level provides information about anaerobic tissue metabolism (perfusion). It does not provide objective data about a client's gas exchange, oxygenation, and ventilation status. Educational objective:Respiratory failure can occur when oxygenation is inadequate (hypoxemic failure) and/or when ventilation is inadequate (hypercapnic failure). Arterial blood gas analysis provides objective data about the efficiency of gas exchange in the lungs.

A nurse in the pulmonology clinic is reinforcing teaching for a college athlete who was recently diagnosed with moderate persistent asthma. Which common asthma trigger should this client avoid? A. Aspirin and nonsteroidal medications B. Latex products C. Penicillin group antibiotics D. Strenuous physical activity

Correct Answer: A. Aspirin and nonsteroidal medications In clients with asthma, the airways are chronically inflamed. Clients must be able to identify triggers that cause airway reactions and then avoid them. Maintaining a daily log can also help with detection. Common asthma triggers to avoid include the following: — Allergen inhalation (eg, mold, pollen, dust mites, animal dander) —Air pollutants (eg, tobacco smoke) —Infections - viral upper respiratory infections are the most common —Pharmacological agents (eg, beta blockers, aspirin, nonsteroidal anti-inflammatory drugs) Incorrect Answers: [B. Latex products] Latex products are not a common trigger for asthma [C. Penicillin group antibiotics] Cephalosporins have some cross-reactivity with penicillin group antibiotics. Asthma exacerbations are unrelated to penicillin allergy. [D. Strenuous physical activity] Participating in regular aerobic exercise would be beneficial to this client's health and should be encouraged. Although exercise-induced asthma is common in asthmatics (40%-90%), it occurs typically after exercising in cold, dry air. Using a metered-dose inhaler bronchodilator 20 minutes before exercising is beneficial. Swimming in an indoor heated pool is a good alternative to jogging or skiing. Educational objective:Common asthma triggers to avoid include allergen inhalation (eg, mold, pollen, dust mites, animal dander), air pollutants (eg, tobacco smoke), infection, and pharmacological agents (eg, beta blockers, aspirin, nonsteroidal anti-inflammatory drugs).

The nurse is preparing to administer morning medications to a client with type 2 diabetes mellitus and end-stage renal disease who is scheduled for dialysis today. Which medication should the nurse hold for clarification prior to administration? Atenolol 50 mg by mouth daily [0900] Calcium acetate 667 mg by mouth [With each meal] Insulin lispro, high-dose sliding-scale subcutaneous injection with meals and before bedtime [0700] Vitamin E 400 IU by mouth daily [0900] A. Atenolol B. Calcium acetate C. Insulin lispro D. Vitamin E

Correct Answer: A. Atenolol Medication administration may require modification on days that clients are scheduled to receive dialysis. The nurse should consider whether the medication will be dialyzed out of the client's system or may create adverse effects during dialysis. Fluid is removed during dialysis, which may cause hypotension. Typically, antihypertensives are held before dialysis to prevent hypotension. In addition, some medications are dialyzed out of the client's system and should therefore be held until after dialysis. Commonly held medications are water-soluble vitamins (eg, vitamins B and C), antibiotics, and digoxin. Incorrect Answers: [B. Calcium acetate] Clients with chronic kidney disease have high phosphorus levels as the kidney is unable to filter the phosphate from the body; dialysis also does not filter it. Therefore, the client should still take phosphate binders prior to dialysis. Phosphate binders (eg, calcium containing [calcium carbonate and calcium acetate]) and non-calcium containing [sevelamer and lanthanum]) block absorption of ingested phosphate from the intestine and excrete it through feces. [C. Insulin lispro] Lispro is a fast-acting insulin that should be given 15-30 minutes before meals. It is appropriate to give scheduled lispro with breakfast prior to dialysis. [D. Vitamin E] Vitamin E is a fat-soluble vitamin that is not affected by dialysis. It is given to some clients to prevent leg cramps that can be experienced by dialysis clients. Educational Objective Unless otherwise indicated by the health care provider, antihypertensives and other blood pressure-lowering medications (eg, furosemide), antibiotics, digoxin, and water-soluble vitamins (B, C, and folic acid) should be held prior to dialysis.

The telemetry nurse is reviewing a client's cardiac rhythm strip. What is the correct interpretation for this strip? A. Atrial paced rhythm B. First-degree atrioventricular block with bigeminy C. Sinus rhythm with premature ventricular contractions D. Ventricular paced rhythm with failure to sense

Correct Answer: A. Atrial paced rhythm The rhythm strip of a client with a single-chamber atrial pacemaker displays a pacer spike before the P wave, followed by a QRS complex, on an electrocardiogram (ECG). The P wave may appear normal or somewhat distorted following the spike. Atrial pacemakers are often placed for clients experiencing sinoatrial node dysfunction (eg, atrial fibrillation, bradycardia, heart blocks). Incorrect Answers: [B. First-degree atrioventricular block with bigeminy] In first-degree atrioventricular block, every impulse is conducted to the ventricles, but the time of atrioventricular conduction is prolonged. This is evidenced by a prolonged PR interval of >0.20 second. Ventricular bigeminy is a rhythm in which every other heartbeat is a premature ventricular contraction (PVC). Unlike the QRS complexes in this client's ECG, PVCs are not associated with P waves, and the QRS complexes are wide and distorted. [C. Sinus rhythm with premature ventricular contractions] Normal sinus rhythms do not have pacer spikes. Unlike the QRS complexes in this client's ECG, PVCs are not associated with P waves, and the QRS complexes are wide and distorted. [D. Ventricular paced rhythm with failure to sense] Failure to sense appears on an ECG as asynchronous pacer spikes in inappropriate or random locations (eg, pacer spike on the T wave). It should not be confused with failure to capture, in which pacer spikes are located appropriately but there is no electrical response elicited from the heart (eg, no QRS complex after a pacer spike). Educational objective:An atrial paced rhythm displays a pacer spike followed by a normal or distorted P wave, then a QRS complex. Atrial pacemakers are often placed for clients experiencing sinoatrial node dysfunction (eg, atrial fibrillation, bradycardia, heart blocks).

The nurse is reinforcing teaching of proper technique for colostomy irrigation for the home health client. Which client action indicates that further instruction is required? A. Attaches an enema set to the irrigation bag, lubricates it, gently inserts it into the stoma, and holds it in place B. Fills irrigation container with 500-1000 mL of lukewarm tap water and flushes the irrigation tubing C. Hangs the irrigation container on a hook at the level of the shoulder approximately 18-24 inches above the stoma D. Slowly opens the roller clamp, allowing the irrigation solution to flow, but clamps the tubing when cramping occurs

Correct Answer: A. Attaches an enema set to the irrigation bag, lubricates it, gently inserts it into the stoma, and holds it in place A colostomy is a surgical procedure that creates an opening (stoma) in the abdominal wall for the passage of stool to bypass an obstructed or diseased portion of the colon. Stool drains through the intestinal stoma into a pouch device secured to the skin. Clients with a descending or sigmoid colostomy drain stool that is more formed and similar to a normal bowel movement. Although less common, some clients choose to learn to irrigate their colostomy with the intention to create a bowel regimen allowing them to wear a smaller pouch or a dressing over the stoma. In addition, when the colostomy is irrigated daily, the client gains increased control over the passage of stool. The procedure for bowel irrigation is as follows: · Fill the irrigation container with 500-1000 mL of lukewarm water, flush irrigation tubing, and reclamp; hang the container on a hook or IV pole. · Instruct the client to sit on the toilet, place the irrigation sleeve over the stoma, extend the sleeve into the toilet, and hang the irrigation container on a hook at shoulder level (approximately 18-24 inches above the stoma). · Lubricate the cone-tipped irrigator, insert cone and attached catheter gently into the stoma, and hold in place. · Slowly open the roller clamp, allowing irrigation solution to flow for 5-10 minutes. · Clamp the tubing if cramping occurs, until it subsides. · Once the desired amount of solution is instilled, the cone is removed and feces is allowed to drain through the sleeve into the toilet. A cone-tip applicator is used to instill the irrigation solution into the stoma. An enema set should never be used to irrigate a colostomy. The cone-tip applicator is specifically made to avoid damaging the sensitive colostomy opening. Educational objective:Colostomy irrigation allows the client to create a bowel regimen and to apply a dressing or smaller pouch device over the stoma. To properly irrigate the stoma, use 500-1000 mL of lukewarm water, hang the bag 18-24 inches above the stoma, use the cone-tipped irrigator to slowly infuse the solution, and allow stool to drain through the sleeve and into the toilet.

A home health nurse is visiting a client with chronic heart failure. The nurse observes that the client is having trouble answering questions due to breathlessness and cough. Which action should the nurse take first? A. Auscultate breath sounds B. Check for peripheral edema C. Measure the client's vital signs D. Review the client's weight log over the past several days.

Correct Answer: A. Auscultate breath sounds The nurse should prioritize focused assessments based on the ABCs (airway, breathing, circulation). This client is at risk for acute decompensated heart failure and pulmonary edema. Pulmonary edema is an acute, life-threatening condition in which fluid leaks from the blood in the pulmonary vasculature and fills the lung alveoli. Auscultation should be the first assessment and may include crackles, wheezes, and rhonchi if fluid has moved into the lungs. The next priority is for the nurse to measure vital signs. This would identify if the client's heart or respiratory rate is elevated and if the oxygen saturation is compromised. Incorrect Answers: [B. Check for peripheral edema] Checking for peripheral edema and review of the client's weight over the past several days are appropriate assessments that may indicate fluid volume overload. However, they do not take priority over auscultation of the lungs. [C. Measure the client's vital signs] The next priority for the nurse is to measure vital signs, but auscultating breath sounds takes priority. [D. Review the client's weight log over the past several days.] Checking for peripheral edema and review of the client's weight over the past several days are appropriate assessments that may indicate fluid volume overload. However, they do not take priority over auscultation of the lungs. Educational objective:The nurse should follow the ABCs (airway, breathing, circulation) of assessment with a heart failure client who is short of breath and coughing. Assessment should include auscultation of breath sounds and measurement of respiratory rate and oxygen saturation.

A client diagnosed with heart failure has an 8-hour urine output of 200 mL. What is the nurse's first action? A. Auscultate the client's breath sounds B. Encourage the client to increase fluid intake C. Report the findings to the supervising registered nurse D. Start an IV line for diuretic administration

Correct Answer: A. Auscultate the client's breath sounds Urine output of <30 mL/hr may indicate low vascular volume (dehydration, blood loss), decreased renal perfusion (low cardiac output), intrinsic kidney injury, or urine outflow obstruction (enlarged prostate, kinked Foley catheter). Given this client's heart failure, low urine output is likely due to decreased cardiac function and buildup of fluid in the lungs. The nurse should assess the lung sounds for crackles and report to the supervising registered nurse (RN) and the health care provider (HCP), who can prescribe loop diuretics. Incorrect Answers: [B. Encourage the client to increase fluid intake] The client with heart failure is at risk for fluid overload. Fluids should not be encouraged before consulting with the HCP to determine the cause of decreased urine output. If this client is dehydrated, fluids should be encouraged. [C. Report the findings to the supervising registered nurse] The nurse should always assess the client first and then report to the supervising RN and HCP. A diuretic may be prescribed by the HCP if crackles and dyspnea are present. [D. Start an IV line for diuretic administration] The nurse should always assess the client first and then report to the supervising RN and HCP. A diuretic may be prescribed by the HCP if crackles and dyspnea are present. Educational objective:Decreased urine output of <30 mL/hr could be due to low vascular volume (dehydration, blood loss), decreased renal perfusion (low cardiac output), intrinsic kidney injury, or urine outflow obstruction (enlarged prostate, kinked Foley catheter). The nurse should always assess the client first and then report to the supervising registered nurse and health care provider.

The nurse assessing a client with an upper gastrointestinal bleed would expect the client's stool to have which appearance? A. Black tarry B. Bright red bloody C. Light gray "clay-colored" D. Small, dry, rocky-hard masses

Correct Answer: A. Black tarry The nurse would expect a client experiencing an upper gastrointestinal (GI) bleed to have black tarry stools (melena). As blood passes through the GI tract, digestion of the blood ensues, producing the black tarry appearance. Incorrect Answers: [B. Bright red bloody] Bright red bloody stool (hematochezia) would indicate a lower GI hemorrhage. [C. Light gray "clay-colored"] Decreased bile flow into the intestine due to biliary obstruction would produce a light gray "clay-colored" stool. [D. Small, dry, rocky-hard masses] Small, dry, rocky-hard masses are an indication of constipation. Inactivity, slow peristalsis, low intake of fiber in the diet, decreased fluid intake, and some medications (eg, anticholinergics) may contribute to constipation. Educational objective:Clients with upper gastrointestinal (GI) bleed tend to have black tarry stools (melena). Lower GI bleeding will have bright red bloody stool. Blood present on surface of stool indicates hemorrhoids.

A client with polycythemia vera comes to the clinic for a monthly evaluation and treatment. The nurse knows that treatment for this condition consists of which of the following? A. Blood removal B. Iron supplements C. Platelet transfusions D. Steroid injections

Correct Answer: A. Blood removal Polycythemia vera (PV) is a chronic disorder in which the bone marrow produces too many RBCs, causing blood to become viscous and sluggish and leading to venous stasis. Clients are at increased risk for thrombosis, embolization, and decreased tissue perfusion. Although PV is an abnormality of the bone marrow, secondary polycythemia can occur in an individual with chronic hypoxemia (eg, chronic obstructive pulmonary disease). Treatment of PV includes phlebotomy (venipuncture to remove 300-500 mL of blood) to reduce the RBC count (eg, hematocrit <45%). Initially, clients may require phlebotomy every other day until the goal hematocrit is reached. Hematocrit is then monitored monthly, and additional phlebotomy is performed as necessary. Incorrect Answers: [B. Iron supplements] Clients with PV should not receive supplemental iron because excess iron can lead to increased hemoglobin production, worsening the PV. [C. Platelet transfusions] Platelet transfusions would further increase the blood volume and risk for thrombi. [D. Steroid injections] Steroid injections are effective at reducing WBC function (eg, prevention or treatment of graft versus host disease); however, they do not have the same effect on RBCs. Educational objective:Polycythemia vera (PV) is a chronic disorder in which the bone marrow produces an abnormally high number of RBCs. Therapeutic phlebotomy is a standard treatment to reduce the RBC count and risk of thrombosis associated with the increased blood viscosity that occurs in clients with PV.

The nurse is caring for a client with primary adrenal insufficiency (Addison disease). The nurse recognizes which finding associated with the disease? A. Bronze pigmentation of the skin B. Increased body or facial hair C. Purple or red striae on the abdomen D. Supraclavicular fat pad

Correct Answer: A. Bronze pigmentation of the skin Addison disease (primary adrenal insufficiency) is due to decreased function of the adrenal gland, which is responsible for the secretion of glucocorticoids, androgens, and mineralocorticoids. Bronze hyperpigmentation of the skin in sun-exposed areas is due to increased secretion of ACTH by the pituitary in response to low cortisol (ie, glucocorticoid) levels. Clients with Addison disease may develop hypovolemia, hyponatremia, and hyperkalemia from low aldosterone (ie, mineralocorticoid) levels. Other manifestations include: · Slow, progressive onset of weakness and fatigue · Anorexia and weight loss · Orthostatic hypotension · Salt cravings · Nausea and vomiting · Depression and irritability · Vitiligo Incorrect Answers: [B. Increased body or facial hair] Hirsutism (ie, increased facial and body hair), purple or red striae, and a supraclavicular fat pad are characteristics of Cushing syndrome, a condition associated with excess glucocorticoid production. [C. Purple or red striae on the abdomen] Hirsutism (ie, increased facial and body hair), purple or red striae, and a supraclavicular fat pad are characteristics of Cushing syndrome, a condition associated with excess glucocorticoid production. [D. Supraclavicular fat pad] Hirsutism (ie, increased facial and body hair), purple or red striae, and a supraclavicular fat pad are characteristics of Cushing syndrome, a condition associated with excess glucocorticoid production. Educational objective:Addison disease, or primary adrenal insufficiency, is due to the undersecretion of glucocorticoids and mineralocorticoids. Manifestations include bronze skin, hypovolemia, hypotension, hyponatremia, hyperkalemia, and vitiligo.

Which clinical finding would the nurse anticipate in a client with chronic venous insufficiency? A. Brownish, hardened skin on lower extremities B. Diminished peripheral pulses C. Nonhealing ulcer on lateral surface of great toe D. Shiny, hairless lower extremities

Correct Answer: A. Brownish, hardened skin on lower extremities Chronic venous insufficiency (CVI) occurs when the valves in the veins of the lower extremities consistently fail to keep venous blood moving forward, which causes chronic increased venous pressure. The increased pressure pushes fluid out of the vascular space and into the surrounding tissues, where tissue enzymes break down red blood cells. The destruction of red blood cells releases hemosiderin (a reddish-brown protein that stores iron), which causes a brownish skin discoloration; chronic edema and inflammation cause the tissue to harden and appear leathery. Affected skin is highly prone to breakdown and ulcerations (eg, venous leg ulcers), commonly on the inside of the ankle. Incorrect Answers: [B. Diminished peripheral pulses] Diminished pulses, nonhealing ulcers on a toe, and shiny, hairless extremities are usually associated with peripheral arterial disease due to hardening of the arterial walls, which constricts blood flow and impairs transportation of nutrients to tissues. [C. Nonhealing ulcer on lateral surface of great toe] Diminished pulses, nonhealing ulcers on a toe, and shiny, hairless extremities are usually associated with peripheral arterial disease due to hardening of the arterial walls, which constricts blood flow and impairs transportation of nutrients to tissues. [D. Shiny, hairless lower extremities] Diminished pulses, nonhealing ulcers on a toe, and shiny, hairless extremities are usually associated with peripheral arterial disease due to hardening of the arterial walls, which constricts blood flow and impairs transportation of nutrients to tissues. Educational objective:Chronic venous insufficiency occurs when the valves in the veins of the lower extremities fail to keep blood moving forward. Chronic edema and inflammatory changes lead to brownish, thickened skin on the extremities and venous leg ulcers (commonly on the inside of the ankle).

The nurse is caring for a client admitted 3 days ago with bacterial pneumonia who has become short of breath, restless, and difficult to rouse. Which additional finding indicates to the nurse that the client may be developing sepsis? A. Capillary refill time of 5 seconds B. Diminished breath sounds in the lung bases C. Hyperactive bowel sounds D. Urine output of 35 mL/hr

Correct Answer: A. Capillary refill time of 5 seconds Sepsis is an exaggerated, life-threatening response by the body to a bloodstream infection that can result in hemodynamic instability, respiratory failure, and multiorgan dysfunction. Sepsis typically occurs when bacteria from a local or regional infection (eg, pneumonia, urinary tract infection) enters the bloodstream. Clients with sepsis often have manifestations of a systemic inflammatory response (eg, tachycardia, fever, elevated WBCs) and may exhibit signs of impaired organ function, such as: — Absent bowel sounds: Ileus occurs in response to sepsis as blood is shunted away from the gastrointestinal tract to vital organ systems (eg, brain, lungs). — Capillary refill time >3 seconds (in adults): Prolonged capillary refill indicates inadequate perfusion of peripheral tissues — Increased blood glucose in the absence of diabetes: Gluconeogenesis occurs in response to the physiologic stress of infection. — Altered mentation: Changes in mental status (eg, difficulty rousing, agitation, confusion) occur from impaired cerebral perfusion and oxygenation. Incorrect Answers: [B. Diminished breath sounds in the lung bases] Diminished breath sounds in the lung bases are expected in a client with pneumonia. [C. Hyperactive bowel sounds] Hyperactive bowel sounds are associated with gastrointestinal distress, not sepsis. [D. Urine output of 35 mL/hr] Urine output of 35 mL/hr is within normal range (ie, ≥30 mL/hr or ≥0.5 mL/kg/hr). Oliguria is a possible sign of sepsis, however. Educational objective:Sepsis is an exaggerated, life-threatening response by the body to a bloodstream infection that results in multiorgan dysfunction. Manifestations of sepsis include tachycardia, tachypnea, and fever. Additional findings include altered tissue perfusion (eg, prolonged capillary refill, absent bowel sounds) and oliguria.

A client receiving total parenteral nutrition reports nausea, abdominal pain, and excessive thirst. What is the best action for the nurse to take? A. Check the client's blood glucose B. Check the client's vital signs C. Report the findings to the health care provider D. Slow down the rate of infusion

Correct Answer: A. Check the client's blood glucose A complication of total parenteral nutrition (TPN) is hyperglycemia, as evidenced by excessive thirst, increased urination, abdominal pain, headache, fatigue, and blurred vision. The development of hyperglycemia is related to the following: · Excessive dextrose infusion · Increased production of counterregulatory hormones in response to acute illness · High infusion rate · Administration of medications such as corticosteroids · Infection Interventions to resolve TPN-associated hyperglycemia include reducing the amount of carbohydrate in the TPN solution, slowing down the infusion rate, and administering subcutaneous insulin. The licensed practical nurse (LPN) should collaborate with the registered nurse (RN) and health care provider (HCP) before implementing these interventions. Incorrect Answers: [B. Check the client's vital signs] Checking vital signs will not confirm that the client is experiencing hyperglycemia. [C. Report the findings to the health care provider] The nurse first needs to gather pertinent data regarding the client's change in status (eg, check the blood glucose). The health care provider will need to be contacted if a change in TPN treatment is indicated. [D. Slow down the rate of infusion] Slowing down the rate of infusion is an intervention to resolve hyperglycemia; the nurse needs to first confirm that the client's symptoms are related to high blood glucose. Educational objective:Hyperglycemia is a complication of total parenteral nutrition (TPN). Based on the client's reported symptoms related to hyperglycemia, the nurse needs to assess the client's blood sugar before implementing an intervention.

A client with myocardial infarction underwent successful revascularization with stent placement, is now chest pain free, and will be attending cardiac rehabilitation as an outpatient. The client is embarrassed to talk to the health care provider about resuming sexual relations. What teaching should the nurse reinforce with this client? A. Client may be ready for sexual activity if no symptoms occur when climbing 2 flights of stairs B. Client will be ready for sexual activity after completion of cardiac rehabilitation C. It will be 6 months before the heart is healthy enough for sexual activity D. Medications such as sildenafil or tadalafil are available as prescriptions from the health care provider

Correct Answer: A. Client may be ready for sexual activity if no symptoms occur when climbing 2 flights of stairs Sexual counseling is important for clients with cardiac alterations, yet the subject of sex can be difficult for clients and health care providers (HCPs) to discuss and is often neglected. A client's concern about resumption of sexual activity can be very stressful, even more stressful than sexual activity. Therefore, the nurse should encourage clients to discuss concerns with the HCP. In general, if a client can walk 1 block or climb 2 flights of stairs without symptoms, the client can resume sexual activity safely after receiving HCP approval. Incorrect Answers: [B. Client will be ready for sexual activity after completion of cardiac rehabilitation] This client will participate in cardiac rehabilitation, but it likely will not impact the client's ability to engage in sexual activity, especially if the client remains asymptomatic. [C. It will be 6 months before the heart is healthy enough for sexual activity] When to resume sexual activity will depend on the emotional readiness of the client and the client's partner and on the HCP's assessment of recovery. In general, it is safe to resume sexual activity 7-10 days after an uncomplicated myocardial infarction (MI). [D. Medications such as sildenafil or tadalafil are available as prescriptions from the health care provider] The use of erectile agents (eg, sildenafil) is contraindicated if the client is consuming any form of nitrates. Sublingual nitroglycerin is often prescribed as prophylaxis for angina in clients with previous acute coronary syndromes (eg, MI). Educational objective:It is important to educate clients and their partners about sexual activity after a myocardial infarction. In general, it is safe for clients to consider resumption of sexual activity when they can walk 1 block or climb 2 flights of stairs without symptoms and have the health care provider's approval.

The nurse is reinforcing discharge instructions for several clients. Which client should receive information about the need for prophylactic antibiotics prior to dental procedures? A. Client who had mechanical aortic valve replacement B. Client who had mitral valvuloplasty repair C. Client who had myocardial infarction with subsequent heart failure D. Client who has mitral valve prolapse with regurgitation

Correct Answer: A. Client who had mechanical aortic valve replacement Certain individuals should receive prophylactic antibiotics prior to dental procedures to prevent infective endocarditis (IE). These include persons with the following: — Prosthetic heart valve or prosthetic material used to repair heart valve — Previous history of IE — Some forms of congenital heart disease: x Unrepaired cyanotic congenital defect x Repaired congenital defect with prosthetic material or device for 6 months after the procedure x Repaired congenital defect with residual defects at the site or adjacent to the site of a prosthetic patch or device — Cardiac transplantation and subsequent development of heart valve disease Incorrect Answers: [B. Client who had mitral valvuloplasty repair] The client who had mitral valve repair without the use of prosthetic material is at low risk for IE. [C. Client who had myocardial infarction with subsequent heart failure] The client who had an acute myocardial infarction and subsequent heart failure is not at risk for IE. [D. Client who has mitral valve prolapse with regurgitation] The client with mitral valve prolapse with or without regurgitation or aortic valve disease does not require prophylaxis for IE. Educational objective:Clients with any form of prosthetic material in their heart valves or who have an unrepaired cyanotic congenital heart defect or a prior history of infective endocarditis (IE) should take prophylactic antibiotics prior to dental procedures to prevent development of IE.

The nurse is caring for a client who was struck in the face by the airbag during a motor vehicle collision. The client reports black spots floating in the field of vision. Which intervention should the nurse implement first? A. Cover both eyes with patches B. Make client npo C. Notify the health care provider D. Place client on bed rest

Correct Answer: A. Cover both eyes with patches This client is showing symptoms of retinal detachment (separation of sensory retina from pigment epithelium and choroid with fluid accumulation). This condition can be caused by aging or head trauma. Classic symptoms include flashes of light, floaters or black spots across the field of vision, the sense of a curtain being drawn over the eye, and loss of a portion of the visual field. This is an emergent condition that can result in blindness if left untreated. The primary intervention should include covering both eyes with patches to prevent further detachment. Up to 25% of clients who experience retinal detachment in one eye will also develop this in the other eye. Additional immediate interventions include placing the client on bed rest, notifying the health care provider or ophthalmology specialist, performing a visual acuity examination, and making the client npo for possible emergency surgery. Educational objective:Emergent treatment of a client with possible retinal detachment should include covering both eyes with patches to prevent further detachment, notifying the health care provider or ophthalmology specialist, placing the client on bed rest, and making the client npo for possible emergency surgery.

A client with coarse crackles at the base of both lungs suddenly becomes agitated, anxious, cyanotic, and dyspneic. Which of the following positions is appropriate? A. High-Fowler's B. Left-Sims' C. Modified Trendelenburg D. Supine

Correct Answer: A. High-Fowler's This client is experiencing acute pulmonary edema. Sitting upright (high Fowler's position) at the edge of the bed with the legs dangling is the appropriate position. This position will reduce the venous return to the heart and congestion to the lung, promote lung expansion, and immediately alleviate the client's symptoms and respiratory effort. Incorrect Answers: [B. Left-Sims'] Left Sims', supine, and modified Trendelenburg positions will increase the venous return, augment pulmonary congestion, and worsen the client's condition. Modified Trendelenburg is an optimal position for a client in hypovolemic shock. [C. Modified Trendelenburg] Left Sims', supine, and modified Trendelenburg positions will increase the venous return, augment pulmonary congestion, and worsen the client's condition. Modified Trendelenburg is an optimal position for a client in hypovolemic shock. [D. Supine] Left Sims', supine, and modified Trendelenburg positions will increase the venous return, augment pulmonary congestion, and worsen the client's condition. Modified Trendelenburg is an optimal position for a client in hypovolemic shock. Educational objective:The proper positioning for acute pulmonary edema is high Fowler's with the legs dangling.

The nurse is assisting in planning care for a client experiencing an acute attack of Ménière disease. Which action is a high priority to include in the plan of care? A. Initiate fall precautions B. Keep the emesis basin at bedside C. Provide a quiet environment D. Start IV fluids

Correct Answer: A. Initiate fall precautions Ménière disease (endolymphatic hydrops) results from excess fluid accumulation inside the inner ear. Clients have episodic attacks of vertigo, tinnitus, hearing loss, and aural fullness. The vertigo can be severe and associated with nausea and vomiting. Clients report feelings of being pulled to the ground (drop attacks). During an attack, the client is treated with vestibular suppressants, including sedatives (eg, benzodiazepines such as diazepam), antihistamines (eg, diphenhydramine, meclizine), anticholinergics (eg, scopolamine), and antiemetics. The nurse's priority is to provide for client safety with fall precautions given the severe vertigo and use of sedating medications. Fall precautions include adjusting the bed to a low position with side rails up and instructing the client to call for help before getting up. Clients should be in a quiet, dark room and avoid sudden head movements to minimize vertigo and should not watch television or look at flickering lights to reduce stimulation. The client's diet should be salt restricted to prevent fluid buildup in the ear. Incorrect Answers: [B. Keep the emesis basin at bedside] An emesis basin should be provided at the bedside, but fall precautions are the priority. [C. Provide a quiet environment] A quiet environment can help minimize vertigo. However, it is a lower priority than fall precautions. [D. Start IV fluids] Most clients with Ménière disease require parenteral fluids due to nausea and vomiting. However, these are not the highest priority. Educational objective:Clients with Ménière disease (endolymphatic hydrops) can have severe vertigo, tinnitus, hearing loss, and aural fullness. The priority is for the nurse to institute safety measures, such as fall precautions, for these clients. Clients will require a salt-restricted diet.

A client was medicated 2 hours ago with IV morphine 2 mg to relieve moderate abdominal pain after an appendectomy. The client becomes lethargic but arouses easily to verbal and tactile stimuli and is oriented to time, place, and person. The pulse oximeter reading has dropped from 99% to 89% on room air. Which oxygen delivery device is the most appropriate for the nurse to apply? A. Nasal cannula B. Non-rebreather mask C. Face mask D. Venturi mask

Correct Answer: A. Nasal cannula The nasal cannula is the most appropriate oxygen delivery device to apply at this time because it is comfortable, used for the short term, inexpensive, and unobtrusive so the client can eat and drink fluids. It can supply adequate oxygen concentrations of up to 44%. This client is most likely hypoventilating as a result of the opioid medication but is still alert, oriented, and able to follow directions. Therefore, the client should be able to breathe deeply through the nose, and the hypoxemia should reverse rapidly. Incorrect Answers: [B. Non-rebreather mask] In emergency situations, a non-rebreather mask is used to deliver high concentrations of oxygen (up to 90%-95%). It requires a tight face seal and is both restrictive and uncomfortable. [C. Face mask] The simple face mask delivers a higher concentration of oxygen (40%-60%). However, it is uncomfortable and restrictive as it must be removed to eat or drink. Use of this mask may be appropriate at a later time if hypoxemia does not resolve. [D. Venturi mask] The simple face mask delivers a higher concentration of oxygen (40%-60%). However, it is uncomfortable and restrictive as it must be removed to eat or drink. Use of this mask may be appropriate at a later time if hypoxemia does not resolve. Educational objective:The nasal cannula is an inexpensive, comfortable, low-flow oxygen delivery device capable of delivering oxygen concentrations of up to 44%. It can be used in the short term to effectively treat hypoventilation and reverse hypoxemia in responsive postoperative clients.

A nurse is caring for an elderly client who had a colectomy for removal of cancer 2 days ago. The client is becoming increasingly restless and has been given IV morphine every 2 hours for severe pain. Respirations are 28/min and shallow. Which arterial blood gas results best indicate that the client is in acute respiratory failure and needs immediate intervention? A. PaO2 49 mm Hg (6.5 kPa), PaCO2 60 mm Hg (8.0 kPa) B. PaO2 64 mm Hg (8.5 kPa), PaCO2 45 mm Hg (6.0 kPa) C. PaO2 70 mm Hg (9.3 kPa), PaCO2 30 mm Hg (4.0 kPa) D. PaO2 86 mm Hg (11.5 kPa), PaCO2 25 mm Hg (3.33 kPa)

Correct Answer: A. PaO2 49 mm Hg (6.5 kPa), PaCO2 60 mm Hg (8.0 kPa) Normal Adult ABG values at sea level pH — 7.35 - 7.45 PaO2 — 80 - 100 mm Hg (10.7 - 13.3 kPa) PaCO2 — 35 - 45 mm Hg (4.66 - 5.98 kPa) Bicarbonate HCO3 — 22 - 26 mEq/L (22 - 26 mmoI/L) Saturation (SaO2) - 95% - 99% Acute respiratory failure (ARF) is defined as inadequate gas exchange that results from too much carbon dioxide or inadequate oxygen. ARF may be intrapulmonary (eg, pneumonia, pulmonary embolism) or extrapulmonary (eg, head injury, opioid overdose) in origin. ARF is a potential complication of oversedation or following major surgical procedures, especially those involving the thorax and abdomen that may result in injury to the lungs. ABG values that indicate the presence of ARF are decreased PaO2 ≤60 mm Hg (8.0 kPa) even with administration of supplemental oxygen and elevated PaCO2 ≥50 mm Hg (6.67 kPa). ARF can occur rapidly over minutes to hours without allowing for physiologic compensation, resulting in respiratory acidosis (pH ≤7.35). Immediate intervention with high concentrations of oxygen is indicated, and positive-pressure mechanical ventilation may be necessary to prevent cardiac arrest. Incorrect Answers: [B. PaO2 64 mm Hg (8.5 kPa), PaCO2 45 mm Hg (6.0 kPa)] PaO2 64 mm Hg (8.5 kPa) indicates hypoxemia, and PaCO2 45 mm Hg (6.0 kPa) is within the normal range. [C. PaO2 70 mm Hg (9.3 kPa), PaCO2 30 mm Hg (4.0 kPa)] PaO2 70 mm Hg (9.3 kPa) indicates hypoxemia, and PaCO2 30 mm Hg (4.0 kPa) indicates increased ventilation and an alkalotic state. Respiratory alkalosis may be caused by hyperventilation; however, in the case of ARF, the increased respirations are an attempt to compensate for hypoxia. [D. PaO2 86 mm Hg (11.5 kPa), PaCO2 25 mm Hg (3.33 kPa)] PaO2 86 mm Hg (11.5 kPa) is within normal range, and PaCO2 25 mm Hg (3.33 kPa) indicates increased ventilation and an alkalotic state. These results do not meet the criteria for ARF. Educational objective:Acute respiratory failure is indicated by decreased PaO2 ≤60 mm Hg (8.0 kPa), PaCO2 ≥50 mm Hg (6.67 kPa), and pH ≤7.35.

The practical nurse is caring for a client with newly diagnosed infective endocarditis. Which assessment finding by the nurse is the highest priority to report to the registered nurse? A. Pain and pallor in one foot B. Pain in both knees C. Splinter hemorrhages in the nail beds D. Temperature of 102.2 F (39 C)

Correct Answer: A. Pain and pallor in one foot In infective endocarditis (IE), the vegetations growing on the valves can break off and embolize to various organs, resulting in life-threatening complications, including: — Stroke: Paralysis on one side — Spinal cord ischemia: Paralysis of both legs — Ischemia to the extremities: Pain, pallor, and cold foot or arm — Intestinal infarction: Abdominal pain — Splenic infarction: Left upper quadrant pain Incorrect Answers: [B. Pain in both knees] IE commonly presents with fever, arthralgias (multiple joints pains), weakness, and fatigue. These are expected and do not need to be reported during the initial stages of treatment. IE clients typically require IV antibiotics for 4-6 weeks. Fever may persist for several days after treatment is started. If the client is febrile persistently after 1-2 weeks of antibiotics, this must be reported as it may indicate ineffective antibiotic therapy. [C. Splinter hemorrhages in the nail beds] Splinter hemorrhages can occur with infection of the heart valves (endocarditis). They may be caused by damage from swelling of the blood vessels (vasculitis) or tiny clots that damage the small capillaries (microemboli). The presence of splinter hemorrhages is not as critical as the macroemboli causing a stroke or painful cold leg. [D. Temperature of 102.2 F (39 C)] IE commonly presents with fever, arthralgias (multiple joints pains), weakness, and fatigue. These are expected and do not need to be reported during the initial stages of treatment. IE clients typically require IV antibiotics for 4-6 weeks. Fever may persist for several days after treatment is started. If the client is febrile persistently after 1-2 weeks of antibiotics, this must be reported as it may indicate ineffective antibiotic therapy. Educational objective:Infective endocarditis causes the formation of vegetations on valve and endocardial surfaces. Embolization to various organ sites can occur. The onset of hemiplegia or painful, pale, cold foot/leg could indicate embolization and should be reported immediately.

An obese 85-year-old client, who is an avid gardener and eats only home-grown fruits, legumes, and vegetables, is admitted to the hospital with pneumonia after having an upper respiratory tract infection for a week. Which factor puts the client at greatest risk for developing pneumonia? A. Advanced age B. Environmental exposure C. Nutritional deficit D. Obesity

Correct Answer: Advanced age Pneumonia is an inflammatory process of the lung usually caused by an infectious or noninfectious agent. Any condition, such as advanced age (>65), that compromises the respiratory system's mechanisms to maintain the sterility of the lower airway can increase the risk for pneumonia. These include: — Advanced age (>65) - presence of coexisting diseases, decreased gag/cough reflex, decrease immune response and lung function — Young age (<2) - immature immune system — Central nervous system depression or decreased level of consciousness - decreased gag/cough reflexes and risk of aspiration — Immunosuppression or malnutrition - impaired ability to fight infection — Immobility, smoking, infection - ineffective airway clearance Incorrect Answers: [B. Environmental exposure] Any condition that compromises the respiratory system's protective mechanisms, which maintain the sterility of the lower airway, can increase the risk for pneumonia. [C. Nutritional deficit] Any condition that compromises the respiratory system's protective mechanisms, which maintain the sterility of the lower airway, can increase the risk for pneumonia. [D. Obesity] Any condition that compromises the respiratory system's protective mechanisms, which maintain the sterility of the lower airway, can increase the risk for pneumonia.

The nurse is caring for a college football player who recently sustained an accidental, forceful, helmet-to-helmet collision with another running football player. Which sign/symptom is most concerning and alerts the practical nurse to contact the registered nurse immediately? A. Continually oozing epistaxis B. "Hairnet" across vision C. One episode of coffee-ground emesis D. Temporal headache

Correct Answer: B. "Hairnet" across vision Classic signs of retinal detachment include a curtain coming across the vision, floaters or lightning flashes in the vision field, and "gnats/hairnet/cobweb" throughout the vision. Aging can be a cause and can result in retinal tears or holes and spontaneous detachment. However, retinal detachment can also be caused by forceful head trauma. Retinal detachment requires emergent consultation and treatment as most untreated, symptomatic detachments result in blindness of the affected eyes. Incorrect Answers: [A. Continually oozing epistaxis] The nose is a very vascular area. Continued slow oozing of blood from the nose may occur after forceful head trauma and is often disturbing to the client. However, it is not a serious condition and not a priority over the detached retina. [C. One episode of coffee-ground emesis] An episode of coffee-ground emesis is probably due to swallowed blood from the nosebleed and is an expected finding in this client. The emesis would be a concern if it was projectile, bright red blood, or ongoing. [D. Temporal headache] Headache would be expected from the trauma and is not in itself a concerning finding. Headache would be a concern if it worsened or was not relieved with over-the-counter analgesics. Educational objective:Retinal detachment can result from trauma and aging. Classic symptoms include lightning flashes, floaters, a curtain across the vision, and "gnats/cobweb/hairnet" throughout the vision. Retinal detachment requires emergent consultation and treatment to prevent loss of vision.

The nurse is reinforcing teaching on nutritional therapy for a client with chronic kidney disease. Which statement indicates that further teaching is needed? A. ""Because I have chronic kidney disease, I should avoid canned soups and cold-cut sandwiches." B. "I can use a salt substitute because I am required to restrict both sodium and potassium in my daily diet." C. "I must avoid eating raw carrots and tomatoes on my salads because I take hemodialysis treatments." D. "The popsicles I eat should be counted in my daily fluid intake because they become liquid at room temperature."

Correct Answer: B. "I can use a salt substitute because I am required to restrict both sodium and potassium in my daily diet." Clients with chronic kidney disease (CKD) are at risk for fluid overload and hyperkalemia. Clients should avoid salt substitutes, which typically contain potassium chloride and may contribute to hyperkalemia. To avoid further complications and prevent progressive kidney damage, clients with CKD are advised to utilize certain dietary restrictions, including: — Sodium restriction - Avoid high-sodium foods such as cured meats, pickled foods, canned soups, frankfurters, cold cuts, soy sauce, and salad dressings — Potassium restriction - Avoid high-potassium foods such as raw carrots, tomatoes, and orange juice — Fluid intake monitoring - Monitor fluid intake closely and accurately, being careful to include foods that are liquid-based (eg, popsicles), because fluid is often restricted — Low-protein diet - Eat 0.6-0.8 g/kg/day of protein to help prevent progression of kidney disease. If the client is already on hemodialysis, increased protein intake is recommended to prevent malnutrition. — Low-phosphorus diet - Avoid foods high in phosphorus (eg, chicken, turkey, dairy). Educational objective:Clients with chronic kidney disease are at risk for fluid overload and hyperkalemia. An appropriate renal diet includes low sodium, low potassium, low phosphorus, and low protein.

The nurse is reinforcing education about lifestyle modifications for a client newly diagnosed with Meniere disease. Which statement by the client indicates a need for further teaching? A. "I need to enroll in a smoking cessation program." B. "I need to restrict the amount of potassium in my diet." C. "I will lie down and avoid walking unassisted during acute attacks." D. "I will limit the amount of caffeine and alcohol that I consume."

Correct Answer: B. "I need to restrict the amount of potassium in my diet." Meniere disease (endolymphatic hydrops) results from excess fluid accumulation in the inner ear. Clients have episodic attacks of vertigo, tinnitus, hearing loss, and feelings of fullness or pressure in the ear. The disorder typically affects only one ear and can lead to permanent hearing loss. Attacks of Meniere disease can result in a total loss of proprioception, and clients often report feeling "pulled to the ground" (drop attacks), making client safety a priority. Vertigo can be severe and is associated with nausea, vomiting, and feelings of anxiety. Self-care for Meniere disease may include: — Consuming a low-sodium diet to decrease the potential for fluid excess within the inner ear. Intake of potassium and other electrolytes does not need to be restricted — Limiting or avoiding aggravating substances (eg, nicotine, caffeine, alcohol) and stimuli (eg, flickering lights, watching television) —Adhering to prescribed therapies for relief of symptoms (eg, antiemetics, antihistamines, sedatives, and mild diuretics) — Avoiding sudden changes in the position of the head (eg, bending over) during vertigo spells — Participating in vestibular rehabilitation therapy — Implementing safety measures during attacks (eg, assistance with walking, bed rest) Educational objective:Meniere disease results from excess fluid accumulation in the inner ear. Attacks involve severe vertigo, nausea, and hearing loss. Clients with Meniere disease should be taught to adhere to a low-sodium diet; eliminate tobacco products; limit caffeine and alcohol; and limit or avoid exacerbating factors (eg, flickering lights).

The nurse is reinforcing teaching of proper foot care to a client with diabetes mellitus. Which statement by the client indicates that further teaching is needed? A. "I will apply lanolin to my feet to prevent dry skin." B. "I will make sure my flip-flops are made of leather." C. "I will not apply a heating pad directly to my feet." D. "I will test the water with a thermometer before bathing."

Correct Answer: B. "I will make sure my flip-flops are made of leather." Individuals with type I or II diabetes mellitus are at increased risk for developing an infection or ulcer on their feet due to the chronic complication of peripheral neuropathy. Peripheral neuropathy results from damage to the nerves in the extremities. Clients may be unable to feel injuries if they occur and must take extra measures in caring for their feet. Clients should be taught to wear closed-toed, leather-based shoes to prevent injury. Careful, daily attention to foot care can prevent long-term complications. The following instructions are used in teaching diabetic foot care: — Proper footwear - Wear shoes that are comfortable, supportive, and well-fitting (preferably leather). Avoid high-heeled, open-heeled, and open-toed shoes. Wear clean, absorbent (eg, cotton) socks. — Daily hygiene and inspection - Use a mild soap and warm water; dry feet thoroughly, especially between toes. Apply lanolin lotion (but not between toes) to prevent drying. Inspect for any break in skin integrity and use a mirror or a second person to visualize the bottom of the feet. Trim toenails straight across; file edges along contour of the toes. — Avoid injury - Do not walk barefoot, use hot water or heating pads, wear restrictive shoes or clothing, or cross the legs for extended periods. A thermometer or sensitive area of the body (eg, wrist) should be used to determine appropriate water temperatures (<104 F [40 C]) — Report problems - Do not self-treat corns, calluses, or ingrown toenails. Cleanse cuts or abrasions with mild soap and water; report nonhealing or infected injuries to the health care provider immediately. Educational objective:Careful, daily attention to foot care can prevent long-term complications. Clients with diabetes should be taught to wear closed-toed, leather-based shoes to prevent injury. Clients should also receive instruction regarding daily hygiene and inspection, injury avoidance, and prompt reporting of problems.

A hospitalized client with thyrotoxicosis receives atenolol 50 mg PO daily. Which statement by the nurse accurately reinforces the client's understanding of this medication's purpose? A. "Atenolol is an iodine-based medication that blocks the release of thyroid hormones." B. "It is used to treat some of the symptoms of hyperthyroidism, such as increased heart rate." C. "This drug is radioactive and damages or destroys the thyroid tissue." D. "This first-line antithyroid drug inhibits the synthesis of thyroid hormones."

Correct Answer: B. "It is used to treat some of the symptoms of hyperthyroidism, such as increased heart rate." Beta-adrenergic blockers (atenolol, metoprolol, and propranolol) are used to relieve some of the symptoms of thyrotoxicosis (thyroid storm), a complication of hyperthyroidism in which excessive thyroid hormones are released into the circulation. Beta blockers block the effects of the sympathetic nervous system and treat symptoms such as tachycardia, hypertension, irritability, tremors, and nervousness in hyperthyroidism. Incorrect Answers: [A. "Atenolol is an iodine-based medication that blocks the release of thyroid hormones."] Atenolol is not iodine based. Iodine is used to treat thyrotoxicosis or to prepare the client for a thyroidectomy. In large doses, iodine quickly blocks the release of T4 and T3 from the gland within hours. In addition, iodine decreases thyroid gland vascularity and is helpful when preparing the client for a thyroidectomy. [C. "This drug is radioactive and damages or destroys the thyroid tissue."] Atenolol does not contain radioactive iodine, the primary treatment for hyperthyroidism. It damages or destroys the thyroid tissue, therefore limiting thyroid secretion and eventually making the client hypothyroid. [D. "This first-line antithyroid drug inhibits the synthesis of thyroid hormones."] Propylthiouracil and methimazole (Tapazole) are first-line antithyroid drugs used to inhibit thyroid hormone synthesis. Educational objective:Beta-adrenergic blockers (atenolol, metoprolol, and propranolol) are given to relieve some of the symptoms of thyrotoxicosis. They block the effects of the sympathetic nervous system and treat symptoms such as tachycardia, hypertension, irritability, tremors, and nervousness in hyperthyroidism.

The nurse is assigned to care for 4 clients at an outpatient ophthalmology clinic. Which client report would most likely indicate a serious pathology that should be given priority? A. "I'm having trouble reading the small print on a distant object." B. "It's as if a curtain is crossing my field of vision today." C. "I've been noticing lately that my central vision is blurry." D. "I've had yellow discharge from my eye for the past week."

Correct Answer: B. "It's as if a curtain is crossing my field of vision today." Retinal detachment is separation of the sensory retina from the underlying pigment epithelium with fluid accumulation. It can be a result of spontaneous atrophic retinal breaks or acute trauma. Common symptoms include a painless loss of vision "like a curtain" coming across the field of vision, lightning flashes, or a gnat/hairnet appearance in the vision field. This report needs emergent evaluation. Untreated symptomatic retinal detachment usually leads to blindness in that eye. In addition, this is the only listed presentation that is acute: the rule for prioritization is acute before chronic. Incorrect Answers: [A. "I'm having trouble reading the small print on a distant object."] Myopia (nearsightedness) is an inability to accommodate for objects at a distance. It is not emergent and will require refractive correction with glasses/contacts. [C. "I've been noticing lately that my central vision is blurry."] Macular degeneration is related to retinal aging and causes progressive, irreversible central vision loss. However, it is not an acute condition. [D. "I've had yellow discharge from my eye for the past week."] Conjunctivitis is an infection or inflammation of the conjunctiva that can be caused by bacteria, viruses, allergens, or chemical irritants. Bacterial infections tend to cause purulent drainage; allergic or viral causes result in serous discharge; and an allergic conjunctivitis usually involves pruritus (itching). The client will probably need eye drops and a reminder about good handwashing technique, but the condition is not emergent. Educational objective:Acute retinal detachment includes the sensation of a curtain coming across the field of vision, lightning flashes, or a gnat/hairnet appearance in the vision field. It requires an emergent evaluation.

An elderly client with dementia frequently exhibits sundowning behavior while living in a community-based residential facility. When the nurse finds the client wandering at night, which of the following statements is most appropriate? A. "Don't you know it's not morning yet?" B. "It's time to get back to bed now." C. "You might fall if you wander in the dark." D. "You should not leave your room without assistance."

Correct Answer: B. "It's time to get back to bed now." Sundowning refers to the increased confusion experienced by an individual with dementia; it occurs at night, when lighting is inadequate, or when the client is excessively fatigued. Wandering is a common associated behavior. A client with mild-to-moderate dementia may need frequent reality reorientation to promote appropriate behaviors. However, with advanced dementia, reality orientation may not be effective and might cause the client to feel anxious, leading to inappropriate behaviors and aggression. In this situation, validation therapy is more appropriate and involves recognizing and exploring the client's feelings and concerns but not reinforcing or arguing with any incorrect perceptions. Incorrect Answers: [A. "Don't you know it's not morning yet?"] This statement calls attention to the client's memory and cognitive issues but does not provide any useful information for reorientation. In addition, this type of statement may reinforce anxieties and fears in a client who is already feeling insecure and scared about the cognitive changes, leading to anger and possible aggression. [C. "You might fall if you wander in the dark."] This option provides little reorientation information. [D. "You should not leave your room without assistance."] This statement has a paternalistic tone and seems to penalize the client. This type of statement may cause the client to get angry, leading to escalating negative behaviors. Educational objective:Appropriate communication techniques to assist a client with dementia while avoiding anxiety and other negative behaviors include reorientation in the earlier stage of dementia and validation in the later stage of dementia.

The nurse cares for a client with type 2 diabetes mellitus and hemoglobin A1C results of 8% at an outpatient health clinic. Which statement by the nurse will best address these results? A. "It is important for us to review the signs and symptoms of a hypoglycemic reaction." B. "Let's review your diet, exercise, and medication regimen over the past 2-3 months." C. "Please describe what you have eaten in the last 24-48 hours." D. "You should fast for at least 8 hours prior to your morning blood work."

Correct Answer: B. "Let's review your diet, exercise, and medication regimen over the past 2-3 months." Hemoglobin A1C is a diagnostic test used to measure the percentage of glycosylated hemoglobin in the blood. A normal hemoglobin A1C is 4%-6% in clients without diabetes; the goal is to keep the level <7% in clients with diabetes. The A1C test measures blood glucose control over a period of 2-3 months; higher measurements indicate higher glycemic levels. High levels may indicate poor adherence to the recommended diet and exercise plan or ineffective antihyperglycemic medication regimen. It is important for the nurse to review the diet, exercise, and medication plan with the client who has a high hemoglobin A1C. Incorrect Answers: [A. "It is important for us to review the signs and symptoms of a hypoglycemic reaction."] Although it is important to review signs and symptoms of hypoglycemia with all clients with diabetes, this statement does not address the elevated hemoglobin A1C. [C. "Please describe what you have eaten in the last 24-48 hours."] A diet recall of the last 24-48 hours will not give the nurse adequate information on possible causes of an elevated hemoglobin A1C as this test measures glycemic control over 2-3 months. [D. "You should fast for at least 8 hours prior to your morning blood work."] A hemoglobin A1C may be tested when the client is not fasting. Educational objective:Hemoglobin A1C is a diagnostic test used to measure the percentage of glycosylated hemoglobin in the blood over a period of 2-3 months. A normal hemoglobin A1C is 4%-6% in clients without diabetes; the goal is to keep the level <7% in clients with diabetes.

An adult diagnosed with celiac disease 3 weeks ago was placed on a gluten-free diet. The client returns for ambulatory care follow-up, reports continuation of symptoms, and does not seem to be responding to therapy. Which is the best response by the nurse? A. "It should take about 6-8 weeks before your symptoms improve." B. "Tell me what you had to eat yesterday." C. "We will refer you to the dietitian." D. "You must not be following your diet."

Correct Answer: B. "Tell me what you had to eat yesterday." The most common reason that clients with celiac disease do not respond to a gluten-free diet is that they have not completely eliminated gluten from their food intake. This client's recent intake must be assessed to determine if the client ate foods containing gluten. Incorrect Answers: [A. "It should take about 6-8 weeks before your symptoms improve."] Most clients experience dramatic relief of gastrointestinal symptoms within a few days of eliminating gluten from their diet. [C. "We will refer you to the dietitian."] Referral to a dietitian is an appropriate intervention. However, the nurse must first explore why the client is not responding to therapy. [D. "You must not be following your diet."] This is a nontherapeutic response that "blames" the client for symptoms. Furthermore, this conclusion cannot be reached without an assessment of the client's intake. Educational objective:When a client with celiac disease does not experience symptom relief after being on a gluten-free diet, it is most important for the nurse to assess the underlying cause. The most common reason for persistent symptoms is failure to follow the strict gluten-free diet.

The nurse administers 8 units of regular insulin subcutaneously at 11:30 AM to a client with type 1 diabetes mellitus and serves the client lunch 30 minutes later. The client eats a few bites, becomes nauseated, and is unable to finish the meal. When is the client at highest risk for experiencing an insulin-related hypoglycemic reaction? A. 12:30 PM B. 2:00 PM C. 5:00 PM D. 6:00 PM

Correct Answer: B. 2:00 PM Insulin is a medication commonly used to control and lower blood glucose levels in clients with diabetes mellitus. The nurse must be familiar with the various insulin types and their times of peak effect, which are the periods of highest risk for hypoglycemic events. Regular insulin is a short-acting insulin that reaches the peak effect within 2-5 hours after subcutaneous administration. Therefore, clients who receive regular insulin subcutaneously at 11:30 AM are at highest risk for hypoglycemia between 1:30 PM and 4:30 PM. Incorrect Answers: [A. 12:30 PM] Rapid-acting insulins (eg, lispro, aspart) take peak effect in 30 minutes to 3 hours. Clients who receive rapid-acting insulin at 11:30 AM would be most at risk for hypoglycemia from 12:00-2:30 PM. [C. 5:00 PM] Both insulin NPH, an intermediate-acting insulin, and insulin detemir, a long-acting insulin, have peak effect times that may cause hypoglycemia at 5-6 PM in clients who receive the medication at 11:30 AM. [D. 6:00 PM] Both insulin NPH, an intermediate-acting insulin, and insulin detemir, a long-acting insulin, have peak effect times that may cause hypoglycemia at 5-6 PM in clients who receive the medication at 11:30 AM. Educational objective:Insulin is a medication used to control and lower blood glucose levels in clients with diabetes mellitus. Peak effect times vary according to insulin type and represent the time of highest risk for hypoglycemic events. Regular insulin, a short-acting insulin, reaches peak effect 2-5 hours after subcutaneous administration.

When no changes are made to the diet or prescribed insulin, which client with type 1 diabetes mellitus does the nurse anticipate having the highest risk of developing hypoglycemia? A. 29-year-old with new onset of influenza B. 40-year-old experienced cyclist who rides an extra 10 miles (16 km) C. 65-year-old with cellulitis of the right leg D. 72-year-old with emphysema who is taking prednisone

Correct Answer: B. 40-year-old experienced cyclist who rides an extra 10 miles (16 km) Hypoglycemia (blood glucose <70 mg/dL [3.9 mmol/L]) is an acute and sometimes life-threatening complication of diabetes mellitus. It occurs when the proportion of insulin exceeds the glucose in the blood. Counterregulatory hormones (eg, epinephrine) are then released and the autonomic nervous system is activated, causing symptoms of hypoglycemia such as sweating, tremor, and hunger. Aerobic exercise typically lowers blood glucose levels. As muscles use up glucose, the liver is unable to produce enough glucose to keep up with the demand. Even an experienced exerciser should check blood glucose levels before, during, and after exercise, and also carry a carbohydrate drink or snack in case of a hypoglycemic episode. Clients with an acute illness (eg, influenza, cellulitis) are more likely to experience hyperglycemia. Increased glucose levels occur due to the physiological stress response caused by infection. Hyperglycemia is also a side effect of prednisone. Educational objective:Aerobic exercise typically lowers blood glucose levels as glucose production in the liver fails to keep up with elevated glucose uptake by the muscles at work.

A client tells the nurse of wanting to lose 20 lb (9 kg) in time for the client's daughter's wedding, which is 16 weeks away. How many calories (kcal) will the client have to eliminate from the diet each day to meet this goal? A. 450 kcal/day B. 625 kcal/day C. 860 kcal/day D. 1,000 kcal/day

Correct Answer: B. 625 kcal/day A reduction or energy expenditure of 3,500 calories (kcal) will result in a weight loss of 1 lb (0.45 kg). To lose 20 lb (9 kg), the client needs to reduce intake by a total of 70,000 kcal (3500 kcal x 20 lb [9 kg] = 70,000 kcal). Over a period of 16 weeks, this would require a daily reduction of: 625 kcal (70,000 kcal / [16 weeks x 7 days] = 625 kcal/day) Adding an exercise regimen to the client's daily routine would facilitate additional weight loss and/or reduce the need for severe caloric restriction. Incorrect Answers: [A. 450 kcal/day] Reducing intake by 450 kcal/day over 16 weeks would result in a weight loss of 14.5 lb (6.5 kg). [C. 860 kcal/day] Reducing intake by 860 kcal/day over 16 weeks would result in a weight loss of 27.5 lb (12.4 kg). [D. 1,000 kcal/day] Reducing intake by 1000 kcal/day over 16 weeks would result in a weight loss of 32 lb (14.5 kg). Educational objective:A reduction or energy expenditure of 3500 calories (kcal) will result in a weight loss of 1 lb.

The nurse assessing a client notices pearly white plaque-like lesions on the mouth mucosa. The nurse understands that which client is at highest risk for oral candidiasis? A. A client with asthma who uses an albuterol nebulizer once a day B. A client receiving intravenous broad-spectrum antibiotics daily C. A teenage client with braces who drinks several sugary drinks daily D. An elderly client with poor oral hygiene and inadequate nutrition

Correct Answer: B. A client receiving intravenous broad-spectrum antibiotics daily Oral candidiasis or thrush (moniliasis), is an infection of the mucous membranes generally caused by the yeastlike fungus Candida albicans. The fungus causes pearly, "milk-curd" lesions on the oral or laryngeal mucosa that may bleed when removed. Immunosuppressed individuals such as those taking corticosteroid medications, clients undergoing chemotherapy or radiation, or clients with immune deficiency states (eg, AIDS) have an increased incidence. Clients receiving prolonged or high-dose antibiotic treatment are at increased risk as the normal microbial flora of the mouth is reduced, allowing other opportunistic infections to arise. Individuals with dentures and infants also commonly experience monilial infections. Treatment is antifungal medications (eg, nystatin) and proper oral hygiene. Incorrect Answers: [A. A client with asthma who uses an albuterol nebulizer once a day] Inhaled beta-2 agonists (eg, albuterol) do not increase the risk for fungal infections. However, individuals taking an inhaled corticosteroid (eg, budesonide, fluticasone) are at increased risk for oral candidiasis. To reduce this risk, the client should rinse the mouth after each inhaled dose and maintain good oral hygiene. [C. A teenage client with braces who drinks several sugary drinks daily] Proper oral hygiene and nutrition are important in prevention of oral candidiasis. However, the client with braces or poor hygiene and inadequate nutrition is at lower risk than one who is immunosuppressed or taking antibiotics. [D. An elderly client with poor oral hygiene and inadequate nutrition] Proper oral hygiene and nutrition are important in prevention of oral candidiasis. However, the client with braces or poor hygiene and inadequate nutrition is at lower risk than one who is immunosuppressed or taking antibiotics. Educational objective:Immunosuppressed clients (eg, taking steroids, undergoing chemotherapy or radiation, with immunodeficient states) and those taking prolonged or high-dose antibiotics are at increased risk of oral candidiasis. Elderly clients with dentures are also at high risk. Infection is treated with antifungals (eg, nystatin) and proper oral hygiene.

A client with type 1 diabetes has a prescription for 30 units of insulin glargine at bedtime. Fingerstick blood glucose measurements are prescribed before meals and at bedtime with regular insulin based on a sliding scale. At 9 PM, the client's blood glucose measurement is 180 mg/dL (10.0 mmol/L). What action should the nurse take? Sliding Scale Blood Glucose Levels | Regular insulin dose < 150 mg/dL (<8.3 mmol/L) | 0 units 150 - 199 mg/dL (8.3 - 11.0 mmol/L) | 2 units 200 - 249 mg/dL (11.1 - 13.8 mmo/L) | 4 units 250 - 299 mg/dL (13.9 - 16.6 mmol/L) | 6 units ≥300 mg/dL (≥16.7 mmol/L) | 8 units and notify health care provider A. Administer 30 units of glargine; give the client a snack, then administer 2 units of regular insulin B. Administer 30 units of glargine and 2 units of regular insulin in 2 different injections C. Mix 30 units of glargine with 2 units of regular insulin in the same syringe, drawing up the glargine first D. Mix 30 units of glargine with 2 units of regular insulin in the same syringe, drawing up the regular insulin first

Correct Answer: B. Administer 30 units of glargine and 2 units of regular insulin in 2 different injections Insulin is a medication commonly used to control and lower blood glucose levels in clients with diabetes mellitus. Clients may require a combination of long-acting insulin (eg, glargine) with rapid- (eg, lispro) or short-acting (eg, regular) insulin to manage glucose levels. The different onsets, peaks, and durations mimic the body's natural insulin levels and enhance glycemic control. Insulin glargine, a long-acting (basal) insulin, has no peak and may last 24 hours or longer. Short-acting insulins peak 2-5 hours after administration and last approximately 5-8 hours. Regular or rapid-acting insulins may be given on a sliding scale at prescribed intervals (eg, before meals and at bedtime) and are dosed based on the client's blood glucose measurement. Insulin glargine and regular insulin may be safely given concurrently due to the differences in onset, peak, and duration. Incorrect Answers: [A. Administer 30 units of glargine; give the client a snack, then administer 2 units of regular insulin] Insulin glargine has no peak effect and should not potentiate hypoglycemia, whereas regular insulin may cause hypoglycemia. Concurrent administration of regular insulin with insulin glargine will not increase the risk of hypoglycemia as each medication has a different onset, peak, and duration; therefore, a snack is not required. [C. Mix 30 units of glargine with 2 units of regular insulin in the same syringe, drawing up the glargine first] Insulin glargine should not be mixed in a single syringe with any other insulin as the mixture may alter the pharmacodynamics of the drug. [D. Mix 30 units of glargine with 2 units of regular insulin in the same syringe, drawing up the regular insulin first] Insulin glargine should not be mixed in a single syringe with any other insulin as the mixture may alter the pharmacodynamics of the drug. Educational objective:Sliding-scale regular insulin can be administered safely with scheduled insulin glargine without potentiating hypoglycemia if both medications are properly dosed and administered as separate injections. Insulin glargine should not be mixed in a syringe with any other insulin.

The nurse is assisting with the care of an adolescent diagnosed with type 1 diabetes. The client has hot, dry skin and a glucose level of 350 mg/dL (19.4 mmol/L). Arterial blood gases show a pH of 7.27. STAT serum chemistry labs have been ordered. Cardiac monitoring reveals a sinus rhythm with peaked T waves, and the client has minimal urine output. What does the nurse anticipate as the next priority action? A. Administer IV regular insulin B. Administer normal saline infusion C. Obtain client's urine for urinalysis D. Request a potassium infusion prescription

Correct Answer: B. Administer normal saline infusion This client has diabetic ketoacidosis, and all clients with this condition experience dehydration due to osmotic diuresis. Prompt and adequate fluid therapy restores tissue perfusion and suppresses the elevated levels of stress hormones. The initial hydrating solution is a normal saline (0.9%) infusion. Incorrect Answers: [A. Administer IV regular insulin] Insulin therapy should be started after the initial rehydration bolus as serum glucose levels fall rapidly after volume expansion. [C. Obtain client's urine for urinalysis] Urinalysis is important but not a priority. [D. Request a potassium infusion prescription] Potassium should never be given until serum potassium is normal or low and urinary voiding is observed. The peaked T waves indicate hyperkalemia in this client. Clients with insulin deficiency frequently have increased serum potassium due to the extracellular shift despite a total body potassium deficit from urinary losses. Once insulin is given, serum potassium levels drop rapidly, often requiring potassium replacement. Potassium is never given as a rapid IV bolus as cardiac arrest may result. Educational objective:Clients with diabetic ketoacidosis and in a hyperosmolar hyperglycemic state require IV normal saline as a priority due to severe dehydration. Once fluids are given as a bolus, insulin is initiated. The serum potassium levels can be elevated in the initial stages despite low total body potassium. Potassium repletion is started once the elevated serum potassium levels are normal or low.

The nurse is caring for a client with an exacerbation of chronic obstructive pulmonary disease and pneumonia. The nurse notes that the client has become disoriented and restless and becomes concerned that the client may have impaired oxygenation due to poor secretion clearance. Which is the nurse's priority action? A. Administer lorazepam 1 mg IM B. Administer oxygen using venturi mask C. Maintain IV normal saline infusion at prescribed rate of 125 mL/hr D. Place the head of the bed in semi-Fowler position

Correct Answer: B. Administer oxygen using venturi mask Chronic obstructive pulmonary disease (COPD) is a progressive, inflammatory lung disease that causes hypersecretion of mucus and airway structure changes that reduce expiratory airflow and impair oxygen and carbon dioxide (CO2) exchange. Clients with COPD have chronic hypoxia and hypercarbia, which alters the sensitivity of chemical receptors in the brain and causes breathing to be triggered by low oxygen levels (ie, hypoxic drive) rather than high CO2 levels. Clients with COPD who develop symptomatic hypoxia (eg, altered mental status, restlessness, respiratory distress) require prompt administration of supplemental oxygen to prevent progression to respiratory failure and arrest. Controlled-flow devices (eg, venturi mask) are preferred as they allow delivery of precise oxygen levels, reducing the risk of delivering too much oxygen and suppressing the respiratory drive. Incorrect Answers [A. Administer lorazepam 1 mg IM] Anxiety and restlessness in clients with COPD often relate to worsening hypoxia. If the client remains anxious after resolving hypoxia, low-dose anxiolytic medications (eg, lorazepam) may be used. [C. Maintain IV normal saline infusion at prescribed rate of 125 mL/hr] Administering fluids and elevating the head of the bed (eg, semi-Fowler position) promote mobilization of secretions and improve work of breathing. However, clients with symptomatic hypoxia require immediate interventions (eg, supplemental oxygen) first. [D. Place the head of the bed in semi-Fowler position] Administering fluids and elevating the head of the bed (eg, semi-Fowler position) promote mobilization of secretions and improve work of breathing. However, clients with symptomatic hypoxia require immediate interventions (eg, supplemental oxygen) first. Educational objective:Clients with chronic obstructive pulmonary disease who develop symptomatic hypoxia (eg, altered mentation, restlessness) require prompt oxygen administration with a controlled-flow device (eg, venturi mask) to prevent respiratory failure and arrest.

The nurse is caring for a client with hyperosmolar hyperglycemic state. The nurse understands which characteristic is most consistent with HHS? A. Abdominal pain B. Altered level of consciousness C. History of type 1 diabetes D. Kussmaul respirations

Correct Answer: B. Altered level of consciousness Hyperosmolar hyperglycemic state (HHS) is a serious complication of type 2 diabetes mellitus (DM). In HHS, the pancreas can produce enough insulin to prevent the breakdown of fats that would result in diabetic ketoacidosis (DKA) but not enough to prevent extreme hyperglycemia, osmotic diuresis, and extracellular fluid deficit. Therefore, DKA is usually associated with type 1 DM, whereas HHS is usually associated with type 2 DM. In HHS, hyperglycemia develops more gradually than in DKA as the pancreas of clients with type 2 DM still produces some insulin. Hyperglycemic symptoms associated with HHS may not be recognized until the glucose level reaches >600 mg/dL. This eventually causes neurological manifestations such as blurry vision, lethargy, altered level of consciousness, and progression to coma. Incorrect Answers: [A. Abdominal pain] The pancreas of clients with type 1 DM is unable to produce endogenous insulin. Without insulin, the body is unable to break down glucose for energy and instead breaks down fat, a process that produces ketones and acidosis (ie, DKA). Symptoms of DKA include Kussmaul respirations (hyperventilation) and abdominal pain. [C. History of type 1 diabetes] The pancreas of clients with type 1 DM is unable to produce endogenous insulin. Without insulin, the body is unable to break down glucose for energy and instead breaks down fat, a process that produces ketones and acidosis (ie, DKA). Symptoms of DKA include Kussmaul respirations (hyperventilation) and abdominal pain. [D. Kussmaul respirations] The pancreas of clients with type 1 DM is unable to produce endogenous insulin. Without insulin, the body is unable to break down glucose for energy and instead breaks down fat, a process that produces ketones and acidosis (ie, DKA). Symptoms of DKA include Kussmaul respirations (hyperventilation) and abdominal pain. Educational objective:Hyperosmolar hyperglycemic state is associated with type 2 diabetes mellitus. Because the pancreas of clients with type 2 diabetes produces some insulin, severe hyperglycemia develops gradually. Manifestations include severe hyperglycemia, blurry vision, and altered level of consciousness.

The practical nurse is assisting the registered nurse during admission of a client with heart failure-related fluid overload. Which action should be completed first? A. Administer oxygen B. Assess the client's breath sounds C. Initiate cardiac monitoring D. Insert a peripheral IV catheter

Correct Answer: B. Assess the client's breath sounds The client being admitted for heart failure-related fluid overload is likely to have dyspnea (difficulty breathing), orthopnea (labored breathing in a supine position), and paroxysmal nocturnal dyspnea (waking suddenly with difficulty breathing). The assessment phase of the nursing process must come before intervention and should be prioritized using the ABCs: airway, breathing, and circulation. Therefore, the nurse should first assess the client's breath sounds. Rales or "crackles" may be auscultated in the lungs as a result of pulmonary congestion. Incorrect Answers: [A. Administer oxygen] This client's current respiratory status (ie, breath sounds, oxygen saturation) must be evaluated prior to giving oxygen. Oxygen saturation should be assessed on admission and every 4-6 hours based on hospital protocol; oxygen is subsequently administered based on client needs. [C. Initiate cardiac monitoring] It is appropriate for this client to have continuous cardiac monitoring that can alert staff to life-threatening rhythms (eg, ventricular tachycardia) if they occur. However, the client's respiratory status should be assessed first. [D. Insert a peripheral IV catheter] This client will require IV administration of diuretics, such as furosemide, to reduce excess fluid volume and preload. A peripheral IV catheter should be placed, but assessment of the client's current status takes priority. Educational objective:The client with heart failure-related fluid overload usually has respiratory symptoms. Assessment of the client's breathing is the highest priority and should be performed before assessment of cardiac rhythm and interventions such as giving oxygen and placing an IV line for diuretic administration.

Which appearance of a stoma immediately after colostomy requires that the practical nurse contact the supervising registered nurse immediately? A. Brick red with slight moisture B. Dusky with moderate edema C. Pink with slight oozing of blood D. Rosy with no stool produced

Correct Answer: B. Dusky with moderate edema Inadequate blood supply can cause a change in the stoma color. Any indications of poor vascularity, including pale, dusky, or cyanotic color changes, require immediate notification of the supervising registered nurse and health care provider. Surgical intervention is required to prevent ischemia and necrosis. Incorrect Answers: [A. Brick red with slight moisture] A colostomy is a surgical procedure that creates an opening (stoma) in the abdominal wall for stool to bypass an obstructed or diseased portion of the colon. The stoma should be pink to brick red, indicating vascularity and viability [C. Pink with slight oozing of blood] Minor bleeding and oozing may occur, and mild to moderate swelling is normal for 2-3 weeks after surgery. [D. Rosy with no stool produced] In the immediate postoperative period, stool will be absent. If the bowel is cleansed prior to surgery, draining of stool will be delayed by several days. Otherwise, stool appears when peristalsis resumes. Educational objective:A healthy stoma has the characteristics of mucosal tissue and should appear vascular and moist. Indications of decreased blood supply (pale, dusky, or cyanotic color changes) should be reported to the registered nurse and health care provider immediately.

A client has just returned from the cardiac catheterization laboratory for a permanent pacemaker placement. How should the nurse document the rhythm on the client's cardiac monitor? A. Atrial paced rhythm B. Atrioventricular paced rhythm C. Biventricular paced rhythm D. Ventricular paced rhythm

Correct Answer: B. Atrioventricular paced rhythm An atrioventricular pacemaker (also known as a sequential or dual chamber pacemaker) paces the right atrium and right ventricle in sequence. The ECG will have 2 pacer spikes, one before the P wave and one before the QRS complex. The P wave following the atrial pacer spike may be normal or abnormal appearing. The QRS complex following the ventricular pacer spike is typically wide and distorted. An atrioventricular pacemaker can improve synchrony between the atria and ventricles. It may be implanted in the client with bradycardia, heart block, or cardiomyopathy. Incorrect Answers: [A. Atrial paced rhythm] An atrial paced rhythm would have a pacer spike before the P wave only. The P wave may appear normal or abnormal; the QRS complex will appear normal. [C. Biventricular paced rhythm] Biventricular pacemakers (also known as sequential biventricular pacemakers) generate impulses in both ventricles. Two ventricular pacing spikes may be seen on the ECG, and one spike may appear after the beginning of the QRS complex. [D. Ventricular paced rhythm] A ventricular paced rhythm would only have a pacer spike prior to a wide QRS complex. Impulses are generated in only one ventricle (typically the right ventricle). Educational objective:An atrioventricular pacemaker (also known as sequential or dual chamber) paces the right atrium and right ventricle in sequence. Two pacer spikes are visible on the ECG, one prior to the P wave and a second prior to the QRS complex. Atrioventricular pacemakers improve cardiac synchrony between the atria and ventricles.

A client comes to the emergency department with severe dyspnea and a cough. Vital signs are temperature 99.2 F (37.3 C), blood pressure 108/70 mm Hg, heart rate 88/min, and respirations 24/min. The client has a history of chronic obstructive pulmonary disease (COPD) and chronic heart failure. Which diagnostic test will be most useful to the nurse in determining if this is an exacerbation of heart failure? A. Arterial blood gases (ABGs) B. B-type natriuretic peptide (BNP) C. Cardiac enzymes (CK-MB) D. Chest x-ray

Correct Answer: B. B-type natriuretic peptide (BNP) BNP is a peptide that causes natriuresis. B-type natriuretic peptides are made, stored, and released primarily by the ventricles. They are produced in response to stretching of the ventricles due to the increased blood volume and higher levels of extracellular fluid (fluid overload) that accompany heart failure. Elevation of BNP >100 pg/mL helps to distinguish cardiac from respiratory causes of dyspnea. Incorrect Answers: [A. Arterial blood gases (ABGs)] ABGs will be helpful in determining the client's oxygenation status and acid-base balance but will not determine whether the cause of the dyspnea is cardiac or respiratory. [C. Cardiac enzymes (CK-MB)] CK-MB is a cardiospecific isozyme that is released in the presence of myocardial tissue injury. Elevations are highly indicative of a myocardial infarction but not specific for heart failure. [D. Chest x-ray] A chest x-ray can show heart enlargement in the case of heart failure and may show infiltrations in the lungs. Pneumonia can also exacerbate COPD and can be confused with heart failure infiltrates. Chest x-ray is not as specific to heart failure as the BNP lab test. Educational objective:Elevation of BNP to >100 pg/mL is seen in heart failure. It aids in the assessment of the severity of heart failure and helps distinguish cardiac from respiratory causes of dyspnea.

A nurse is reinforcing information on dietary management to a group of clients with newly diagnosed type 2 diabetes. Which meal represents the best adherence to the principles of and recommendations for diabetic meal planning? A. Baked tilapia with tomato salsa, steamed white rice B. Black bean chili with brown rice, mixed greens salad C. Grilled chicken breast with baked French fries D. Hamburger on a whole wheat bun with lettuce and tomato

Correct Answer: B. Black bean chili with brown rice, mixed greens salad Black bean chili with brown rice and salad is a low-fat, low glycemic index, high-fiber meal. The other meals do contain some acceptable items but none are the best option due to their low-fiber content and high glycemic index. The American Diabetes Association recommends a simple "Create My Plate" method for meal planning. Specific dietary recommendations include the following: 1. Monitor carbohydrate intake 2. Manage caloric intake if weight loss is desired. 3. Consume high-fiber foods (30 - 35 g of fiber per day), including whole grains, legumes, fruits, vegetables, and low-fat dairy products. 4. Use monounsaturated fats, limit use of saturated fat, and eliminate trans fatty acids. 5. Choose foods with a low glycemic index 6. Consume <300 mg per day of total cholesterol 7. Reduce sodium intake 8. Limit take of foods containing sucrose 9.Limit intake of alcoholic beverages Incorrect Answers: [A. Baked tilapia with tomato salsa, steamed white rice] Although tilapia is a good selection of lean protein, the white rice has minimal fiber and a high glycemic index. [C. Grilled chicken breast with baked French fries] Although grilled chicken is a good selection of lean protein, the baked French fries have minimal fiber and a high glycemic index and are high in sodium. [D. Hamburger on a whole wheat bun with lettuce and tomato] Although the whole wheat bun, lettuce, and tomato provide limited amounts of fiber, the hamburger contains saturated fat, which must be limited in the diabetic diet. Educational objective:Clients with diabetes should eat foods with a low glycemic index and high fiber content. Saturated fats and sodium should be restricted.

During a screening clinic, the practical nurse working with the registered nurse performs a health assessment on several adult clients. Which finding is most important for the practical nurse to advise the client to follow-up with the health care provider? A. BMI of 23 kg/m2 B. Brownish skin thickening on the neck C. Fasting total cholesterol of 180 mg/dL (4.7 mmol/L) D. Round 3 x 3 mm pale pink mole

Correct Answer: B. Brownish skin thickening on the neck Acanthosis nigricans is a skin disorder characterized by the presence of symmetric, hyperpigmented velvety plaques located in flexural and intertriginous regions of skin (axilla, neck). Skin tags (acrochordons) are commonly present on regions affected by acanthosis nigricans. Recognition of these skin disorders is important due to their association with insulin resistance. The client should be referred to the health care provider for evaluation of undiagnosed diabetes mellitus and/or metabolic syndrome. Incorrect Answers: [A. BMI of 23 kg/m2] A BMI of 18.5-24.9 kg/m2 indicates a normal weight. [C. Fasting total cholesterol of 180 mg/dL (4.7 mmol/L)] A fasting total cholesterol <200 mg/dL (5.2 mmol/L) is normal. [D. Round 3 x 3 mm pale pink mole] Although any change in or growth of a mole should be reported, a pale pink or brown round mole <5 mm is typically a normal finding. Educational objective:Acanthosis nigricans is a skin condition characterized by dark, thick, velvety skin, commonly seen in the axillae, flexures, or on the neck. It is indicative of insulin resistance and requires further evaluation. Skin tags (acrochordons) are commonly present on regions affected by acanthosis nigricans.

A client was struck on the head by a baseball bat during a robbery attempt. The nurse gives shift report to the oncoming nurse and conveys that the client's current Glasgow Coma Scale score is a "10." Which other information is most important for the reporting nurse to include? A. Client's blood pressure was 120/80 mm Hg and pulse was 82/min recently B. Client's Glasgow Coma Scale score was "11" one hour ago C. Client believes that the current surroundings are a racetrack D. Client is allergic to penicillin and vancomycin

Correct Answer: B. Client's Glasgow Coma Scale score was "11" one hour ago The Glasgow Coma Scale (GCS) quantifies the level of consciousness in a client with acute brain injury by measuring eye opening (alertness), verbal response (orientation), and motor response (eg, obeying a command, frontal lobe function). The maximum score on the GCS is 15 and the lowest is 3. If a client is trending for deterioration, this should always be noted in neurological assessments. A numerical decline of a single point within 1 hour is significant. Incorrect Answers: [A. Client's blood pressure was 120/80 mm Hg and pulse was 82/min recently] This client's vital signs are within normal limits and are not significant. It would be more important to report if the client exhibits Cushing's triad (bradycardia, bradypnea/Cheyne-Stokes, and widening pulse pressure) or to provide a brief summary of overall vital signs (eg, "normal"); exact readings are accessible and can be recited if the oncoming nurse needs to know them. However, reporting the negative neurological trend in the GCS score is the priority. [C. Client believes that the current surroundings are a racetrack] Orientation to place is part of the GCS score (under best verbal). The total score and the negative trend are more indicative of the client's condition than any individual GCS component. [D. Client is allergic to penicillin and vancomycin] Although it is important to be aware of allergies, the oncoming nurse can find that information on the chart if these medications are ordered. The main concern is blunt head trauma, not infection. Therefore, it is unlikely that the oncoming nurse will be required to know this information and need to administer antibiotics. Educational objective:It is a priority to report a negative neurological trend as evidenced by a decreasing Glasgow Coma Scale score in a client with blunt head trauma.

The nurse prepares to administer a dose of radioactive iodine (RAI) to a 39-year-old female client with Graves' disease. Which action is most important for the nurse to take? A. Ask client when her last menstrual cycle occurred B. Confirm pregnancy test result is negative C. Obtain a baseline assessment of the mouth and throat D. Teach the client the signs and symptoms of hypothyroidism

Correct Answer: B. Confirm pregnancy test result is negative RAI is the primary treatment for nonpregnant adults with hyperthyroid disorders such as Graves' disease (a type of autoimmune hyperthyroid disease). The use of RAI is contraindicated in pregnancy and could cause harm to a fetus. Incorrect Answers: [A. Ask client when her last menstrual cycle occurred ] Pregnancy results should therefore be confirmed using a valid pregnancy test in all clients who still have menstrual cycles rather than using a subjective form of assessment such as asking when the last menstrual period occurred. [C. Obtain a baseline assessment of the mouth and throat] Radiation thyroiditis and parotitis, which cause dryness and irritation to the mouth, may occur after RAI treatment. A baseline assessment is helpful but is not the most important action listed. The nurse can teach the client to take sips of water frequently or to use a salt and soda gargle solution 3-4 times daily to relieve these symptoms. [D. Teach the client the signs and symptoms of hypothyroidism] RAI damages or destroys the thyroid tissue, thereby limiting thyroid secretion, and can result in hypothyroidism. Clients need to take thyroid supplementation (levothyroxine) for life. Because these symptoms are delayed, this teaching can occur before or after the procedure. It is not as important as assessing pregnancy status. Educational objective:RAI destroys the thyroid gland, making clients permanently hypothyroid and requiring life-long thyroid supplements. In female clients, a nonpregnant status should be confirmed with a valid pregnancy test prior to administering RAI. RAI is contraindicated in pregnancy and may cause harm to a fetus.

The clinic nurse is caring for a client who had cataract surgery with intraocular lens implantation 2 days ago. Which client report requires priority intervention? A. Blurry vision in the affected eye B. Constipation C. Itching in the affected eye D. Sleeping on 2 pillows at night

Correct Answer: B. Constipation Following cataract surgery, the client will be instructed that, for several days (or until approved by the surgeon), activities that may increase intraocular pressure should be avoided to decrease the risk of damage to sutures or surgical sites. These include bending (eg, vacuuming floors, playing golf), lifting more than 5 lb, sneezing, coughing, rubbing or placing pressure on the eye, or straining during a bowel movement. The nurse should encourage this client to increase fluids and fiber in the diet as well as consider an over-the-counter stool softener or laxative. Incorrect Answers: [A. Blurry vision in the affected eye] It may take 1-2 weeks before visual acuity is improved. [C. Itching in the affected eye] It is common for the client to experience itching ("sand" in the eye), photophobia, and mild pain for several days following surgery. Purulent drainage, increased redness, and severe pain should be reported. [D. Sleeping on 2 pillows at night] Sleeping on 2 pillows will elevate the head and decrease intraocular pressure. Educational objective:Following cataract surgery, the client should be instructed to avoid for several days those activities that may increase intraocular pressure, such as coughing, sneezing, lifting over 5 lb, bending, rubbing the eye, or straining during bowel movements. It is common for the client to experience itching ("sand" in the eye), photophobia, and mild pain during this time frame.

A client with type 1 diabetes is prescribed NPH insulin before breakfast and dinner. Although the client reports feeling well, the 6 AM blood glucose level has averaged 60 mg/dL (3.3 mmol/L) over the past week. Which action is appropriate for the nurse to recommend to the client? A. Collect urine sample to check for urine ketones B. Consume a snack of milk and cereal at bedtime C. Increase carbohydrate intake at each meal D. Take only the prebreakfast dose of NPH

Correct Answer: B. Consume a snack of milk and cereal at bedtime NPH insulin is intermediate-acting with an onset of 1-2 hours, peak of 4-12 hours, and duration of 12-18 hours. Due to its long peak, hypoglycemia (blood glucose <70 mg/dL [<3.9 mmol/L]) can result from use of NPH, especially because the overnight hours (during sleep) typically represent the longest interval between meals. To prevent hypoglycemia related to an evening dose of NPH, the client should eat a bedtime snack consisting of protein and complex carbohydrates (eg, cereal with milk, crackers with peanut butter). Complex carbohydrates paired with protein provide sustained, slow release of glucose, thereby preventing hypoglycemia. Incorrect Answers: [A. Collect urine sample to check for urine ketones] Testing for urine ketones should be done as part of a sick-day plan or when evaluating for diabetic ketoacidosis. Ketosis occurs in hyperglycemia, especially when blood glucose is >180 mg/dL (>10 mmol/L). [C. Increase carbohydrate intake at each meal] Increasing the client's carbohydrate intake at each meal may disrupt glycemic control and lead to a persistent elevation of blood glucose. Elevated blood glucose can cause complications, such as diabetic ketoacidosis. [D. Take only the prebreakfast dose of NPH] The client may require an adjustment to the prescribed insulin regimen; however, it is dangerous for the client to drastically alter the prescribed dose. In addition, it is outside the nurse's scope of practice to recommend a change in the prescribed dose. Educational objective:NPH insulin is an intermediate-acting insulin that peaks in 4-12 hours. In asymptomatic clients, the best intervention to prevent low blood glucose levels related to an evening dose of NPH is to consume a bedtime snack of protein and complex carbohydrates.

The nurse is preparing to administer medications to a client with an asthma exacerbation. Which prescription should the nurse confirm with the health care provider prior to administration? Click on the exhibit button for additional information. Day — Day 1 | Day 5 Hemocrit: 37% (0.37) | 36% (0.36) Platelets: 250,000/mm*3 (250 × 10*9/L) | 96,000/mm3 (96 × 10*9/L) White Blood Cells: 9,100/mm3 (9.1 × 10*9/L) | 15,000/mm*3 (15.0 × 10*9/L) Potassium: 3.8 mEq/L (3.8 mmol/L) | 3.6 mEq/L (3.6 mmol/L) A. Albuterol 2.5 mg per nebulizer every 4 hours B. Enoxaparin 40 mg subcutaneously every 24 hours C. Methylprednisolone succinate 20 mg IV every 6 hours D. Potassium chloride 20 mEq IV every 24 hours

Correct Answer: B. Enoxaparin 40 mg subcutaneously every 24 hours A significant reduction in platelets after initiation of heparin or low-molecular-weight heparin (eg, enoxaparin [Lovenox]) therapy can indicate heparin-induced thrombocytopenia (HIT), a severe, potentially lethal complication. HIT is an immune reaction to heparin-based anticoagulants that causes a drastic decrease in platelet count (ie, ≤50% of pretreatment levels and/or platelet count <150,000/mm3 [150 × 109/L]) and a paradoxical increase in risk for arterial and venous thrombosis (eg, deep venous thrombosis, pulmonary embolism). The nurse should notify the health care provider immediately of decreased platelet levels and anticipate stopping enoxaparin therapy and initiating a nonheparin anticoagulant (eg, rivaroxaban, argatroban). Incorrect Answers: [A. Albuterol 2.5 mg per nebulizer every 4 hours] Beta-2 adrenergic agonists (eg, albuterol, salmeterol) are medications used to dilate the airways. The nurse should clarify the prescription if hypokalemia or tachycardia, common adverse effects, are present. [C. Methylprednisolone succinate 20 mg IV every 6 hours] Methylprednisolone is a glucocorticoid medication used to reduce airway inflammation in asthma. Glucocorticoids can cause an expected, transient elevation in the white blood cell count during initiation of treatment. [D. Potassium chloride 20 mEq IV every 24 hours] Potassium chloride is an electrolyte replacement drug used to prevent and treat hypokalemia (<3.5 mEq/L [3.5 mmol/L]). The nurse should clarify the prescription if hyperkalemia or kidney injury is present. This client has an additional risk for low potassium due to the continued use of albuterol. Educational objective:Heparin-induced thrombocytopenia is a reaction to heparin or low-molecular-weight heparin (eg, enoxaparin) that causes decreased platelet levels and may result in thromboembolic events. The nurse should monitor platelet counts in clients receiving enoxaparin and clarify the prescription if the platelet count decreases by >50% or is <150,000/mm*3 (150 × 10*9/L).

A client with mitral valve prolapse has been experiencing occasional palpitations, lightheadedness, and dizziness. The health care provider prescribes a beta blocker. What additional teaching should the nurse reinforce for this client? A. Avoid aerobic exercise B. Ensure you receive antibiotics prior to dental work C. Stay well hydrated and avoid caffeine D. Wear a medical alert bracelet

Correct Answer: B. Ensure you receive antibiotics prior to dental work Clients with mitral valve prolapse (MVP) may have palpitations, dizziness, and lightheadedness. Chest pain can occur, but its etiology is unknown in this client population. It may be a result of abnormal tension on the papillary muscles. Chest pain that occurs in MVP does not typically respond to antianginal treatment (eg, nitrates). Beta blockers may be prescribed to treat chest pain and palpitations. Client teaching for MVP management includes the following: — Adopt healthy eating habits and avoid caffeine as it is a stimulant and may exacerbate symptoms — Check ingredients of over-the-counter medications or diet pills for stimulants such as caffeine or ephedrine as they can exacerbate symptoms — Reduce stress and avoid alcohol use Incorrect Answers: [A. Avoid aerobic exercise] Clients should be taught to begin or maintain an exercise program, preferably aerobic exercise, to achieve optimal health. [B. Ensure you receive antibiotics prior to dental work] Although MVP may place the client at increased risk for infective endocarditis, there is no clinical evidence to support the need for prophylactic antibiotics prior to dental procedures. Antibiotic prophylaxis is indicated for clients who have prosthetic valve replacement, repaired valves, or a history of infectious endocarditis. [D. Wear a medical alert bracelet] MVP is usually a benign condition so there is no need for a medical alert bracelet. Educational objective:The nurse should teach the client with mitral valve prolapse (MVP) to stay hydrated, avoid caffeine and alcohol, exercise regularly, reduce stress, and take beta blockers as prescribed for palpitations and chest pain. Nitrates are usually not effective for MVP-related chest pain.

A nurse is collecting data on a 58-year-old client with blurred vision and reduced visual fields. The nurse finds which manifestation most concerning? A. Difficulty adjusting to dimmed lights B. Extreme eye pain C. Gradual loss of peripheral vision D. Opaque appearance of lens

Correct Answer: B. Extreme eye pain Acute angle-closure glaucoma is a form of glaucoma that requires immediate medical intervention. Glaucoma disorders are characterized by increased intraocular pressure (IOP) due to decreased outflow of the aqueous humor, resulting in compression of the optic nerve that can lead to permanent blindness. In acute angle-closure glaucoma, IOP increases rapidly and drastically, which can lead to the following manifestations: — Sudden onset of severe eye pain — Reduced central vision — Blurred vision — Ocular redness — Report of seeing halos around lights Incorrect Answers: [A. Difficulty adjusting to dimmed lights] Gradual loss of peripheral vision and difficulty adjusting to different lighting are manifestations of chronic open-angle glaucoma. Although further evaluation and treatment are necessary, this condition develops slowly and is not considered an emergency situation. [C. Gradual loss of peripheral vision] Gradual loss of peripheral vision and difficulty adjusting to different lighting are manifestations of chronic open-angle glaucoma. Although further evaluation and treatment are necessary, this condition develops slowly and is not considered an emergency situation. [D. Opaque appearance of lens] Opaque lenses are characteristic of cataracts , which are not a medical emergency. Educational objective:Manifestations of acute angle-closure glaucoma include sudden onset of severe eye pain, reduced central vision, blurred vision, ocular redness, and report of seeing halos around lights. This condition requires immediate medical intervention to reduce increased intraocular pressure and prevent permanent blindness.

The nurse is caring for a client with a history of headaches who has come to the clinic reporting a "bad migraine." The client was able to provide a full health history while waiting to be seen. Which finding is most concerning? A. Blood pressure of 136/88 mm Hg B. Flat affect and drowsiness C. Nausea and poor appetite D. Respiratory rate of 12/min

Correct Answer: B. Flat affect and drowsiness Although a client may have a history of recurring headaches (eg, migraine, tension, cluster), the nurse should not assume a headache is benign, as it may be a sign of increased intracranial pressure (ICP). Level of consciousness (LOC) is the most important, sensitive, and reliable indicator of the client's neurological status. A change in LOC (eg, confusion, drowsiness, flat affect) is the earliest sign of increased ICP or reduced cerebral blood flow (Option 2). Later signs of increased ICP include Cushing triad (eg, bradycardia, widening pulse pressure [increasing systolic/decreasing diastolic], altered respiratory pattern); fixed, dilated pupil(s); and decreased motor function. Incorrect Answers: [A. Blood pressure of 136/88 mm Hg] The client's vital signs should be monitored for changes that may occur with increased ICP; however, a mildly elevated blood pressure (eg, 136/88 mm Hg) does not take priority over an acute change in LOC. [C. Nausea and poor appetite] Headaches are frequently accompanied by nausea and poor appetite. The priority is to determine and address the cause of the decreased LOC, after which antiemetics (eg, ondansetron [Zofran]) can be administered. Active vomiting further increases ICP and puts the client with decreased LOC at risk for aspiration. [D. Respiratory rate of 12/min] A respiratory rate on the low end of normal (12-20/min) warrants further assessment and continued monitoring; however, it is not as significant as an acute change in LOC. Educational objective:A client's level of consciousness is the most sensitive and reliable indicator of neurological status. Change in the level of consciousness of any client should be reported immediately.

The nurse cares for a client with type 2 diabetes mellitus. The client is alert and oriented but also shaky, pale, and diaphoretic. The client's fingerstick blood glucose is 50 mg/dL (2.8 mmol/L). Which of the following is the best next step the nurse can take? A. Administer dextrose 50 mg IV push B. Give client 6 oz of orange juice or low-fat milk C. Inject the client with glucagon 2 mg intramuscularly D. Verify fingerstick blood glucose with serum blood draw

Correct Answer: B. Give client 6 oz of orange juice or low-fat milk This client is exhibiting signs and symptoms of a hypoglycemic reaction (evidenced by low blood glucose <70 mg/dL [3.9 mmol/L]). The client who is alert enough to ingest food/liquids orally should be given 15 grams of a simple carbohydrate such as orange juice or low-fat milk. The fat in whole milk slows the absorption process and will not treat hypoglycemia quickly enough. For this reason, low-fat milk is recommended. Fingerstick blood glucose should be checked 10-15 minutes after this. If the client shows no improvement, the simple carbohydrate can be readministered orally. Incorrect Answers: [A. Administer dextrose 50 mg IV push] Dextrose (D50 IV push), a highly concentrated sugar, and glucagon (intramuscular, subcutaneous, intravenous/gel), a hormone that stimulates glycogenolysis (conversion of glycogen to glucose), are administered to hypoglycemic clients who are unable to ingest a simple oral carbohydrate. These can cause rebound hypoglycemia by stimulating additional insulin release from the body in response to increased serum glucose levels. [C. Inject the client with glucagon 2 mg intramuscularly] Dextrose (D50 IV push), a highly concentrated sugar, and glucagon (intramuscular, subcutaneous, intravenous/gel), a hormone that stimulates glycogenolysis (conversion of glycogen to glucose), are administered to hypoglycemic clients who are unable to ingest a simple oral carbohydrate. These can cause rebound hypoglycemia by stimulating additional insulin release from the body in response to increased serum glucose levels. [D. Verify fingerstick blood glucose with serum blood draw] Fingerstick and blood glucose should be checked 10 - 15 minutes after this. Educational objective:A client who experiences a hypoglycemic reaction (evidenced by low blood glucose <70 mg/dL [3.9 mmol/L]) and is alert enough to ingest food/liquids orally should be given a simple carbohydrate (eg, orange juice, low-fat milk).

The nurse administers 15 units of aspart insulin subcutaneously at 7:00 AM to a hospitalized client with type 1 diabetes mellitus for a fasting blood glucose of 180 mg/dL (10.0 mmol/L). Which nursing action is a priority? A. Ensure that the client continues to fast for at least 30 more minutes B. Give the client breakfast within 15 minutes C. Recheck the client's blood glucose in 1 hour D. Reinforce teaching about signs and symptoms of hyperglycemia

Correct Answer: B. Give the client breakfast within 15 minutes Aspart (NovoLog) is a rapid-acting insulin with an onset of 10-15 minutes. Onset is the time it takes for the insulin to enter the circulation and begin to lower blood glucose. The peak effect for aspart insulin takes 30 minutes-3 hours, and the duration of action is 3-5 hours. It is important for the nurse to ensure that the client eats within 15 minutes of administration of aspart, lispro, or glulisine to prevent an insulin-related hypoglycemic reaction. Incorrect Answers: [A. Ensure that the client continues to fast for at least 30 more minutes ] The client is at risk for a hypoglycemic reaction if breakfast is delayed for 30 minutes. [C. Recheck the client's blood glucose in 1 hour] Rechecking the blood glucose in 1 hour is not indicated unless hypoglycemia is suspected. [D. Reinforce teaching about signs and symptoms of hyperglycemia] Reinforcing teaching is vital, but the priority is to ensure that the client eats breakfast to prevent a hypoglycemic reaction at drug onset. Educational objective:It is important for the nurse to ensure that the client eats within 15 minutes of administration of rapid-acting insulins such as aspart (NovoLog), lispro (Humalog), and glulisine (Apidra) to prevent an insulin-related hypoglycemic reaction.

The nurse is caring for a 72-year-old client 1 day postoperative colectomy. The nurse finds increased work of breathing, diminished breath sounds at the bases with fine inspiratory crackles, respirations 12/min and shallow, and pulse oximetry 96% on 2 L oxygen. There is no jugular venous distension or peripheral edema. Pain is regulated with client-controlled morphine. Which prescription does the nurse anticipate? A. Bolus dose of intravenous (IV) morphine B. Incentive spirometer C. IV furosemide D. Non-rebreather mask

Correct Answer: B. Incentive spirometer During the initial postoperative period, a client needs respiratory interventions to keep the lungs expanded and prevent atelectasis and postoperative pneumonia. Atelectasis is maximal during the second postoperative night. Clients can be asymptomatic or have increased work of breathing, hypoxia, and basal crackles. Postoperative pain, opioid respiratory depression, limited mobility, and reluctance to take a deep breath due to anticipated pain contribute to postoperative atelectasis. The elderly and postoperative abdominal and thoracic surgery clients are at increased risk for atelectasis. The incentive spirometer encourages the client to breathe deeply with maximum inspiration. This action improves ventilation and oxygenation by expanding the lungs, encourages coughing, and prevents or improves atelectasis. It is the most appropriate prescription for this client. Incorrect Answers: [A. Bolus dose of intravenous (IV) morphine ] In a client whose pain is regulated with client-controlled analgesia (eg, morphine), administration of a bolus dose is not indicated and may increase the risk for respiratory depression. [C. IV furosemide] Fine crackles in the lungs usually indicate atelectasis. The presence of coarse crackles, elevated jugular venous distension, and peripheral edema usually indicates volume overload (fluid in the alveoli). In addition, clients with fluid overload breathe at a rapid rate (tachypnea) rather than take slow, shallow breaths. IV furosemide (Lasix) is an appropriate intervention for volume overload but not for atelectasis. [D. Non-rebreather mask] As-needed oxygen may be prescribed postoperatively, especially with blood loss. A non-rebreather mask, which has 100% oxygen, is not indicated in this client as the pulse oximeter shows 96% saturation, indicating adequate oxygenation. Educational objective:The incentive spirometer is a handheld, inexpensive breathing device. It encourages the client to breathe deeply with maximum inspiration, which improves ventilation and oxygenation and encourages coughing. The incentive spirometer is used to prevent or improve atelectasis in clients who are postoperative, have respiratory problems (eg, pneumonia), or have experienced trauma.

The nurse is caring for a client with cholelithiasis and acute cholecystitis. The client suddenly vomits 250 mL of greenish-yellow emesis and reports severe right upper quadrant pain with radiation to the right shoulder. Which intervention would have the highest priority? A. Administer promethazine suppository B. Initiate NPO status C. Insert nasogastric tube set to low suction D. Obtain prescription for pain medication

Correct Answer: B. Initiate NPO status Cholelithiasis (gallstones) can obstruct the bile duct, causing acute inflammation of the gallbladder (cholecystitis). Signs and symptoms include nausea/vomiting and right upper quadrant pain that may radiate to the right shoulder. The gallbladder secretes digestive enzymes in response to food. Therefore, the highest priority intervention for an actively vomiting client with cholelithiasis is maintenance of strict NPO status to avoid gallbladder stimulation. Additional interventions include management of nausea/vomiting, pain, and fluid and electrolyte balance (eg, IV normal saline); and gastric decompression. Incorrect Answers: [A. Administer promethazine suppository] Promethazine (Phenergan) is an antiemetic that promotes the relief of nausea/vomiting, which helps minimize further fluid loss. However, this is not the highest priority. [C. Insert nasogastric tube set to low suction] A nasogastric tube set to low suction provides gastric decompression, alleviates nausea/vomiting, and promotes bowel rest. This would be the second highest priority. [D. Obtain prescription for pain medication] The nurse should treat the client's pain with prescribed medications and nonpharmacologic interventions (eg, positioning), ideally before the pain becomes severe. However, it is most important that the client remain NPO. Educational objective:The highest priority intervention for an actively vomiting client with acute cholecystitis is maintenance of strict NPO status to avoid stimulation of the gallbladder. Additional interventions include management of nausea/vomiting, pain, and fluid and electrolyte balance; and gastric decompression.

The clinic nurse is reinforcing instructions to a client who will be wearing a Holter monitor for the next 24 hours. Which instructions are important to review with the client? A. How to transmit the readings over the phone B. Keep a diary of activities and any symptoms experienced C. Refrain from exercising while wearing the monitor D. The monitor may be removed only when bathing

Correct Answer: B. Keep a diary of activities and any symptoms experienced A Holter monitor continuously records a client's electrocardiogram rhythm for 24-48 hours. Electrodes are placed on the client's chest and a portable recording unit is kept with the client. At the end of the prescribed period, the client returns the unit to the health care provider's (HCP) office. The data can then be recalled, printed, and analyzed for any abnormalities. Client instructions include the following: 1. Keep a diary of activities and any symptoms experienced while wearing the monitor so that these may later be correlated with any recorded rhythm disturbances 2. Do not bathe or shower during the test period 3. Engage in normal activities to simulate conditions that may produce symptoms that the monitor can record Incorrect Answer: [A. How to transmit the readings over the phone] The data are not generally transmitted over the phone. The client simply takes the monitor back to the HCP's office. Educational objective:The nurse should instruct the client with a Holter monitor to keep a diary of activities and any symptoms that occur while wearing it. The client should also be taught not to bathe during the testing period but to continue all other normal activities.

The nurse is caring for a client with hypothyroidism who has become lethargic and difficult to rouse. Which action is the priority? Vital signsTemperature 95.0 F (35.0 C)Blood pressure 90/50 mm HgHeart rate 50/minRespirations 6/minO2 saturation or SpO2 83% A. Administer scheduled oral levothyroxine B. Manually ventilate the client with a bag-valve-mask C. Place a warming blanket on the client D. Review client's serum thyroid laboratory results

Correct Answer: B. Manually ventilate the client with a bag-valve-mask Myxedema coma refers to a state of severe hypothyroidism causing decreased level of consciousness (eg, lethargy, stupor) that may progress to a comatose state. Myxedema coma is characterized by hypothermia, bradycardia, hypotension, and hypoventilation. Hypoventilation may occur as a result of respiratory depression, respiratory muscle fatigue, and mechanical obstruction by an edematous tongue. Clients with signs of respiratory failure (eg, slow or shallow breathing, low oxygen saturation) will require emergent endotracheal intubation and mechanical ventilation. The nurse should provide respiratory support (eg, ventilation with a bag-valve-mask) and prepare to assist with intubation. Incorrect Answer: [A. Administer scheduled oral levothyroxine] Oral medications should never be administered to clients with a decreased level of consciousness due to risk for aspiration. Clients with myxedema coma require thyroid hormone replacement with IV levothyroxine to correct the hypothyroid state. [C. Place a warming blanket on the client] A warming blanket should be placed on the client to treat hypothermia; however, respiratory support is the priority. [D. Review client's serum thyroid laboratory results] A serum thyroid panel (eg, T3, T4) is required to confirm hypothyroidism and should be monitored during treatment; however, the nurse should ensure that the client is stable before reviewing laboratory values. Educational objective:Myxedema coma is a state of severe hypothyroidism and decreased level of consciousness that may progress to coma and respiratory failure. The nurse should provide respiratory support (eg, ventilation with a bag-valve-mask) and prepare to assist with intubation.

A female client with liver cirrhosis and chronic anemia is hospitalized for a deep venous thrombosis. The client is receiving a heparin infusion and suddenly develops epistaxis. Which laboratory value would indicate that the heparin infusion needs to be turned off? A. Hematocrit of 30% B. Partial thromboplastin time of 110 seconds C. Platelet count of 80,000/mm*3 (80 x 10*9/L) D. Prothrombin time of 11 seconds

Correct Answer: B. Partial thromboplastin time of 110 seconds Heparin is an anticoagulant that helps prevent further clot formation. It is titrated based on a partial thromboplastin time (PTT). The therapeutic PTT target is 1.5-2.0 times the normal reference range of 25-35 seconds. A PTT value >100 seconds would be considered critical and could result in life-threatening side effects. Common sentinel events that result from heparin drips include epistaxis, hematuria, and gastrointestinal bleeds. Incorrect Answers: [A. Hematocrit of 30%] A normal hematocrit for a female is 35%-47% (0.35-0.47). In a client with a history of chronic anemia, a hematocrit of 30% (0.30) may be an expected finding. [C. Platelet count of 80,000/mm3 (80 x 109/L)] A normal platelet count is 150,000-400,000/mm*3 (150-400 x 10*9/L). In a client with a history of liver cirrhosis, a platelet count of 80,000/mm3 (80 x 10*9/L) would be anticipated. An episode of bleeding rarely occurs with a platelet count >50,000 mm3 (50 x 10*9/L). [D. Prothrombin time of 11 seconds] A normal prothrombin time is 11-16 seconds, and so a level of 11 seconds would not be concerning. Educational objective:Heparin infusions require close monitoring by the nurse. The partial thromboplastin time is the laboratory value required to accurately monitor the therapeutic effects of heparin.

An 86-year-old client with diabetes and gastroparesis has had repeated hospitalizations for aspiration pneumonia following a stroke and is now hospitalized with altered level of consciousness. Which nursing action is most appropriate to decrease the client's risk for developing aspiration pneumonia? A. Assessing client's breath sounds every 2 hours B. Placing client in the side lying position in bed C. Titrating client's oxygen to maintain saturation ≥93% D. Turning and repositioning the client every 2 hours

Correct Answer: B. Placing client in the side lying position in bed Clients with decreased level of consciousness may not be alert enough to protect their own airways; therefore, a side lying or lateral position is used to decrease the risk for developing aspiration pneumonia. If vomiting were to occur, this position promotes drainage of emesis out of the mouth instead of down the pharynx where it can be aspirated into the lungs. Maintaining an upright position during and after meals will allow remaining food particles to clear from the pharynx. Incorrect Answers: [A. Assessing client's breath sounds every 2 hours] Assessing breath sounds can help identify the presence of pneumonia (eg, diminished, bronchial, crackles) but does not prevent the client from aspirating. [C. Titrating client's oxygen to maintain saturation ≥93%] Titrating oxygen to maintain saturation ≥93% provides for adequate oxygenation but does not prevent the client from aspirating. [D. Turning and repositioning the client every 2 hours] Turning and repositioning every 2 hours helps to prevent stasis of secretions in the lungs but does not prevent the client from aspirating. Educational objective:Placing a client with a decreased level of consciousness in a position that uses gravity to help drain oropharyngeal and gastric secretions can be effective in preventing aspiration and reducing the risk for aspiration pneumonia.

An elderly client has a 17-mm induration after a tuberculin skin test. Based on this result, which statement is most accurate? A. The client has a false-positive reaction due to advanced age. B. The client has a tuberculosis infection. C. The client has active tuberculosis disease. D. The client must be isolated immediately.

Correct Answer: B. The client has a tuberculosis infection. In a heathy client, an induration >15 mm indicates a positive tuberculin skin test (TST); this means that the client was exposed to tuberculosis (TB), developed antibodies to the disease, and has a TB infection. Additional tests are needed to determine if the client has latent TB infection (LTBI) or active TB disease. Clients with LTBI are asymptomatic and cannot transmit the bacteria to others, whereas those with active TB disease usually are symptomatic and can transmit the bacteria through the air. Incorrect Answers: [A. The client has a false-positive reaction due to advanced age.] The elderly have decreased immunity and may be unable to develop antibodies that react to tuberculin; this can result in a false-negative TST reaction. [C. The client has active tuberculosis disease.] A positive reaction to TST means that the client is infected with TB bacteria that are concealed by the body's immune system and do not lead to active TB disease in most individuals. When the client has decreased immunity (eg, immunosuppression), the offending pathogen causes active TB disease. Additional diagnostic tests (eg, chest x-rays, bacteriologic sputum smear for acid-fast bacilli and culture) are needed to determine if this client has active TB disease. [D. The client must be isolated immediately.] A positive reaction indicates a TB infection only. Further evaluation and bacteriologic testing is necessary. If active TB is suspected before testing is completed, airborne transmission precautions should be initiated. Educational objective:A positive reaction to a tuberculin skin test means that a client was exposed to the tuberculosis (TB) bacteria, developed antibodies, and now has a TB infection. Additional testing is needed to determine if the client has latent TB infection or active TB disease.

A client with newly diagnosed chronic heart failure is being discharged home. Which statement(s) by the client indicate a need for further teaching by the nurse? Select all that apply. A. "I don't plan on eating any more frozen meals." B. "I plan to take my diuretic pill in the morning." C. "I will weigh myself at least every other day." D. "I'm going to look into joining a cardiac rehabilitation program." E. "Ibuprofen works best for me when I have pain."

Correct Answer: C and E Client and family education is important for those with heart failure to prevent/minimize exacerbations, decrease symptoms, prevent target organ damage, and improve quality of life. The use of any nonsteroidal anti-inflammatory drugs (NSAIDS) is contraindicated as they contribute to sodium retention, and therefore fluid retention. To monitor fluid status, clients are instructed to weigh themselves daily, at the same time, with the same amount of clothing, and on the same scale. Weights should be recorded to allow for day-to-day comparisons to help identify early signs of fluid retention. Incorrect Answers: [A. "I don't plan on eating any more frozen meals."] Frozen meals are often high in sodium. Most heart failure clients are instructed to limit sodium intake. All foods high in sodium (>400 mg/serving) should be avoided. [B. "I plan to take my diuretic pill in the morning."] Diuretic medications cause clients to urinate more. Morning is the appropriate time to take this type of medication. Evening administration would cause nocturia and interrupted sleep. [D. "I'm going to look into joining a cardiac rehabilitation program."] Exercise training, such as cardiac rehabilitation, improves symptoms of chronic heart failure. It has been found to be safe and improves the client's overall sense of well-being. It has also been correlated with reduction in mortality. Educational objective:Discharge education for the client with chronic heart failure should include daily weights, drug regimens, diet, and exercise plans. The use of any NSAIDS is contraindicated in heart failure as these contribute to sodium retention, and therefore fluid retention.

The practical nurse is assisting the registered nurse with completing a health history of a client with suspected rheumatic fever. Which question is most important to ask the client in order to establish a diagnosis? A. "Do you typically take all of your antibiotics when they are prescribed?" B. "Has anyone in your family had rheumatic fever?" C. "Have you recently had a streptococcal throat infection?" D. "What has your temperature been over the past several days?"

Correct Answer: C. "Have you recently had a streptococcal throat infection?" Rheumatic fever (RF) is an acute inflammatory disease of the heart that can occur as a complication 2-3 weeks after streptococcal pharyngitis. RF is caused by a delayed-onset autoimmune reaction involving anti-streptococcal antibodies that cross-react with antigens in the heart and other organs. Recurrent, untreated streptococcal pharyngitis will lead to faster onset and increased severity of rheumatic heart disease due to increased autoimmune activity. RF affects the heart, skin, joints, and central nervous system. Evidence of a preceding streptococcal infection with the presence of either 2 major criteria or 1 major and 2 minor criteria indicate a high probability of RF. Incorrect Answers: [A. "Do you typically take all of your antibiotics when they are prescribed?"] Failing to complete a course of antibiotics is the cause of resistant infection strains and can result in recurring illness. It could contribute to the risk for RF but is not part of the criteria for diagnosing it. [B. "Has anyone in your family had rheumatic fever?"] Family history is not a risk factor for RF. [D. "What has your temperature been over the past several days?"] Fever is a symptom of many illnesses, not just RF. Educational objective:Streptococcal pharyngitis can precede the development of rheumatic fever (RF) by 2-3 weeks. The nurse should ask about streptococcal throat infection when collecting health history information in a client suspected of having RF.

The nurse is reinforcing instructions to a client scheduled for cardiac pharmacologic nuclear stress testing. Which client statements indicate appropriate understanding? A. "I can smoke 1 cigarette the day of the test so that I won't have withdrawal." B. "I should eat a hearty breakfast the morning of the test to avoid nausea." C. "I should stop drinking coffee 24 hours before the procedure." D. "I should take my usual dose of insulin the day of the test."

Correct Answer: C. "I should stop drinking coffee 24 hours before the procedure." A pharmacologic nuclear stress test uses vasodilators (eg, adenosine, dipyridamole) to simulate exercise when clients are unable to tolerate continuous physical activity or when their target heart rate is not achieved through exercise alone. These drugs produce vasodilation of the coronary arteries in clients with suspected coronary heart disease. A radioactive dye is injected to allow a special camera to produce images of the heart. Based on these images, the health care provider (HCP) can determine whether there is adequate coronary perfusion. Preprocedure client instructions include: — Do not eat, drink, or smoke on the day of the test (NPO for at least 4 hours) — Avoid caffeine (including decaffeinated products) 24 hours before the test — If insulin/pills are prescribed for clients with diabetes, consult the HCP about appropriate dosage on the day of the test. Hypoglycemia can result if the medicine is taken without food — Some medications can interfere with test results by masking angina. Do not take certain cardiac medications (eg, nitrates, dipyridamole, beta blockers) unless the HCP directs otherwise or unless needed to treat chest discomfort on the day of the test. Educational objective:Clients scheduled for cardiac nuclear pharmacologic stress testing should not eat, drink, or smoke on the day of the test; they should avoid caffeinated products for 24 hours before the test and avoid taking certain cardiac medications (eg, nitrates, beta blockers) unless otherwise instructed by the health care provider.

A client with a history of headaches is scheduled for a lumbar puncture to assess the cerebrospinal fluid pressure. The nurse is preparing the client for the procedure. Which statement by the client indicates a need for further teaching by the nurse? A. "I may feel a sharp pain that shoots to my leg, but it should pass soon." B. "I will go to the bathroom and try to urinate before the procedure." C. "I will need to lie on my stomach during the procedure." D. "The physician will insert a needle between the bones in my lower spine."

Correct Answer: C. "I will need to lie on my stomach during the procedure." Cerebrospinal fluid (CSF) is assessed for color, contents, and pressure. Normal CSF is clear and colorless, and contains a little protein, a little glucose, minimal white blood cells, no red blood cells, and no microorganisms. Normal CSF pressure is 60-150 mm H2O. Abnormal CSF pressure or contents can help diagnose the cause of headaches in complicated cases. CSF is collected via lumbar puncture or ventriculostomy. Prior to a lumbar puncture, clients are instructed as follows: 1. Empty the bladder before the procedure 2. The procedure can be performed in the lateral recumbent position or sitting upright. These positions help widen the space between the vertebrae and allow easier insertion of the needle 3. A sterile needle will be inserted between the L3/4 or L4/5 interspace 4. Pain may be felt radiating down the leg, but it should be temporary After the procedure, instruct the client as follows: 1. Lie flat with no pillow for at least 4 hours to reduce the chance of spinal fluid leak and resultant headache 2. Increase fluid intake for at least 24 hours to prevent dehydration Educational objective:Lumbar puncture can be performed with clients in the sitting position or positioned on the left side with the knees drawn up (fetal position).

The nurse is caring for several clients in a women's health clinic. Based on the data collected, which client's history is most concerning for an increased risk of endometrial cancer? A. 40-year-old client who has been taking hormonal birth control pills for the past 10 years B. 45-year-old client who reports a history of an ectopic pregnancy with a ruptured ovary and two preterm births C. 47-year-old client with polycystic ovary syndrome, obesity, and a history of unsuccessful infertility treatments D. 60-year-old client who recently had a colposcopy after testing positive for a high-risk type of human papillomavirus

Correct Answer: C. 47-year-old client with polycystic ovary syndrome, obesity, and a history of unsuccessful infertility treatments Endometrial cancer arises from the inner lining of the uterus and forms after the development of unregulated endometrial overgrowth (ie, hyperplasia). Although typically slow growing, it can metastasize to the myometrium (ie, uterine muscle tissue), cervix, and nearby lymph nodes and eventually beyond the pelvis. Many signs of endometrial cancer are nonspecific (eg, lower back or abdominal pain), but the hallmark symptom is abnormal uterine bleeding (eg, heavy, prolonged, intermenstrual, and/or postmenopausal bleeding). As with many cancers, the client's family and genetic history (eg, BRCA mutation carrier) are significant risk factors; however, prolonged estrogen exposure without adequate progesterone is the greatest risk factor for developing endometrial cancer. Factors increasing estrogen exposure and endometrial cancer risk include: — Conditions associated with infrequent or anovulatory menstrual cycles (eg, polycystic ovary syndrome, infertility, late menopause, early menarche) — Obesity — Tamoxifen (a medication given for breast cancer) Incorrect Answers: [A. 40-year-old client who has been taking hormonal birth control pills for the past 10 years ] Progestin-containing contraceptives (ie, birth control pills) are associated with a decreased endometrial cancer risk because progestins thin the uterine lining, therefore preventing endometrial hyperplasia. [B. 45-year-old client who reports a history of an ectopic pregnancy with a ruptured ovary and two preterm births] Ectopic pregnancy with a ruptured ovary or preterm birth is not associated with endometrial cancer, although never giving birth at term gestation may increase ovarian cancer risk. [D. 60-year-old client who recently had a colposcopy after testing positive for a high-risk type of human papillomavirus] Infection with a high-risk type of human papillomavirus increases cervical (not endometrial) cancer risk. Educational objective:Endometrial cancer is a slow-growing malignancy that arises from the inner lining of the uterus. Major risk factors include conditions associated with infrequent or anovulatory menstrual cycles (eg, polycystic ovary syndrome, infertility), obesity, and tamoxifen therapy.

The nurse is reinforcing the proper use of a peak flow meter with a client recently diagnosed with asthma. Place the steps for measuring peak expiratory flow using a peak flow meter in the correct order. All options must be used. 1. Instruct the client to breathe in deeply, place the mouthpiece in the mouth, and close the lips tightly around it to form a seal. 2. Instruct the client to exhale as quickly and forcibly as possible and note the reading on the numbered scale. 3. Record the highest reading achieved 4.Reeat the procedure 2 more times with a 5-10-second rest period between exhalation. 5. Slide the indicator on the numbered scale on the flow meter to the 0 or lowest number and instruct the client to stand or sit as upright as possible A. 3, 5, 1, 2, 4 B. 5, 2, 1, 4, 3 C. 5, 1, 2, 4, 3 D. 1, 2, 5, 4, 3

Correct Answer: C. 5, 1, 2, 4, 3 A peak flow meter is a portable hand-held device used to measure the client's ability to push air out of the lungs. Flow meters are available in low ranges for measuring smaller volumes and for small children, and in a standard range for older children, teenagers, and adults. To obtain the most accurate readings to help guide, maintain, and evaluate treatment in clients with moderate to severe asthma, the procedure is carried out in the following order: 1. To obtain the most accurate readings to help guide, maintain, and evaluate treatment in clients with moderate to severe asthma, the procedure is carried out in the following order: 2. Instruct the client to breathe in deeply, place the mouthpiece in the mouth, and the close lips tightly around it to form a seal 3. Instruct the client to exhale as quickly and forcibly as possible and note the reading on the numbered scale 4. Repeat the procedure 2 more times with a 5-10-second rest period between exhalations 5. Record the highest of the 3 readings achieved. This reading is compared to the client's established personal best to determine current level of control. Educational objective:A peak flow meter is a hand-held device used to measure the client's ability to push air out of the lungs. It is used for assessment of moderate to severe asthma.

A client underwent a transurethral resection of the prostate (TURP) today and has a 3-way Foley urinary catheter with continuous bladder irrigation (CBI). The client reports lower abdominal pain rated as an 8 on a scale of 0-10. What action should the nurse carry out first? A. Administer prescribed belladonna-opium suppositories prn B. Administer prescribed morphine intravenous push prn C. Check amount and characteristics of urine output D. Check when the client had the last flatus or bowel movement

Correct Answer: C. Check amount and characteristics of urine output Blood and mucus can obstruct the Foley catheter if the CBI is not infused at a sufficient rate. Bladder pain will result from distention if the flow is obstructed. The nurse should ensure that there is adequate urinary drainage and no blockage from blood clots before treating the pain. If the urinary flow is obstructed, manual irrigation with sterile normal saline should be performed until there are no clots or the urine is clear/pink. Incorrect Answers [A. Administer prescribed belladonna-opium suppositories prn] Belladonna-opium suppositories or antispasmodics (eg, oxybutynin) are used for bladder spasms, an expected complication of the TURP procedure. Clients should be instructed not to urinate around the catheter as this would increase bladder pressure and spasms. Narcotics can be used for postoperative pain. If the urinary flow is adequate, a description of the pain would help to determine whether to give the client a narcotic or an antispasmodic. Before treating the resulting pain, the possibility of a physiological etiology for procedure-related pain (eg, blockage of urinary flow from blood clots) should be ruled out first. [B. Administer prescribed morphine intravenous push prn] Belladonna-opium suppositories or antispasmodics (eg, oxybutynin) are used for bladder spasms, an expected complication of the TURP procedure. Clients should be instructed not to urinate around the catheter as this would increase bladder pressure and spasms. Narcotics can be used for postoperative pain. If the urinary flow is adequate, a description of the pain would help to determine whether to give the client a narcotic or an antispasmodic. Before treating the resulting pain, the possibility of a physiological etiology for procedure-related pain (eg, blockage of urinary flow from blood clots) should be ruled out first. [D. Check when the client had the last flatus or bowel movement] Large intestine peristalsis does not usually return for at least 24 hours. Intestinal pain is usually related to the presence of flatus. It is too soon for this to be the primary cause. An etiology related to the procedure should be ruled out first. Educational objective:Causes of postoperative pain from TURP with a CBI include a kinked blocked catheter, bladder spasms, and general postoperative pain. The nurse should ensure first that urinary flow is intact prior to treating the pain with analgesics.

The nurse responds to the call light of a client with chronic obstructive pulmonary disease (COPD) who says, "I can't breathe." The client seems to be having difficulty breathing and is nervous and tremulous. Vital signs are stable, oxygen saturation is 92% on 2 L, and there are clear breath sounds bilaterally. Which intervention would be most appropriate at this time? A. Administer albuterol nebulizer B. Assist the client in identifying the trigger and ways to avoid it C. Coach the client through controlled breathing exercises D. Continue to monitor oxygen saturation

Correct Answer: C. Coach the client through controlled breathing exercises Anxiety is an emotional reaction to a perceived threat. For the client with COPD, the fear of having difficulty breathing can actually trigger difficulty breathing, which worsens as the client's anxiety increases. This client is stable, with no obvious cause of shortness of breath. The nurse should intervene by calmly coaching the client through breathing exercises, which will promote relaxation and help alleviate the anxiety that is causing the client to feel short of breath. Incorrect Answers: [A. Administer albuterol nebulizer] The client's lung sounds are clear bilaterally and so albuterol, a bronchodilator used for wheezing, will not be helpful. Its action as an adrenergic agonist may cause tachycardia and tremulousness and actually worsen the client's anxiety. [B. Assist the client in identifying the trigger and ways to avoid it] Trigger avoidance and problem solving are appropriate strategies for long-term control of anxiety and shortness of breath. However, these are not appropriate at this time as the client has acute symptoms that need to be controlled. [D. Continue to monitor oxygen saturation] This client has normal oxygen saturation. Constant monitoring is not likely to alleviate the symptoms unless the client is reassured by this knowledge. However, the client's anxiety may actually be worsened by worrying about the saturation results and the alarms that are likely to be triggered by monitoring. Educational objective:Anxiety is common in clients with COPD and can contribute to difficulty breathing. In the client with acute shortness of breath and normal assessment findings, appropriate interventions are controlled breathing and relaxation.

Which group of food selections would be the best choice for a client advancing to a full liquid diet 3 days after bariatric surgery? A. Apple juice, mashed potatoes, chocolate pudding B. Chicken broth, low-fat cheese omelet, strawberry ice cream C. Creamy wheat cereal, blended cream of chicken soup, protein shake D. Low-fat vanilla yogurt, smooth peanut butter, vegetable juice

Correct Answer: C. Creamy wheat cereal, blended cream of chicken soup, protein shake Bariatric surgery (eg, gastric banding, sleeve gastrectomy) reduces stomach capacity. A client's bariatric postoperative diet is restricted to foods that are low in simple carbohydrates and high in nutrients (eg, protein, fiber). After gastric surgery, consumption of simple carbohydrates can lead to dumping syndrome (ie, cramping, diarrhea). The client will tolerate only small meals of clear liquids at first, advance to full liquids 24-48 hours after surgery, and then progress gradually to solid foods as the gastrointestinal tract heals. Small, frequent meals are recommended to avoid overstretching of the pouch and to prevent nausea, vomiting, and regurgitation. The best food choices for a bariatric full liquid diet are cream soups, refined cooked cereals, sugar-free drinks, and low-sugar protein shakes and dairy foods. Incorrect Answers: [A. Apple juice, mashed potatoes, chocolate pudding] Fruit juices and puddings are high in sugar and not acceptable for a bariatric full liquid diet. Mashed potatoes are considered appropriate for a soft diet. [B. Chicken broth, low-fat cheese omelet, strawberry ice cream] Ice cream is high in sugar and not acceptable for a bariatric full liquid diet. Eggs are appropriate for a soft diet. [D. Low-fat vanilla yogurt, smooth peanut butter, vegetable juice] Yogurt is high in sugar and not appropriate for a bariatric full liquid diet. Peanut butter and vegetable juice are appropriate for a soft diet. Educational objective:Clients recovering from bariatric surgery are given small, frequent meals to prevent nausea, vomiting, and regurgitation related to overstretching of the stomach. The bariatric postoperative diet is restricted to foods that are high in nutrients (eg, protein, fiber) and low in simple carbohydrates to prevent dumping syndrome.

A client admitted with acute myocardial infarction suddenly displays air hunger, dyspnea, and coughing with frothy, pink-tinged sputum. What would the nurse anticipate when auscultating the breath sounds of this client? A. Bronchial breath sounds at lung periphery B. Clear vesicular breath sounds at lung bases C. Diffuse bilateral crackles at lung bases D. Stridor in upper airways

Correct Answer: C. Diffuse bilateral crackles at lung bases Acute-onset dyspnea and cough productive of pink, frothy sputum indicate severe pulmonary edema, likely a complication from myocardial infarction. Pink sputum results from ruptured bronchial veins due to high back pressure. The mix of blood and airway fluids creates the pink tinge. On assessment, crackles can be heard at the lung bases. Incorrect Answers: [A. Bronchial breath sounds at lung periphery] Bronchial breath sounds are normally heard over the trachea. These are harsh and high-pitched; inspiration and expiration are of similar duration. The presence of these on lung periphery indicates pneumonia (consolidation). [B. Clear vesicular breath sounds at lung bases] Clear vesicular breath sounds (normal breath sounds) are not expected in pulmonary edema. [D. Stridor in upper airways] Stridor is consistent with a laryngospasm or edema of the upper airway. Educational objective:Acute-onset dyspnea and cough with frothy, pink-tinged sputum indicate pulmonary edema. Auscultation reveals crackles at the lung bases.

The nurse is caring for a hospice client with advanced heart failure who is having trouble breathing. Which comfort intervention should the nurse implement first? A. Administer PRN albuterol by nebulizer B. Assist with guided imagery to relieve anxiety C. Elevate the head of the bed D. Give PRN sublingual morphine

Correct Answer: C. Elevate the head of the bed Dyspnea (eg, difficulty breathing) is a common symptom in the hospice client with advanced heart failure. Excess fluid volume and decreased cardiac output produce pulmonary edema, impairing gas exchange and causing dyspnea. Elevating the head of the bed reduces abdominal pressure on the diaphragm, making it easier for the client to breathe. It is an effective intervention that can be implemented first, quickly, and easily. Incorrect Answers: [A. Administer PRN albuterol by nebulizer] Albuterol, a bronchodilator, is unlikely to relieve dyspnea caused by the fluid overload present in heart failure. [B. Assist with guided imagery to relieve anxiety] Relaxation techniques (eg, guided imagery) may help relieve the anxiety associated with dyspnea, but the nurse should first raise the head of the bed to facilitate adequate breathing. [D. Give PRN sublingual morphine] Morphine used with diuretics can alleviate dyspnea associated with fluid overload in heart failure, but this would not be the first intervention. Educational objective:The client with advanced heart failure on hospice is likely to have dyspnea associated with fluid overload. The first intervention should be to elevate the head of the bed to reduce abdominal pressure on the diaphragm. Morphine with diuretics can help alleviate persistent dyspnea.

The nurse monitoring a client with appendicitis will expect the client to give which description of the associated abdominal pain? A. A burning sensation; in the upper abdomen B. An 8 out of 10; on the left side below the belly button C. Excruciating; in the lower abdomen above the right hip D. Intermittent; in the abdomen and right shoulder

Correct Answer: C. Excruciating; in the lower abdomen above the right hip The appendix is a blind pouch at the junction of the ileum of the small intestine and the large intestine (cecum). When infected or obstructed (foreign body, fecal material, tumor, lymph tissue), the appendix becomes inflamed, causing acute appendicitis. Signs and symptoms of acute appendicitis include the following: · Pain: Continuous; begins in the periumbilical region and moves to the right lower quadrant, centering at McBurney's point (one-third of the distance from the right anterior superior iliac spine to the umbilicus) · Gastrointestinal symptoms: Anorexia, nausea, and vomiting · Rebound tenderness and guarding Clients with acute appendicitis attempt to decrease pain by avoiding increased intraabdominal pressure (eg, from coughing, sneezing, deep inhalation) and lying still with the right leg flexed. Incorrect Answers [A. A burning sensation; in the upper abdomen] Burning pain in the upper abdomen can be due to gastric or duodenal ulcers. If the ulcers are located posteriorly, the client may experience back pain. [B. An 8 out of 10; on the left side below the belly button] Pain in the left lower quadrant is associated with diverticulitis (often in the sigmoid colon). Other signs and symptoms include a palpable, tender abdominal mass and systemic symptoms of infection (fever, increased C-reactive protein, leukocytosis with a left shift). [D. Intermittent; in the abdomen and right shoulder] Pain and tenderness in the epigastric or right upper quadrant that are referred to the right scapula are associated with acute cholecystitis. Clients may also experience indigestion, nausea, vomiting, restlessness, and diaphoresis. Educational objective:Pain associated with acute appendicitis typically begins in the periumbilical region and migrates to the area overlying the appendix (McBurney's point). The client will attempt to decrease pain by lying still with the right leg flexed and avoiding increased intraabdominal pressure (eg, from coughing, sneezing, deep inhalation).

A client with heart failure has gained 5 lb (2.26 kg) over the last 3 days. The nurse reviews the client's blood laboratory results. Based on this information, what medication administration does the nurse anticipate? Laboratory Results Sodium 126 mEq/L Potassium 4.8 mEq/L Calcium 9.0 mg/dL A. 0.45% sodium chloride IV B. Calcium gluconate C. Furosemide D. Sodium polystyrene sulfonate

Correct Answer: C. Furosemide In heart failure, cardiac output is reduced because the heart is unable to pump blood adequately. This reduction in cardiac output reduces perfusion to the vital organs, including the kidneys. Decreased renal blood flow triggers the kidneys to activate the renin-angiotensin system as a compensatory mechanism, which increases blood volume by increasing water resorption in the kidneys. This compensatory mechanism results in fluid volume excess and dilutional hyponatremia (more free water than sodium). Dilutional hyponatremia can be treated with fluid restriction, loop diuretics, and ACE inhibitors (eg, lisinopril, captopril). Furosemide works to resolve hyponatremia by promoting free water excretion, allowing for hemoconcentration and increased sodium levels. Incorrect Answers: [A. 0.45% sodium chloride IV] 0.45% sodium chloride is a hypotonic solution. Giving hypotonic saline would provide more free water than sodium, thereby worsening fluid overload and hyponatremia. [B. Calcium gluconate] The client's calcium is within normal limits and does not need replacement. [D. Sodium polystyrene sulfonate] Sodium polystyrene sulfonate (Kayexalate, Kionex) is a medication used to treat hyperkalemia that works by exchanging sodium for potassium across the mucous membranes of the bowel and then excreting potassium via stool. Sodium polystyrene sulfonate is not indicated if potassium is within normal limits. Educational objective:Heart failure is characterized by reduced cardiac output, which can reduce renal blood flow. Reduced renal blood flow activates the renin-angiotensin system, resulting in fluid volume excess and dilutional hyponatremia. Loop diuretics (eg, furosemide) promote free water excretion, allowing for hemoconcentration and increased sodium levels.

The nurse is caring for a client with a history of type 2 diabetes mellitus who reports feeling lightheaded and shaky. The client's finger-stick blood glucose is 58 mg/dL. Which action should the nurse take next? A. Administer glucagon by subcutaneous injection B. Administer rapid-acting insulin per sliding scale as prescribed C. Give the client 4 oz of fruit juice or a regular soft drink D. Give the client a snack of cheese or peanut butter with crackers

Correct Answer: C. Give the client 4 oz of fruit juice or a regular soft drink This client is exhibiting signs and symptoms of a hypoglycemic reaction (eg, diaphoresis, shakiness, hunger). A client with blood glucose <70 mg/dL and who is alert enough to ingest food or fluids orally should be given 15 g of a simple carbohydrate (eg, 4 oz fruit juice or regular soft drink, 8 oz low-fat milk, 1 tablespoon of honey or jelly). After receiving a simple carbohydrate, the client's blood glucose should be rechecked 15-30 minutes later for resolution of hypoglycemia. If the client remains hypoglycemic, treatment should be repeated with continued blood glucose monitoring. Incorrect Answers: [A. Administer glucagon by subcutaneous injection] A glucagon injection can be administered if the client becomes unconscious or cannot follow commands, or if oral glucose replacement is unsuccessful. For a client who is alert and oriented, oral carbohydrates should be given first. [B. Administer rapid-acting insulin per sliding scale as prescribed] This client's blood glucose is 58 mg/dL. Administration of insulin will worsen hypoglycemia and could precipitate coma and seizure. [D. Give the client a snack of cheese or peanut butter with crackers] A complex snack with carbohydrates, fats, and proteins can help stabilize a client's blood sugar after initial treatment with simple carbohydrates. Fats and proteins are slow to digest and release sugars, and are therefore not recommended during the initial treatment of hypoglycemia. Educational objective:For alert clients with hypoglycemia (blood glucose <70 mg/dL), the nurse should provide 15 g of simple carbohydrates (eg, 4 oz fruit juice, 8 oz low-fat milk). Blood glucose is rechecked 15-30 minutes later. After resolution of hypoglycemia, a carbohydrate snack with fats and proteins is given to prevent relapse.

The nurse is reinforcing meal planning teaching to a group of clients with celiac disease. Which meal is appropriate for the nurse to include? A. Baked salmon with rice, steamed vegetables, and dinner roll B. Breaded pork chops, corn on the cob, and steamed snow peas C. Grilled chicken, green beans, and mashed potatoes D. Spaghetti with Italian tomato sauce and meatballs

Correct Answer: C. Grilled chicken, green beans, and mashed potatoes Celiac disease is an autoimmune disorder in which chronic inflammation caused by gluten damages the small intestine. The following are important dietary principles to teach clients with celiac disease: 1. All gluten-containing products should be eliminated from the diet. These include wheat, barley, rye, and oats. 2. Rice, corn, and potatoes are gluten-free and are allowed on the diet 3. Processed foods (eg, chocolate candy, hot dogs) may contain "hidden" sources of gluten such as modified food starch, malt, and soy sauce. Food labels should indicate that the product is gluten-free. 4. Clients will need to be on a gluten-free diet for the rest of their lives. Eliminating gluten from the diet reduces the risk for nutritional deficiencies and intestinal cancer (lymphoma). 5. Eating even small amounts of gluten will damage the intestinal villi, even though the client may have no clinical symptoms. Therefore, all sources of gluten must be eliminated from the diet. Incorrect Answers: [A. Baked salmon with rice, steamed vegetables, and dinner roll] Baked salmon, rice, and steamed vegetables are permitted on a gluten-free diet. The dinner roll contains gluten and should be avoided. Baked goods and breads (including white and wheat) contain gluten unless the package is labeled "gluten-free" or the products are made from nongluten sources (eg, rice flour). [B. Breaded pork chops, corn on the cob, and steamed snow peas] Although meat, fish, and poultry are permitted, marinated and breaded protein sources should be avoided. Corn and snow peas are appropriate selections for a gluten-free diet. [D. Spaghetti with Italian tomato sauce and meatballs] Pasta contains gluten and should be avoided. Gluten-free pastas are available and are safe to consume. Educational objective:All sources of gluten must be eliminated from the diet of a client with celiac disease. Consuming small amounts, even in the absence of clinical symptoms, will increase the risk for damage to the intestinal villi. Clients can have foods containing rice, corn, and potatoes. They should read food labels and follow the diet for the rest of their lives.

The nurse is caring for a client with Guillain-Barré syndrome after a recent gastrointestinal illness. Monitoring for which symptom is a nursing care priority in this client? A. Diaphoresis with facial flushing. B. Hypoactive or absent bowel sounds C. Inability to cough or lift the head D. Warm, tender, and swollen leg

Correct Answer: C. Inability to cough or lift the head Guillain-Barré syndrome (GBS) is an acute, immune-mediated polyneuropathy most often accompanied by ascending muscle weakness and absent deep-tendon reflexes. Many clients have a history of recent viral respiratory tract or gastrointestinal infection. Lower extremity weakness progresses over hours to days to involve the thorax, arms, and cranial nerves. However, neuromuscular respiratory failure is the most life-threatening complication. Early signs indicating impending respiratory failure include the following: · Inability to cough · Shallow respirations · Dyspnea and hypoxia · Inability to lift the head or eyebrows Assessing the client's pulmonary function by serial spirometry is also recommended. Measurement of forced vital capacity (FVC) is the gold standard for assessing ventilation; a decline in FVC indicates impending respiratory arrest, requiring endotracheal intubation. Incorrect Answers: [A. Diaphoresis with facial flushing.] Severe autonomic dysfunction can present as diaphoresis and facial flushing. [B. Hypoactive or absent bowel sounds] The client with GBS is also at risk for paralytic ileus, which is related to either immobility or nerve damage. As a result, the nurse should monitor for hypoactive/absent bowel sounds. [D. Warm, tender, and swollen leg] Clients with GBS are at risk for deep venous thrombosis due to lack of ambulation and should receive pharmacologic prophylaxis (eg, heparin) and support stockings. Although the symptoms in options 1, 2, and 4 represent a progressive illness and are important to communicate to the registered nurse promptly, they are not the highest priority compared to impending respiratory failure. Educational objective:Respiratory distress is a potential complication of progressing paralysis in clients with Guillain-Barré syndrome. The nurse should prioritize and monitor for the presence of this complication. Measurement of serial spirometry (forced vital capacity) is the gold standard for assessing ventilation.

A client's arterial blood gases (ABGs) are shown in the exhibit. The nurse would expect which finding to demonstrate that the client is compensating for the ABGs? Click on the exhibit button for additional information. A. Decrease in bicarbonate reabsorption B. Decrease in respiratory rate C. Increase in bicarbonate reabsorption D. Increase in respiratory rate

Correct Answer: C. Increase in bicarbonate reabsorption The client's ABGs have low pH consistent with acidosis. If it is a primary respiratory acidosis, pCO2 would be higher. If it is metabolic acidosis, bicarbonate would be lower. Because this client has low pH coupled with low bicarbonate, the most likely diagnosis is primary metabolic acidosis. Respiratory alkalosis is the body's natural compensation for metabolic acidosis. Respiratory alkalosis is achieved by blowing more CO2 off from the system through rapid breathing. Incorrect Answers: [A. Decrease in bicarbonate reabsorption] Decreased bicarbonate reabsorption would produce metabolic acidosis; this would occur as a compensation for primary respiratory alkalosis (decreased pCO2 and high pH). [B. Decrease in respiratory rate] When the respiratory rate is decreased, pCO2 would increase, creating a respiratory acidosis; this would occur in response to a primary metabolic alkalosis. [C. Increase in bicarbonate reabsorption] Increased bicarbonate reabsorption would produce metabolic alkalosis; this would occur as a compensation for primary respiratory acidosis (increased pCO2 and low pH). Educational objective:Respiratory alkalosis is the body's natural compensation for metabolic acidosis. It is achieved by blowing more CO2 off from the system through rapid breathing.

A client comes to the emergency department reporting alkaline drain cleaner splashed into the eye. The conjunctiva of the affected eye is erythematous, and the client reports a burning sensation. What action is appropriate at this time? A. Administer PO analgesic medication B. Cover the affected eye with an eye patch C. Initiate continuous eye irrigation D. Perform a Snellen vision test

Correct Answer: C. Initiate continuous eye irrigation Ocular chemical burns require emergency care to prevent permanent vision loss. Alkali burns (eg, ammonia, cement, lye-containing drain cleanser) are particularly dangerous as they will quickly penetrate deep into the eye, causing severe, irreversible damage. For all types of ocular chemical burns, copious eye irrigation with sterile saline or water should begin immediately to flush the chemical irritant out of the eye. Before transport to an emergency care facility, tap water can be used for eye irrigation. If transported by ambulance, emergency care personnel continue irrigation during transport with IV tubing or a Morgan lens. Irrigation is continued until the pH of the eye returns to normal (pH 6.5-7.5), which typically requires 30-60 minutes depending on the type of chemical. Incorrect Answers: [A. Administer PO analgesic medication] Depending on the severity of the burn, anesthetic eye drops may be instilled prior to irrigation because ocular burns are very painful, but systemic analgesia is not a priority. [B. Cover the affected eye with an eye patch] Care of ocular burns may include covering the eye with an eye patch and use of eye drops to prevent eye muscle spasms; however, eye irrigation should be performed first. [D. Perform a Snellen vision test] The Snellen eye chart is commonly used to assess visual acuity. However, eye irrigation is essential and should not be delayed. Educational objective:Ocular chemical burns require immediate, copious irrigation with sterile saline or water to flush the chemical irritant out of the eye. Irrigation is continued until the pH of the eye returns to normal, which typically takes 30-60 minutes.

A client with a permanent pacemaker with continuous telemetry calls the nurse and reports feeling lightheaded and dizzy. The client's blood pressure is 75/55 mm Hg. What is the nurse's priority action? A. Administer atropine 0.5 mg IV B. Administer dopamine 5 mcg/kg/min IV C. Initiate transcutaneous pacing D. Notify the health care provider

Correct Answer: C. Initiate transcutaneous pacing The client is experiencing failure to capture from the permanent pacemaker with subsequent bradycardia and hypotension. Failure to capture appears on the cardiac monitor as pacemaker spikes that are not followed by QRS complexes. Pacemaker malfunction may be caused by a failing battery, malpositioned lead wires, or fibrosis at the tip of lead wire(s) preventing adequate voltage for depolarization. This client is symptomatic (eg, hypotension, dizziness) from insufficient perfusion. The nurse's priority is to use transcutaneous pacemaker pads to normalize the heart rate, stabilize blood pressure, and adequately perfuse organs until the permanent pacemaker is repaired or replaced. Administer analgesia and/or sedation as prescribed as transcutaneous pacing is very uncomfortable for the client. Incorrect Answers: [A. Administer atropine 0.5 mg IV] Atropine is administered to clients with symptomatic bradycardia; however, this client's symptoms are caused by failure to capture. Therefore, obtaining capture via transcutaneous pacing should resolve the client's symptoms. [B. Administer dopamine 5 mcg/kg/min IV] Dopamine is an inotrope used to treat hypotension due to bradycardia. This client is bradycardic and hypotensive due to failure to capture. If hypotension persists after transcutaneous pacing is initiated, an inotrope may be necessary. [D. Notify the health care provider] The health care provider needs to be notified, but the nurse should first use the transcutaneous pacemaker to stabilize the client. Educational objective:Signs and symptoms of a failing pacemaker include failure to capture (pacer spikes without associated QRS complexes) with bradycardia and hypotension. The nurse should use a transcutaneous pacemaker to stabilize the client until the internal pacemaker can be repaired or replaced.

After assisting a client with a lower gastrointestinal bleed back to bed, the nurse finds approximately 600 mL of frank red blood in the toilet. The client is pale and diaphoretic and reports dizziness. Which action should the nurse perform first? A. Document the output and vital signs B. Draw blood for hemoglobin and hematocrit C. Lower the head of the bed D. Notify the registered nurse

Correct Answer: C. Lower the head of the bed Acute blood loss is a medical emergency, and the nurse must intervene quickly. Lowering the head of the bed or placing the client in the supine position maintains blood perfusion to the brain and other vital organs. This can be done quickly to help stabilize the client before other interventions are performed. Incorrect Answers: [A. Document the output and vital signs] Documenting output and vital signs is an important action but should be done after lowering the head of the bed to improve perfusion. [B. Draw blood for hemoglobin and hematocrit] Monitoring hemoglobin and hematocrit levels is appropriate for assessing the severity of blood loss and need for possible blood transfusion. Blood loss typically takes a few hours to reflect on the client's laboratory report, and therefore, this is not the priority. [D. Notify the registered nurse] The registered nurse should be notified of the findings, which can be done after first improving perfusion by lowering the head of the bed. Educational objective:A client with significant blood loss has a medical emergency, and interventions that improve perfusion to the brain and vital organs (eg, lowering the head of the bed) should take priority.

A client is admitted to the hospital for severe headaches. The client has a history of increased intracranial pressure, which was relieved by draining cerebrospinal fluid via lumbar puncture. The client suddenly vomits and states, "That's weird; I didn't even feel nauseated." Which action by the nurse is most appropriate? A. Document amount of emesis B. Lower head of bed C. Notify supervising registered nurse D. Offer antinausea medication

Correct Answer: C. Notify supervising registered nurse Unexpected and projectile vomiting without nausea can be a sign of increased intracranial pressure (ICP), especially in the client with a history of increased ICP. The unexpected vomiting is related to pressure changes in the cranium. It can be associated with headache and gets worse with a lowered head position. The most appropriate action is to immediately obtain a full set of vital signs and report the findings to the supervising registered nurse. Incorrect Answers: [A. Document amount of emesis] Documentation is important, but it is not the priority action. [B. Lower head of bed] The head of the bed should be raised, not lowered, for clients with suspected ICP. Raising the head of the bed to 30 degrees helps to drain the cerebrospinal fluid via the valve system without lowering the cerebral blood pressure. [D. Offer antinausea medication] The vomiting is caused by pressure changes in the cranium, not by nausea. Antinausea medications are often not effective. Decreasing ICP will help the vomiting. Educational objective:Notify the supervising registered nurse of signs/symptoms of increased intracranial pressure, including unexpected vomiting. The vomiting is often projectile, is associated with headache, and gets worse with a lowered head position.

The nurse responds to a call for help from another staff member. Upon entering the client's room, the nurse observes an unlicensed assistive personnel (UAP) performing chest compressions on an unconscious adult client while another nurse is calling for the emergency response team. What action by the arriving nurse is the priority? A. Ask the UAP to stop compressions and check for a pulse B. Establish additional IV access with large-bore IVs C. Obtain the defibrillator and apply the pads to the client's chest D. Prepare to administer 100% O2 with a bag valve mask

Correct Answer: C. Obtain the defibrillator and apply the pads to the client's chest For the client in cardiac arrest, cardiopulmonary resuscitation must be started immediately. Effective chest compressions are essential for maintaining perfusion to vital organs. Early defibrillation is key in resolving life-threatening ventricular fibrillation or ventricular tachycardia and should not be delayed. The arriving nurse should obtain the defibrillator and apply the pads to the client's chest. Incorrect Answers: [A. Ask the UAP to stop compressions and check for a pulse] Interruptions in compressions should be kept to a minimum. Pulse checks are performed every 2 minutes per basic life support (BLS) guidelines or if a rhythm change is noted. [B. Establish additional IV access with large-bore IVs] Additional large-bore IVs may be needed for emergency medication administration (eg, epinephrine, amiodarone) but can be completed when more help arrives to the client's room. [D. Prepare to administer 100% O2 with a bag valve mask] Bag valve mask breaths with 100% oxygen should be initiated after obtaining the defibrillator. Educational objective:For the client in cardiac arrest, cardiopulmonary resuscitation must be started immediately. Early defibrillation is key in resolving life-threatening ventricular fibrillation or ventricular tachycardia and should not be delayed. The arriving nurse should obtain the defibrillator and apply the pads to the client's chest.

A nurse is caring for a client 2 days after surgical creation of an arteriovenous fistula in the forearm. Which finding should the nurse report immediately to the health care provider? A. 2+ pitting edema of the extremity with the arteriovenous fistula. B. Loud swooshing sound auscultated over the arteriovenous fistula. C. Pale skin of the hand of the arm with the arteriovenous fistula. D. Surgical site pain reported by the client as 3 on a scale of 0-10 hand exercises.

Correct Answer: C. Pale skin of the hand of the arm with the arteriovenous fistula. Arteriovenous fistula (AVF) is a permanent hemodialysis access surgically created by connecting an artery to a vein, typically in the forearm or upper arm. This anastomosis diverts arterial blood into the vein, which increases intravenous blood flow and causes the vein to thicken and expand (ie, "mature"). The matured AVF can then sustain frequent access by large-bore needles during hemodialysis. Arterial steal syndrome is an AVF complication that occurs when the anastomosed vein "steals" too much arterial blood, causing distal extremity ischemia. Symptoms occur distal to the AVF, including skin pallor, pain, numbness, tingling, diminished pulses, and poor capillary refill. Without prompt intervention, ischemia may lead to limb necrosis. Incorrect Answers: [A. 2+ pitting edema of the extremity with the arteriovenous fistula.] After AVF creation, edema may occur due to venous congestion but typically improves spontaneously. Extremity elevation helps reduce edema. Severe or prolonged edema (eg, >2 weeks) could indicate venous hypertension that may require surgery to prevent AVF failure. [B. Loud swooshing sound auscultated over the arteriovenous fistula.] A loud swooshing sound (ie, bruit) auscultated over the AVF is expected due to turbulent blood flow at the arteriovenous anastomosis. [D. Surgical site pain reported by the client as 3 on a scale of 0-10 hand exercises.] Hand-grip exercises (eg, ball squeezing, hand flexing) are encouraged after AVF creation to promote fistula maturation. Postoperative surgical site pain is expected; however, pain distal to the AVF may indicate tissue ischemia. Educational objective:Arterial steal syndrome is a complication of arteriovenous fistula (AVF) creation that impairs distal extremity perfusion and may result in tissue ischemia and necrosis. Symptoms include skin pallor, pain, numbness, tingling, diminished pulses, and poor capillary refill distal to the AVF.

During a camping trip, a camp counselor falls and gets a small splinter of wood embedded in the right eye. What action should the volunteer camp nurse take first? A. Gently flush the eye with cool water B. Instill optic antibiotic ointment C. Patch both eyes with eye shields D. Remove the splinter using tweezers

Correct Answer: C. Patch both eyes with eye shields The camp nurse protects the injured eye using an eye shield (eg, small Styrofoam or paper cup), ensuring the shield does not touch the foreign body. The eyes work in synchrony with each other; therefore, the non-injured eye is patched to prevent further eye movement. The nurse also facilitates transport to the nearest emergency care center for assessment and treatment by an ophthalmologist. Incorrect Answers: [A. Gently flush the eye with cool water] Flushing the eye with cool water is contraindicated as it may cause further damage by moving the splinter and/or introducing potential wound pathogens. [B. Instill optic antibiotic ointment] Instilling optic antibiotic ointment would interfere with ophthalmologic medical examination. Optic antibiotic ointment may be prescribed by the health care provider to reduce the risk of infection once the object is removed from the eye. [D. Remove the splinter using tweezers] The nurse should not attempt to remove a foreign body embedded in the eye. An ophthalmologist, a health care provider who specializes in the surgical and nonsurgical evaluation and treatment of eye conditions, should remove the embedded object as soon as possible. Educational objective:When a foreign body becomes accidentally embedded in the eye, both eyes should be shielded to prevent eye movement and additional injury. The nurse should immediately refer the client to an ophthalmologist for further evaluation and treatment.

When unlicensed assistive personnel (UAP) assist a client with a chest tube back to bed from the bedside commode, the plastic chest drainage unit accidentally falls over and cracks. The UAP immediately report this incident to the nurse. What is the nurse's immediate action? A. Clamp the tube close to the client's chest until a new chest drainage unit is set up B. Notify the health care provider C. Place the distal end of the chest tube into a bottle of sterile saline D. Position the client on the left side

Correct Answer: C. Place the distal end of the chest tube into a bottle of sterile saline If the chest tube is disconnected or if the chest drainage unit breaks, cracks, or malfunctions, submerge the distal end of the chest tube 1-2 in (2-4 cm) below the surface of a 250 mL bottle of sterile water or saline. This creates an immediate water seal and prevents air from entering the pleural space as the new chest drainage system is established. To prepare for this contingency, emergency equipment, including 2 chest tube clamps, a 250-mL bottle of sterile water or saline solution, and antiseptic wipes, should be kept at the bedside. To prevent accidental disconnection of the chest tube from the tubing, secure all connections with tape or bands according to hospital policy and procedure. Incorrect Answers: [A. Clamp the tube close to the client's chest until a new chest drainage unit is set up] Unless prescribed by the health care provider (HCP), chest tube clamping time should not exceed 1 minute as it raises intrapleural pressure, which can lead to a tension pneumothorax. Chest tubes are not routinely clamped. Clamping briefly is acceptable when checking for an air leak in the system or when changing the disposable collection unit. [B. Notify the health care provider] It is not necessary to notify the HCP when replacing a chest drainage system unless the client develops respiratory distress. [D. Position the client on the left side] Positioning the client on the left side is appropriate if a central line is inadvertently pulled out as this will allow any air that may have been sucked in to rise to the right atrium. It is not an appropriate intervention for a chest tube disconnection or a crack or malfunction in a chest drainage unit. Educational objective:If a chest tube disconnects from the chest drainage system or if a chest drainage unit cracks or malfunctions, submerge the distal end of the chest tube 1-2 in (2-4 cm) below the surface of a 250 mL bottle of sterile water or saline solution.

The nurse is caring for a client with a history of tonic-clonic seizures. After a seizure lasting 25 seconds, the nurse notes that the client is confused for 20 minutes. The client does not know the current location, does not know the current season, and has a headache. The nurse documents the confusion and headache as which phase of the client's seizure activity? A. Aural phase B. Ictal phase C. Postictal phase D. Prodromal phase

Correct Answer: C. Postictal phase A seizure is an uncontrolled electrical discharge of neurons in the brain that interrupts normal function. Seizure manifestations generally are classified into 4 phases: 1. The prodromal phase is the period with warning signs that precede the seizure (before the aural phase). 2. The aural phase is the period before the seizure when the client may experience visual or other sensory changes. Not all clients experience or can recognize a prodromal or aural phase before the seizure. 3. The ictal phase is the period of active seizure activity. 4. During the postictal phase, the client may experience confusion while recovering from the seizure. The client may also experience a headache. Postictal confusion can help identify clients by differentiating seizures from syncope. In syncope, there will be only a brief loss of consciousness without prolonged post-event confusion. Educational objective:Clients may experience confusion after a seizure during the postictal phase. The client should be observed for safety and abnormalities documented before and during this phase.

The nurse is caring for a client after percutaneous placement of a coronary stent for a myocardial infarction. The client rates lower back pain as 5 on a scale of 0-10 and has blood pressure of 140/92 mm Hg. The cardiac monitor shows normal sinus rhythm with occasional premature ventricular contractions. Which prescription should the nurse administer first? Laboratory Results Potassium — 3.3 mEq/L (3.3 mmol/L) Sodium — 149 mEq/L (149 mmol/L) Glucose — 157 mg/dL (8.7 mmol/L) A. Captopril PO every 8 hours B. Morphine IV PRN for pain C. Potassium chloride IVPB once D. Regular insulin subcutaneous with meals

Correct Answer: C. Potassium chloride IVPB once Clients with myocardial infarction (MI) are at risk for life-threatening dysrhythmias (eg, heart block, ventricular tachycardia, ventricular fibrillation) both during the MI and following reperfusion therapy (eg, coronary artery stenting). Myocardial ischemia damages cardiac muscle cells, causing electrical irritability (eg, premature ventricular contractions) that can be exacerbated by electrolyte imbalances (eg, hypokalemia). Hypokalemia hyperpolarizes cardiac electrical conduction pathways, increasing the risk for dysrhythmias. Therefore, prompt potassium replacement is the priority in these clients. Incorrect Answers: [A. Captopril PO every 8 hours] ACE inhibitors (eg, captopril, enalapril, lisinopril) help reduce the risk of future MIs by reducing blood pressure and cardiac workload and inhibiting ventricular remodeling. ACE inhibitors should be administered after MI; however, life-threatening dysrhythmias pose a higher risk to the client. [B. Morphine IV PRN for pain] Administering morphine is an appropriate intervention to address the client's back pain, but it is not the priority. [D. Regular insulin subcutaneous with meals] Strict glycemic control in the resolution phase of an acute MI is associated with better long-term outcomes (eg, reduced morbidity/mortality), but it does not take priority. Educational objective:Prompt potassium replacement is the priority action for hypokalemic clients with myocardial infarction because they are at increased risk for life-threatening dysrhythmias (eg, heart block, ventricular tachycardia, ventricular fibrillation) and cardiac arrest.

The nurse is caring for a 40-year-old client with acromegaly in an outpatient health clinic. Which new finding is most important for the nurse to report? A. Fasting blood glucose of 126 mg/dL B. Knee pain when walking C. Presence of S3 and S4 heart sounds D. Thick, leathery skin

Correct Answer: C. Presence of S3 and S4 heart sounds Acromegaly is an uncommon condition caused by an overproduction of growth hormone (GH) in adults. The onset of acromegaly is usually around age 40-45, as a result of a pituitary adenoma. GH oversecretion results in overgrowth of soft tissues of the face, hands, feet, and organs (including the heart). An enlarged heart (cardiomegaly) places the client at risk for cardiomyopathy and heart failure. The nurse should report extra heart sounds (S3, S4) to the health care provider. Incorrect Answers: [A. Fasting blood glucose of 126 mg/dL] Although joint pain, hyperglycemia (fasting glucose ≥100 mg/dL), and skin changes are associated with acromegaly, these are not as concerning as S3 and S4 heart sounds. [B. Knee pain when walking] Although joint pain, hyperglycemia (fasting glucose ≥100 mg/dL), and skin changes are associated with acromegaly, these are not as concerning as S3 and S4 heart sounds. [D. Thick, leathery skin] Although joint pain, hyperglycemia (fasting glucose ≥100 mg/dL), and skin changes are associated with acromegaly, these are not as concerning as S3 and S4 heart sounds. Educational objective:Acromegaly is an uncommon condition caused by growth hormone overproduction, leading to overgrowth of soft tissues of the face, hands, feet, and organs. Presence of extra heart sounds (ie, S3, S4) may indicate cardiomyopathy or heart failure.

The client recently admitted to the assisted living center has impaired vision related to primary open-angle glaucoma. Select the graphic that best illustrates the effects of glaucoma on the client's vision. A. Spots in vision B. Reduced center of vision C. Reduced outer ring of vision D. Blurry vision

Correct Answer: C. Reduced outer ring of vision Primary open-angle glaucoma (POAG) is an eye condition characterized by an increase in intraocular pressure and gradual loss of peripheral vision (ie, tunnel vision). The signs/symptoms of POAG develop slowly and include painless impairment of peripheral vision with normal central vision, difficulty with vision in dim lighting, increased sensitivity to glare, and halos observed around bright lights. POAG can lead to blindness if left untreated. Incorrect Answers: [A. Spots in vision] Retinal detachment is separation of the retina from the underlying epithelium that allows fluid to collect in the space. The signs/symptoms include sudden onset of light flashes, floaters, cloudy vision, or a curtain appearing in the vision. [B. Reduced center of vision] Age-related macular degeneration is a degenerative eye disease that brings about the gradual loss of central vision, leaving peripheral vision intact. [D. Blurry vision] A cataract is cloudiness (ie, opacity) of the lens that may occur at birth or more commonly in older adults. The signs/symptoms of a cataract include painless, gradual loss of visual acuity with blurry vision; scattered light on the lens producing glare and halos, which are worse at night; and decreased color perception. Educational objective:Primary open-angle glaucoma is characterized by an increase in intraocular pressure and gradual loss of peripheral vision (ie, tunnel vision).

The nurse on a medical surgical unit enters a room, finds a client unresponsive with no pulse, and starts 2 minutes of CPR. The nurse receives and attaches an automated external defibrillator, but no shock is advised. Which action should the nurse perform next? A. Check for a carotid pulse for at least 10 seconds. B. Provide rescue breaths at a rate of 10-12/min C. Resume chest compressions at a rate of 100/min D. Use the jaw-thrust maneuver to assess the airway

Correct Answer: C. Resume chest compressions at a rate of 100/min The basic life support sequence is compressions, airway, and breathing (mnemonic - CAB). High-quality CPR is associated with improved client outcomes and begins with high-quality chest compressions (ie, 100-120/min, 2-2.4 in [5-6 cm] deep). Any unwitnessed collapse should be treated with 2 minutes of CPR, followed by activating the emergency response system and obtaining an automated external defibrillator. If no shock is advised, the nurse should resume high-quality chest compressions immediately. Incorrect Answers: [A. Check for a carotid pulse for at least 10 seconds.] Chest compressions should not be interrupted for more than 10 seconds when assessing for a pulse and chest rise/fall. [B. Provide rescue breaths at a rate of 10-12/min] Rescue breaths every 5-6 seconds (10-12 breaths/min) are given to clients who have a pulse but are not breathing normally. For clients with no pulse, the nurse should deliver cycles of 30 compressions followed by 2 rescue breaths. [D. Use the jaw-thrust maneuver to assess the airway] The jaw-thrust maneuver is used instead of the head-tilt/chin-lift method in clients who may have a head/spinal injury. Repositioning the jaw forward opens the airway to allow for assessment and delivery of rescue breathing. Assessing the airway is not indicated at this time. Educational objective:In basic life support for an unresponsive, pulseless client, the nurse should begin with 2 minutes of CPR in cycles of 30 high-quality chest compressions to 2 rescue breaths, followed by activating the emergency response system and obtaining an automated external defibrillator. If no shock is advised, the nurse should resume chest compressions immediately.

The nurse is caring for a client involved in a motor vehicle collision who had a chest tube inserted to evacuate a pneumothorax caused by fractured ribs. Where would the nurse observe an air leak? A. Section A B. Section B C. Section C D. Section D

Correct Answer: C. Section C The presence of an air leak is indicated by continuous bubbling of fluid at the base of the water seal chamber. If the client has a known pneumothorax, intermittent bubbling would be expected. Once the lung has re-expanded and the air leak is sealed, the bubbling will cease. The nurse is expected to assess for the presence or absence of an air leak and to determine whether it originates from the client or the chest tube system. Incorrect Answers: [A. Section A] Section A is the suction control chamber. Gentle, continuous bubbling indicates that suction is present. [B. Section B] Section B is part of the water seal chamber, but an air leak will not be evident in this upper portion. Tidaling of fluid is expected in this portion of the chamber and indicates patency of the tube. [D. Section D] Section D is the collection chamber, where drainage from the client will accumulate. The nurse will assess amount and color of the fluid and record these as output. Educational objective:An air leak is indicated by bubbling of fluid in the base of the water seal chamber of a chest tube drainage unit. The client with a known pneumothorax is expected to have an intermittent air leak, with bubbling in the water seal chamber. Continuous bubbling indicates an air leak somewhere in the chest tube system.

A client is admitted to the emergency department after a fall with dizziness and light-headedness. Blood pressure is 88/62 mm Hg, and the cardiac monitor displays the rhythm in the exhibit. The nurse recognizes it as which rhythm? A. Complete heart block B. 1st-degree heart block C. Sinus bradycardia D. Sinus rhythm

Correct Answer: C. Sinus bradycardia Sinus bradycardia (SB) has the same conduction pathway as sinus rhythm, but the sinoatrial node fires at a rate of <60/min. SB is classified as symptomatic if, in addition to a heart rate <60/min, the client experiences such symptoms as dizziness, syncope, chest pain, and hypotension. The clinical significance of SB depends on how the client tolerates it. The client with symptomatic SB is first treated with atropine. If atropine is ineffective, transcutaneous pacing or an infusion of dopamine or epinephrine is considered. A permanent pacemaker may be needed. If SB is the result of a medication (eg, beta blocker, digoxin), the drug may need to be held, discontinued, or given in a reduced dosage. Incorrect Answer: [A. Complete heart block] Complete heart block, or 3rd-degree atrioventicular (AV) block, is a form of AV dissociation in which no impulses from the atria are conducted to the ventricles. The atria are stimulated and contract independently of the ventricles. The ventricular rhythm is an escape rhythm. [B. 1st-degree heart block] In 1st-degree AV block, every impulse is conducted to the ventricles, but the time of AV conduction is prolonged. This is evidenced by a prolonged PR interval of >0.20 second. [D. Sinus rhythm] Sinus rhythm has a rate of 60-100/min. Educational objective:The nurse should be able to recognize SB on the ECG and assess for clinical significance (eg, chest pain, syncope, hypotension) in the client. Initial expected treatment for symptomatic clients includes atropine and transcutaneous pacing.

A graduate student, who has been studying for final exams and using energy drinks to stay awake, comes to the clinic reporting a fluttering feeling in the chest. The student is connected to the cardiac monitor that displays the rhythm in the exhibit. The nurse recognizes this as which rhythm? A. Atrial fibrillation B. Sinus rhythm with premature atrial contractions C. Sinus rhythm with premature ventricular contractions D. Ventricular tachycardia

Correct Answer: C. Sinus rhythm with premature ventricular contractions A premature ventricular contraction (PVC) is a contraction coming from an ectopic focus in the ventricles. It is a premature (early) conduction of a QRS complex. PVCs are wide and distorted in shape compared to a QRS conducted through the normal conduction pathway. PVCs can be associated with stimulants (eg, caffeine), medications (eg, digoxin), heart diseases, electrolyte imbalances, hypoxia, and emotional stress. PVCs are usually not harmful in the client with a healthy heart. In the client with myocardial ischemia/infarction, PVCs indicate ventricular irritability and increase the risk for the rhythm to deteriorate into a life-threatening dysrhythmia (eg, ventricular tachycardia, ventricular fibrillation). The nurse should assess the client's physiological response, including apical-radial pulse. Treatment is based on the underlying cause of the PVCs (eg, oxygen for hypoxia, reduction of caffeine intake, electrolyte replacement). Incorrect Answers: [A. Atrial fibrillation] Atrial fibrillation is a total disorganization of atrial electrical activity. P waves are replaced by fibrillatory waves. Ventricular rate varies and the rhythm is usually irregular. [B. Sinus rhythm with premature atrial contractions] A premature atrial contraction (PAC) is a contraction starting from an ectopic focus in the atrium (other than the sinus node) and coming sooner than the next sinus beat. The P wave of a PAC has a different shape than the P wave that originated in the sinus node. [D. Ventricular tachycardia] Ventricular tachycardia has a ventricular rate of 150-250/min and originates from foci firing repeatedly in the ventricle. Educational objective:Premature ventricular contractions (PVCs) are wide and distorted and occur early in the underlying rhythm. They are usually not harmful in the client with a healthy heart. PVCs in the client with myocardial infarction indicate ventricular irritability and should be assessed immediately.

A client with a 10-year history of major depressive disorder has relapsed and is now hospitalized. The client is currently on phenelzine and weighs 115 lb (52.2 kg) but weighed 150 lb (68 kg) 3 months before admission. Which foods would be best for this client? A. Crackers and cheddar cheese B. Hard-boiled egg with tomatoes C. Steamed fish and potatoes D. Tortilla chips with avocado dip

Correct Answer: C. Steamed fish and potatoes The diagnostic criteria for major depressive disorder include a significant, unintentional weight change (loss or gain) of ≥5% in 1 month. A 35-lb (15.9-kg) weight loss within 3 months is a 23% change in this client's usual body weight and is considered severe. The client needs a diet high in calories and protein to promote adequate nutrition and weight gain. In addition, the client may have a low energy level; providing foods that are easier to chew and swallow may improve intake. Appropriate foods that are protein and/or calorie dense include: · Whole milk and dairy products (eg, milkshakes), fruit smoothies · Granola, muffins, biscuits · Potatoes with sour cream and butter · Meat, fish, eggs, dried beans, almond butter · Pasta/rice dishes with cream sauce Incorrect Answers: [A. Crackers and cheddar cheese] The client is taking phenelzine (Nardil), which is a monoamine oxidase inhibitor. Foods high in tyramine (eg, aged cheese, yogurt, cured meats, fermented foods, broad beans, beer, red wine, chocolate, avocados) need to be restricted to reduce the risk of hypertensive crisis. [B. Hard-boiled egg with tomatoes] This choice is an excellent protein option but is low in calories. The client needs foods high in calories and protein. [D. Tortilla chips with avocado dip] This choice provides calories but is low in protein and high in tyramine. The client needs to consume foods that are high in calories and protein but low in tyramine. Educational objective:Clients with major depressive disorder are likely to have a significant change in weight (either increased or decreased). Interventions to promote adequate nutritional intake include providing small frequent meals and snacks that are dense in protein and calories. In addition, drug-food interactions need to be considered when choosing foods for a client taking a monoamine oxidase inhibitor.

The nurse is caring for a client with a chest tube that was placed 2 hours ago for a pneumothorax. Where would the nurse expect gentle, continuous bubbling? A. Air leak monitor B. Collection chamber C. Suction control chamber D. Water seal chamber

Correct Answer: C. Suction control chamber The suction control chamber (Section A) maintains and controls suction to the chest drainage system; continuous, gentle bubbling indicates that the suction level is appropriate. The amount of suction is controlled by the amount of water in the chamber and not by wall suction. Increasing the amount of wall suction would cause vigorous bubbling but does not increase suction to the client as excess suction is drawn out through the vent of the suction control chamber. Vigorous bubbling would increase water evaporation and therefore decrease the negative pressure applied to the system. The nurse should check the water level and add sterile water, if necessary, to maintain the prescribed level. Incorrect Answers: [A. Air leak monitor] The air leak monitor (Section C) is part of the water seal chamber. Continuous or intermittent bubbling seen here indicates the presence of an air leak. [B. Collection chamber] The collection chamber (Section D) is where drainage from the client will accumulate. The nurse will assess amount and color of the fluid and record as output. [D. Water seal chamber] The water seal chamber contains water, which prevents air from flowing into the client. Up and down movement of fluid (tidaling) in Section B would be seen with inspiration and expiration and indicates normal functioning of the system. This will gradually reduce in intensity as the lung reexpands. Educational objective:The nurse should observe gentle, continuous bubbling in the suction control chamber. This indicates patency and the appropriate level of suction being applied to the drainage system.

The home care nurse visits the house of an elderly client. Which assessment finding requires immediate intervention? A. The client cannot remember what was done yesterday B. The client has a painful red area on the buttocks C. The client has new dependent edema of the feet D. The client has strong, foul smelling urine

Correct Answer: C. The client has new dependent edema of the feet New onset of dependent edema of the feet could represent congestive heart failure. This is an urgent medical condition that needs prompt evaluation for characteristic signs (eg, weight gain, lung crackles) and treatment. Incorrect Answers: [A. The client cannot remember what was done yesterday] Loss of short-term memory could be an early sign of dementia. It is important to assess clients' mental status to ensure safety in their homes. Further intervention is required, but this condition is not life-threatening. [B. The client has a painful red area on the buttocks] A painful red area on the buttocks represents the beginning stages of a pressure injury. Although not emergent, this does require further intervention. It is important to recognize pressure injuries early and start treatment promptly before they progress to advanced stages. Advanced pressure injuries are more difficult to treat and heal slower in the elderly. [D. The client has strong, foul smelling urine] Strong, foul smelling urine is likely due to a urinary tract infection. This does require treatment to prevent further complications but is not a priority over suspected heart failure. Urinary tract infections can cause fever with confusion in the elderly. Educational objective:New onset of dependent edema in an elderly client could be due to heart failure; the client needs further assessment for characteristic signs such as lung crackles and increased body weight (fluid retention).

A client diagnosed with pneumonia is experiencing shortness of breath, chest pain, and orthopnea. The chest x-ray reveals a very large right pleural effusion. Which intervention should the nurse anticipate for this client? A. Endotracheal intubation B. Paracentesis C. Thoracentesis D. Ventilation-perfusion scan

Correct Answer: C. Thoracentesis The pleural space is the area between the visceral pleura (membrane lining the lungs) and the parietal pleura (membrane lining the chest cavity). A pleural effusion refers to excess fluid in the pleural space. Small pleural effusions are often asymptomatic, but larger effusions cause dyspnea, diminished breath sounds, and pain. When a pleural effusion causes respiratory difficulty, a thoracentesis is performed to remove the fluid. A needle is inserted into the pleural space, and fluid is removed. Incorrect Answers: [A. Endotracheal intubation] Endotracheal intubation for mechanical ventilation is performed when clients experience acute respiratory failure and are unable to protect their own airways or maintain adequate exchange of oxygen and carbon dioxide in the lungs. If a thoracentesis resolves this client's pleural effusion, endotracheal intubation should be unnecessary. [B. Paracentesis] A paracentesis is a procedure to remove ascites, a fluid buildup in the peritoneal cavity. [D. Ventilation-perfusion scan] A ventilation-perfusion scan (V/Q) is used to diagnose a pulmonary embolism. Perfusion scanning uses a radioisotope to visualize blood flow through the lungs; ventilation scanning involves inhalation of a radioisotope to visualize air movement through the lungs. This procedure is not used to assess pleural effusions. Educational objective:When a large pleural effusion interferes with breathing, a thoracentesis is performed to remove fluid from the pleural space.

The health care provider prescribes a multivitamin regimen that includes thiamine for a client with a history of chronic alcohol abuse. The nurse is aware that thiamine is given to this client population for which purpose? A. To lower the blood alcohol level B. To prevent gross tremors C. To prevent Wernicke encephalopathy D. To treat seizures related to acute alcohol withdrawal

Correct Answer: C. To prevent Wernicke encephalopathy Clients with chronic alcohol abuse suffer from poor nutrition related to improper diet and altered nutrient absorption. Poor thiamine intake and/or absorption can lead to Wernicke encephalopathy, a serious complication that manifests as altered mental status, oculomotor dysfunction, and ataxia. Clients are prescribed thiamine to prevent this condition. Incorrect Answers: [A. To lower the blood alcohol level] Thiamine is not used to reduce the blood alcohol level; there is no antidote for elevated blood alcohol levels. Supportive measures are used until levels fall and the client is no longer experiencing acute withdrawal symptoms. [B. To prevent gross tremors] Gross tremors related to acute alcohol withdrawal are treated with benzodiazepines (eg, lorazepam, diazepam). [D. To treat seizures related to acute alcohol withdrawal] Seizures related to acute alcohol withdrawal are treated with benzodiazepines (eg, lorazepam, diazepam, chlordiazepoxide). Thiamine is used to prevent encephalopathy. Encephalopathy may lead to seizures, but thiamine is not used to treat seizures. Educational objective:A major complication of chronic alcohol abuse is encephalopathy related to poor thiamine absorption. It is critical that these clients receive thiamine replacement. Wernicke encephalopathy can lead to more significant and progressive complications, including death.

What information would be most important for the nurse to obtain from a client with suspected bladder cancer who reports blood in the urine but no associated pain? A. Family History B. Industrial chemical exposure C. Tobacco use D. Usual diet

Correct Answer: C. Tobacco Use The telltale symptom of bladder cancer, seen in >75% of cases, is painless hematuria; the client will report seeing blood in the urine but will have no associated pain. As with many other types of cancer, the primary factor contributing to bladder cancer is cigarette smoking or other tobacco use. Poorer outcomes are seen in those who have smoked longer and those who have smoked a higher number of cigarette packs per day. Incorrect Answers: [A. Family History] Clients who have family members with bladder cancer have an increased risk of developing bladder cancer themselves; however, the primary risk factor is tobacco use. [B. Industrial chemical exposure] Occupational carcinogen exposure is the second most common risk factor. Occupational exposures include printing, iron and aluminum processing, industrial painting, metal work, machining, and mining. Clients are exposed to carcinogens through direct skin contact and inhalation (aerosols, vapors). [D. Usual diet] Consuming a high-fat diet and using artificial sweeteners may contribute to the development of bladder cancer, but tobacco use is the most commonly associated contributing factor. Educational objective:Painless hematuria is the most common presenting symptom of bladder cancer. Cigarette smoking or other tobacco use is the primary risk factor.

A client comes to the emergency department with diplopia and recent onset of nausea. Which statement by the client would indicate to the nurse that this is an emergency? A. "I am very tired, and it's hard for me to keep my eyes open." B. "I don't feel good, and I want to be seen." C. "I have not taken my blood pressure medicine in over a week." D. "I have the worst headache I've ever had in my life."

Correct Answer: D. "I have the worst headache I've ever had in my life." A ruptured cerebral aneurysm is a surgical emergency with a high mortality rate. Cerebral aneurysms are usually asymptomatic unless they rupture; they are often called "silent killers" as they may go undetected for many years before rupturing without warning signs. The distinctive description of a cerebral aneurysm rupture is the abrupt onset of "the worst headache of my life" that is different from previous headaches (including migraines). Immediate evaluation for a possible ruptured aneurysm is critical for any client experiencing a severe headache with changes in or loss of consciousness, neurologic deficits, diplopia, seizures, vomiting, or a stiff neck. Early identification and prompt surgical intervention help increase the chance for survival. Incorrect Answers: [A. "I am very tired, and it's hard for me to keep my eyes open."] A change in level of consciousness, increased blood pressure, or a feeling of illness should be investigated but alone does not indicate an emergency. [B. "I don't feel good, and I want to be seen."] A change in level of consciousness, increased blood pressure, or a feeling of illness should be investigated but alone does not indicate an emergency. [C. "I have not taken my blood pressure medicine in over a week."] A change in level of consciousness, increased blood pressure, or a feeling of illness should be investigated but alone does not indicate an emergency. Educational objective:Sudden onset of a severe headache described as "the worst headache of my life" is characteristic of a ruptured cerebral aneurysm and should be treated as an emergency.

The long-term care nurse is caring for a client diagnosed with macular degeneration. Which client statement supports this diagnosis? A. "I have been seeing small flashes of light." B. "I have trouble threading my sewing needle. I have to hold it far away to see it." C. "I notice that my peripheral vision is becoming worse." D. "I see a blurry spot in the middle of the page when I read."

Correct Answer: D. "I see a blurry spot in the middle of the page when I read." Macular degeneration is a progressive, incurable disease of the eye in which the central portion of the retina, the macula, begins to deteriorate. This deterioration causes distortion (blurred or wavy visual disturbances) or loss of the central field of vision, whereas the peripheral vision remains intact. Macular degeneration has two different etiologies. "Dry" macular degeneration involves ischemia and atrophy of the macula that results from blockage of the retinal microvasculature. "Wet" macular degeneration involves the abnormal growth of new blood vessels in the macula that bleed and leak fluid, eventually destroying the macula. Progression of macular degeneration may be slowed with smoking cessation, intake of specific supplements (eg, carotenoids, vitamins C and E), laser therapy, and injection of antineoplastic medications. Risk factors for macular degeneration include advanced age, family history, hypertension, smoking, and long-term poor intake of carotenoid-containing fruits and vegetables. Incorrect Answers: [A. "I have been seeing small flashes of light."] Seeing small flashes of light is associated with retinal detachment. [B. "I have trouble threading my sewing needle. I have to hold it far away to see it."] Inability to see things close up, known as presbyopia, occurs when the lens of the eye becomes less elastic with age and therefore unable to adjust to near and far vision. [C. "I notice that my peripheral vision is becoming worse."] Poor peripheral vision, also called tunnel vision, can result from optic nerve damage seen in glaucoma. Educational objective:Macular degeneration is a progressive, incurable eye disease that occurs when the central portion of the retina deteriorates, giving rise to distortion (blurred or wavy visual disturbances) or loss in the center of the visual field.

The nurse in the outpatient clinic is speaking with a client diagnosed with cerebral arteriovenous malformation. Which statement would be a priority for the nurse to report to the health care provider? A. "I got short of breath this morning when I worked out." B. "I have cut down on smoking to 1/2 pack per day." C. "I haven't been feeling well, so I have been sleeping a lot." D. "I took an acetaminophen in the waiting room for this bad headache."

Correct Answer: D. "I took an acetaminophen in the waiting room for this bad headache." An arteriovenous malformation (AVM) is a tangle of veins and arteries that is believed to form during embryonic development. The tangled vessels do not have a capillary bed, causing them to become weak and dilated. AVMs are usually found in the brain and can cause seizures, headaches, and neurologic deficits. Treatment depends on the location of the AVM, but blood pressure control is crucial. Clients with AVMs are at high risk for having an intracranial bleed as the veins can easily rupture because they lack a muscular layer around their lumen. Any neurologic changes, sudden severe headache, nausea, and vomiting should be evaluated immediately as these are usually the first symptoms of a hemorrhage. Incorrect Answers: [A. "I got short of breath this morning when I worked out."] The report of dyspnea may prompt further evaluation depending on the type of exercise performed, but it is not the priority. Clients with AVMs should be discouraged from engaging in heavy exercise as it increases blood pressure. [B. "I have cut down on smoking to 1/2 pack per day."] Clients with AVMs should avoid smoking to prevent hypertension. This client needs education on smoking cessation, but it is not the priority. [C. "I haven't been feeling well, so I have been sleeping a lot."] Reports of not feeling well and sleeping a lot may be related to the headache and possible hemorrhage, but this alone would not prompt a call to the health care provider. Educational objective:An arteriovenous malformation is a congenital deformity of tangled blood vessels often occurring in the brain. These vessels may weaken and rupture, causing an intracranial hemorrhage. Any neurologic changes and severe headache need to be addressed immediately as these may indicate hemorrhage.

A client with atrial fibrillation is being discharged home after being stabilized with medications, including digoxin. Which client statement regarding digoxin toxicity indicates that teaching reinforcement is needed? A. "I must visit my health care provider to check my drug levels." B. "I should report to my health care provider if I develop nausea and vomiting." C. "I should tell my health care provider if my heart rate is below 60 beats per minute." D. "I will need to increase my potassium intake."

Correct Answer: D. "I will need to increase my potassium intake." Drug toxicity is common with digoxin use due to its narrow therapeutic range. Many contributing factors (eg, hypokalemia) can cause toxicity. However, in the absence of other factors, potassium intake does not need to be increased when a client is taking digoxin. If the client also takes other potassium-depleting medications (eg, diuretics), potassium supplements may be needed. Signs and symptoms of digoxin toxicity include the following: — Gastrointestinal symptoms (eg, anorexia, nausea, vomiting, abdominal pain) are frequently the earliest symptoms — Neurologic manifestations (eg, lethargy, fatigue, weakness, confusion) — Visual symptoms are characteristic and include alterations in color vision, scotomas, or blindness — Cardiac arrhythmias are the most dangerous Incorrect Answers: [A. "I must visit my health care provider to check my drug levels."] Drug levels are frequently monitored not only until a steady state is achieved but also when changes are expected, such as in clients with chronic kidney disease and electrolyte disturbances (eg, hypokalemia, hypomagnesemia). [C. "I should tell my health care provider if my heart rate is below 60 beats per minute."] Digoxin toxicity can result in bradycardia and heart block. Clients are instructed to check their pulse and report to the health care provider if it is low or has skipped beats. Educational objective:Drug toxicity is common with digoxin due to its narrow therapeutic range. Drug levels are frequently monitored. Nonspecific gastrointestinal symptoms similar to gastroenteritis are common and can lead to serious cardiac arrhythmias if not recognized.

The nurse is reinforcing teaching to a client with a hiatal hernia. Which statement by the client indicates that further teaching is needed? A. "I need to raise the head of my bed on blocks by at least 6 inches." B. "I will remain sitting up for several hours after I eat any food." C. "If my reflux and abdominal pain don't improve, I might need surgery." D. "Losing weight may reduce my reflux, so I plan to take a weight-lifting class."

Correct Answer: D. "Losing weight may reduce my reflux, so I plan to take a weight-lifting class." Hiatal hernia is a group of medical conditions characterized by abnormal movement of the stomach and/or esophagogastric junction into the chest due to a weakness in the diaphragm. Although hiatal hernias may be asymptomatic, many people experience heartburn, chest pain, dysphagia, and shortness of breath when the abdominal organs move into the chest. Symptoms of hiatal hernias are often exacerbated by increased abdominal pressure, which promotes upward movement of abdominal organs. Clients with hiatal hernias who are obese are often encouraged to lose excess weight by performing light activities (eg, short walks) because obesity increases abdominal pressure. However, nurses should teach clients to avoid activities that promote straining (eg, weight lifting), which increases abdominal pressure. Incorrect Answers: [A. "I need to raise the head of my bed on blocks by at least 6 inches."] Sitting up for several hours after meals and sleeping with the head of the bed elevated at least 6 inches (15 cm) reduces upward movement of the hernia and decreases the risk of gastric reflux. [B. "I will remain sitting up for several hours after I eat any food."] Sitting up for several hours after meals and sleeping with the head of the bed elevated at least 6 inches (15 cm) reduces upward movement of the hernia and decreases the risk of gastric reflux. [C. "If my reflux and abdominal pain don't improve, I might need surgery."] If symptoms of hiatal hernias are uncontrolled with home management (eg, weight loss, diet modification, positioning after meals), surgical revision of the diaphragm may be required to prevent organ movement. Educational objective:Hiatal hernia is characterized by abnormal movement of the stomach and/or esophagogastric junction into the chest due to diaphragmatic weakness. Nurses educating clients with hiatal hernias about symptom management should instruct them to avoid activities that increase abdominal pressure (eg, weight lifting), sleep with the head of the bed elevated, and remain upright for several hours after meals.

A nurse is reinforcing discharge instructions to a client during the fifth hospitalization for pulmonary edema caused by congestive heart failure exacerbation. Which statement by the client indicates that further teaching is required? A. "I should supplement my potassium intake." B. "I should weigh myself daily." C. "Moderate exercise may be helpful in my condition." D. "Potato chips are an acceptable snack in moderation."

Correct Answer: D. "Potato chips are an acceptable snack in moderation." The client is likely dealing with some level of denial regarding the diagnosis of congestive heart failure. Any client statement that does not indicate understanding of the importance of salt avoidance should be addressed to help the client avoid further hospitalizations related to this condition. Incorrect Answers: [A. "I should supplement my potassium intake."] Adding potassium to a diet, especially when substituting it for sodium, can decrease blood pressure and fluid retention. Some diuretics, such as furosemide (Lasix), may also cause low potassium levels. [B. "I should weigh myself daily."] Tracking the level of fluid retention with daily weigh-ins is the easiest way for clients and health care providers to monitor the effects of medication on congestive heart failure. [C. "Moderate exercise may be helpful in my condition."] Physical activity is very important in preserving cardiac function. Educational objective:In congestive heart failure, many changes in the client's established dietary habits are necessary to avoid repeated hospitalizations caused by salt overload.

The nurse monitors a client who has followed a vegan diet for several years. Which client statement would indicate a possible complication resulting from a vegan diet? A. "I have had some visual disturbances while driving at night." B. "I have had trouble falling asleep over the past few months." C. "Scaly patches of skin are developing on my elbows and knees." D. "Sometimes my hands and feet get a tingling sensation."

Correct Answer: D. "Sometimes my hands and feet get a tingling sensation." People who follow a vegan diet eat only plant-based foods and omit animal proteins and byproducts such as milk and eggs. They do not have an adequate intake of vitamin B12 (cobalamin) and are at risk for developing megaloblastic anemia and neurological manifestations. Vitamin B12 deficiency affects the entire nervous system, from peripheral nerves to the spinal cord and brain. Peripheral neuropathy manifests as tingling and numbness. Spinal cord involvement can cause gait problems. Brain involvement causes memory loss/dementia (late). Vitamin B12 is obtained only from animal proteins, so clients who follow a vegan diet must be educated regarding supplementation. Incorrect Answers: [A. "I have had some visual disturbances while driving at night."] Visual disturbances, difficulty falling asleep, and scaly patches of skin are not complications of a vegan diet. [B. "I have had trouble falling asleep over the past few months."] Visual disturbances, difficulty falling asleep, and scaly patches of skin are not complications of a vegan diet. [C. "Scaly patches of skin are developing on my elbows and knees."] Visual disturbances, difficulty falling asleep, and scaly patches of skin are not complications of a vegan diet. Educational objective:Clients who follow a vegan diet should be taught about vitamin B12 deficiency and the importance of supplementation. Vitamin B12 deficiency affects the entire nervous system, from peripheral nerves to the spinal cord and brain.

The nurse is reinforcing discharge instructions for a postoperative client who had a partial laryngectomy for laryngeal cancer. The client is concerned because the health care provider said there was damage to the ninth cranial nerve. Which statement made by the nurse is most appropriate? A. "I will ask the health care provider to explain the consequences of your procedure." B. "This is a common complication that will require you to have a hearing test every year." C. "This is a common complication; your health care provider will order a consult for the speech pathologist." D. "This is the reason you are using a special swallowing technique when you eat and drink."

Correct Answer: D. "This is the reason you are using a special swallowing technique when you eat and drink." Cranial nerve IX (glossopharyngeal) is involved in the gag reflex, ability to swallow, phonation, and taste. Postoperative partial laryngectomy clients will need to undergo evaluation by a speech pathologist to evaluate their ability to swallow safely to prevent aspiration. Clients are taught the supraglottic swallow, a technique that allows them to have voluntary control over closing the vocal cords to protect themselves from aspiration. Clients are instructed to: 1. Inhale deeply 2. Hold breath tightly to close the vocal cords 3. Place food in mouth and swallow while continuing to hold breath 4. Cough to dispel remaining food from vocal cords 5. Swallow a second time before breathing Incorrect Answers: [A. "I will ask the health care provider to explain the consequences of your procedure."] This would be considered "passing the buck." The nurse should try to address the client's concerns before calling the health care provider. [B. "This is a common complication that will require you to have a hearing test every year."] Cranial nerve VIII (vestibulocochlear) affects hearing and equilibrium, not swallowing. [C. "This is a common complication; your health care provider will order a consult for the speech pathologist."] The speech pathologist conducts a swallowing assessment early on to evaluate a client's ability to swallow safely. This consult is not done at discharge. Educational objective:Clients who undergo a partial laryngectomy are at increased risk for aspiration. As a result, they are taught a swallowing technique (supraglottic swallow) to decrease this risk.

The nurse is caring for a client with increased intracranial pressure (ICP). Which statement by the unlicensed assistive personnel would require immediate intervention by the nurse? A. "I will raise the head of the bed so it is easier to see the television." B. "I will turn down the lights when I leave." C. "Let me move your belongings closer so you can reach them." D. "You should do deep breathing and coughing exercises."

Correct Answer: D. "You should do deep breathing and coughing exercises." Clients with elevated ICP should avoid anything that increases intrathoracic or intraabdominal pressure as these also indirectly increase ICP. These activities include straining, coughing, and blowing the nose. Respiratory interventions, if needed, may include deep breathing and incentive spirometry in the absence of coughing. The head of the bed should be maintained at 30 degrees, high enough to allow for cerebrospinal fluid drainage, but low enough to maintain cerebral perfusion pressure. Clients should have minimal stimuli, including no bright lights or multiple visitors, as stimulation can increase ICP. Incorrect Answers: [A. "I will raise the head of the bed so it is easier to see the television."] The head of the bed should be raised to 30 degrees and maintained. This may require follow-up, but not immediate intervention, by the nurse. [B. "I will turn down the lights when I leave." ] Turning down the lights is appropriate as clients with ICP benefit from a quiet, nonstimulating environment. [C. "Let me move your belongings closer so you can reach them."] Bringing items closer to the client prevents straining from reaching and is appropriate. Educational objective:Clients with increased ICP should be encouraged not to cough, strain, or increase abdominal or thoracic pressure. The head of the bed should be maintained at 30 degrees, and stimulation in the room should be minimized.

A home health nurse visits a client with chronic obstructive pulmonary disease. The nurse reinforces the use of the "huff" coughing technique to facilitate secretion removal. Place the steps in the correct order. All options must be used. 1. "Hold your breath for 2-3 seconds and then forcefully exhale quickly." 2. "Perform a slow, deep inhalation with your mouth using your diaphragm." 3. "Repeat the huff once or twice more, while refraining from performing a normal cough." 4. "Rest for 5-10 normal breaths and repeat as necessary until mucus is cleared." 5. "Sit upright in a chair with feet spread shoulder-width apart and lean forward." A. 5, 2, 1, 3, 4 B. 5, 2, 1, 4, 3 C. 5, 1, 2, 3, 4 D. 5, 2, 1, 3, 4

Correct Answer: D. 5, 2, 1, 3, 4 Clients with chronic obstructive pulmonary disease (COPD) often develop ineffective coughing patterns due to weakened muscles and narrowed airways prone to collapse under increased pressure. Therefore, clients with COPD are unable to cough effectively and require additional teaching to effectively expectorate secretions and prevent overexertion. Huff coughing is a series of low-pressure coughs using the following steps: — Sit upright in a chair with feet spread shoulder-width apart and lean forward with shoulders relaxed; forearms supported on thighs or pillows; head and knees slightly flexed; and feet touching the floor — Perform a slow, deep inhalation through the mouth or nose using the diaphragmatic muscle — Hold breath for 2-3 seconds, keeping the throat open, and then perform a quick, forceful exhalation, creating an audible "huff" sound — Repeat the "huff" once or twice more to expectorate any mucus — Rest for 5-10 regular breaths and repeat as necessary until all mucus is cleared Educational objective:Clients with chronic obstructive pulmonary disease benefit from breathing techniques to facilitate effective coughing. Huff coughing is a forced expiratory technique in which the client sits relaxed, upright, and leaning forward; slowly inhales using the diaphragmatic muscle; holds breath for 2-3 seconds and then quickly exhales; and repeats as necessary until remaining secretions are clear.

The practical nurse is assisting the registered nurse in caring for 4 clients. Which client is at greatest risk for the development of deep venous thrombosis? A. 25-year-old client with abdominal pain who smokes cigarettes and takes oral contraceptives B. 55-year-old ambulatory client with exacerbation of chronic bronchitis and hematocrit of 56% C. 72-year-old client with a fever who is 2 days post coronary stent placement D. 80-year-old client who is 4 days postoperative from repair of a fractured hip

Correct Answer: D. 80-year-old client who is 4 days postoperative from repair of a fractured hip Venous thromboembolism includes both deep venous thrombosis (DVT) and pulmonary embolism (PE). DVT is more common and occurs most often (80%) in the proximal deep veins (iliac, femoral) of the lower extremities. Virchow's triad describes the 3 most common theories behind the pathophysiology of venous thrombosis: venous stasis, endothelial damage, and hypercoagulability of blood. Risk factors associated with DVT formation include the following: 1. Trauma (endothelial injury and venous stasis from immobility) 2. Major surgery (endothelial injury and venous stasis from immobility) 3. Prolonged immobilization (eg, stroke, long travel) causing venous stasis 4. Pregnancy (induced hypercoagulable state and some venous stasis by the pressure on inferior vena cava) 5. Oral contraceptives (estrogen is thrombotic) 6. Underlying malignancy (cancer cells release procoagulants) 7. Smoking (produces endothelial damage by inflammation) 8. Old age 9. Obesity and varicose veins (venous stasis) 10. Myeloproliferative disorders (increase in blood viscosity from an increase in one or more blood cell types) such as polycythemia vera The 80-year-old 4-day postoperative client is at greatest risk for developing DVT due to having the most risk factors: orthopedic hip surgery, prolonged period of immobility/inactivity, and advanced age. Incorrect Answers: [A. 25-year-old client with abdominal pain who smokes cigarettes and takes oral contraceptives] Smoking cigarettes and using oral contraceptives increase plasma fibrinogen and coagulation factors and cause hypercoagulability of blood; however, this client is not at greatest risk. Hormonal contraceptives are not recommended if the client is age >35 and smokes. [B. 55-year-old ambulatory client with exacerbation of chronic bronchitis and hematocrit of 56%] An elevated hemoglobin/hematocrit level (erythrocytosis) causes increased blood viscosity and hypercoagulability of blood, which increase the risk for DVT. However, this client is not at greatest risk. [C. 72-year-old client with a fever who is 2 days post coronary stent placement] Anticoagulants and antiplatelet agents are administered before and after coronary stent placement. This client is at increased risk due to endothelial damage and advanced age but is not at greatest risk. Educational objective:Deep venous thrombosis (DVT) is a frequent, often preventable complication of hospitalization, surgery, and immobilization. Factors that increase the risk for developing DVT include trauma, surgery (especially orthopedic, knee, hip), prolonged immobility/inactivity, oral contraceptives, pregnancy, varicose veins, obesity, smoking, and advanced age.

The nurse is caring for a client admitted to the hospital 2 days ago with acute pancreatitis. Which finding is the most concerning? A. Blood glucose levels for the past 24 hours are ≥250 mg/dL (13.9 mmol/L) B. Client is lying with knees drawn up to the abdomen to alleviate pain C. Five large, liquid stools are yellow and foul-smelling D. Temperature is 102.2 F (39 C) and abdominal pain is increasing

Correct Answer: D. Temperature is 102.2 F (39 C) and abdominal pain is increasing Clients with acute pancreatitis are at risk for pancreatic abscess development. This mainly results from secondary infection of pancreatic pseudocysts or pancreatic necrosis. High fever, increasing abdominal pain, and leukocytosis may indicate abscess formation. The abscess must be treated promptly to prevent sepsis. The health care provider should be notified immediately as antibiotic therapy and immediate surgical management may be required. Incorrect Answers: [A. Blood glucose levels for the past 24 hours are ≥250 mg/dL (13.9 mmol/L)] Elevated blood glucose is an expected finding in clients with pancreatitis. Elevated blood glucose is associated with pancreatic dysfunction and may necessitate insulin administration, but this is not the most concerning finding. [B. Client is lying with knees drawn up to the abdomen to alleviate pain] Clients with acute pancreatitis often report severe, burning midepigastric abdominal pain that radiates to the back. Clients may seek relief from pain by positioning themselves in the knee-chest position to decrease intra-abdominal pressure. Pain relief interventions should be attempted, but this is not the priority. [C. Five large, liquid stools are yellow and foul-smelling] The client with pancreatitis may develop steatorrhea (eg, fatty, yellow, foul-smelling stools) due to a decrease in lipase production. Although fluid and nutritional status are important, these does not take precedence over a possible surgical emergency. Educational objective:Acute pancreatitis may cause severe midepigastric abdominal pain, elevated blood glucose levels, and steatorrhea. The nurse should watch closely for high fever, increasing abdominal pain, and leukocytosis as these findings may indicate infection of the necrosed pancreas or pancreatic abscess formation.

The nurse receives the handoff of care report on four clients. Which client should the nurse see first? A. Client reporting incisional pain of 8 on a scale of 0-10 with a respiratory rate of 25/min who had a right pneumonectomy 12 hours ago B. Client with a left pleural effusion who has crackles, absent breath sounds in the left base, and an SpO2 of 94% on room air C. Client with a temperature of 100.4 F (38 C) and a respiratory rate of 12/min who had a small bowel resection 1 day ago D. Client with a temperature of 100.4 F (38 C) and a respiratory rate of 12/min who had a small bowel resection 1 day ago

Correct Answer: D. Client with a temperature of 100.4 F (38 C) and a respiratory rate of 12/min who had a small bowel resection 1 day ago Acute respiratory failure (ARF) is a life-threatening impairment of the lungs' ability to oxygenate blood and excrete carbon dioxide (CO2). ARF may occur from exacerbation of chronic (eg, chronic obstructive pulmonary disease, asthma) or acute (eg, pneumonia, pulmonary edema) illnesses. Nurses assessing for signs of ARF should consider both respiratory and neurological manifestations. Altered mental status (eg, confusion, agitation, somnolence) is a common and often overlooked symptom that may occur because of the brain's sensitivity to inadequate oxygenation and alterations in acid-base balance from retained CO2. Additional signs and symptoms may include paresthesias, dyspnea, tachypnea, and hypoxemia. Incorrect Answers: [A. Client reporting incisional pain of 8 on a scale of 0-10 with a respiratory rate of 25/min who had a right pneumonectomy 12 hours ago] Clients recovering from recent pneumonectomy (ie, surgical removal of part or all of the lung) often experience considerable pain, which may cause respiratory distress if not adequately controlled. A client with tachypnea and severe pain should be seen promptly but only after addressing potential ARF. [B. Client with a left pleural effusion who has crackles, absent breath sounds in the left base, and an SpO2 of 94% on room air] Crackles, absent or diminished breath sounds over the affected lobe, and slightly decreased oxygen saturation are expected findings in pleural effusion, in which fluid collects in the space surrounding the lung. [C. Client with a temperature of 100.4 F (38 C) and a respiratory rate of 12/min who had a small bowel resection 1 day ago] Low-grade fever may occur following surgery (due to the release of inflammatory cytokines) or from postoperative atelectasis. The client should be encouraged to ambulate and deep-breathe. Educational objective:Acute respiratory failure is a life-threatening impairment of lung function that inhibits gas exchange. Common symptoms include altered mental status (eg, confusion, agitation, somnolence), paresthesias, dyspnea, tachypnea, and hypoxemia, all of which should be addressed immediately.

A client with active herpes lesions has new prescriptions for oral acyclovir and topical lidocaine. What discharge teaching will the nurse reinforce to the client? A. Adhesive bandaging should remain on the lesions to prevent virus shedding B. Blood tests will be drawn to ensure that the virus is eradicated C. Condoms should be used during intercourse until the lesions are healed D. Gloves should be used to apply the medication to the lesions

Correct Answer: D. Gloves should be used to apply the medication to the lesions Acyclovir (Zovirax), famciclovir, and valacyclovir are commonly used to treat herpes infection as they shorten the duration and severity of active lesions. Genital herpes is a sexually transmitted infection caused by a herpes simplex virus and is highly contagious, especially when lesions are active. It remains dormant in the body even when active lesions are healed; however, it is still contagious, even when dormant. The infection can be spread to other people or other parts of the body via skin-to-skin contact; therefore, gloves should be used when applying topical antiviral or analgesic (eg, lidocaine) medications. There is no cure for genital herpes; treatment is aimed at relieving symptoms and preventing the spread of infection. Incorrect Answers: [A. Adhesive bandaging should remain on the lesions to prevent virus shedding] Herpetic lesions should be kept clean and dry. They can be cleansed with warm water and soap or other solutions. Bandages should not be applied to the lesions. [B. Blood tests will be drawn to ensure that the virus is eradicated] There is no cure for herpes infection, and recurrence of active lesions is common. Some clients may need long-term suppressive therapy. [C. Condoms should be used during intercourse until the lesions are healed] During periods of active lesions, barrier contraception is not sufficient to prevent the spread of infection; therefore, abstinence from sexual intercourse is indicated. Condoms should be used during periods of dormancy due to viral shedding. Educational objective:Genital herpes is a highly contagious, sexually transmitted infection for which there is no cure. Recurrences are common. Clients should be taught methods to prevent the spread of infection (using gloves to apply topical medication to active lesions, practicing abstinence from sexual intercourse when active lesions are present, using condoms when the virus is dormant).

The nurse assesses a client who had a thyroidectomy 8 hours ago. The nurse finds the client anxious, with tingling around the mouth and muscle twitching in the right arm. Which action is most important for the nurse to implement first? A. Change the surgical dressing to assess for bleeding B. Document the findings in the electronic medical record C. Draw arterial blood gases D. Obtain a serum calcium level

Correct Answer: D. Obtain a serum calcium level Normal serum calcium is 8.6-10.2 mg /dL (2.15-2.55 mmol/L). Hypocalcemia (serum calcium <8.6 mg/dL [2.15 mmol/L]) is a potential complication of thyroidectomy because the parathyroids that regulate calcium levels in the blood are accidentally removed during this surgical procedure. The nurse should monitor the client closely for signs of hypocalcemia, which include tetany (overactive neurological responses such as tingling in the hands, feet, and around the mouth; spasms or cramps that can occur even in the larynx; positive Trousseau or Chvostek sign). A serum calcium level should be drawn, and the nurse should ensure that calcium gluconate is readily available in case this complication occurs. Incorrect Answers: [A. Change the surgical dressing to assess for bleeding] Monitoring for bleeding is an important assessment. However, when bleeding occurs post thyroidectomy, blood typically trickles and pools behind the client's neck. This client's symptoms are more consistent with hypocalcemia. [B. Document the findings in the electronic medical record] It is important to document findings in the electronic medical record, but the nurse should do this after taking action to help the client. [C. Draw arterial blood gases] If laryngeal spasm occurs as a result of hypocalcemia, hypoxia may be evident in arterial blood gases. However, the client will also exhibit signs of hypoxemia (eg, stridor, respiratory distress, low pulse oximetry reading). Checking the calcium level so that effective treatment can begin is the highest priority. Educational objective:Normal serum calcium is 8.6-10.2 mg /dL (2.15-2.55 mmol/L). Hypocalcemia (serum calcium <8.6 mg/dL [2.15 mmol/L]) is a potential complication of thyroidectomy. The nurse should monitor for signs and symptoms of tetany (tingling of hands, toes, and circumoral region; positive Trousseau or Chvostek sign), confirm with serum calcium results, and administer calcium gluconate as prescribed. Untreated clients can develop life-threatening laryngeal spasm.

A client undergoes transurethral resection of the prostate for benign prostatic hyperplasia. The client has a 28 Fr, 30-mL balloon, 3-way Foley catheter with continuous bladder irrigation. Which finding by the nurse best indicates that the bladder irrigation is running at an adequate rate? A. Blood pressure 120/80 mm Hg, pulse 80/min B. Intake 3200 mL, output 3000 mL C. Lack of bladder spasms D. Output urine is light pink in color

Correct Answer: D. Output urine is light pink in color Transurethral resection of the prostate (TURP) involves insertion of a rectoscope to excise obstructing prostate tissue. Continuous bladder irrigation is initiated after the procedure. The large catheter and balloon apply direct pressure on the bleeding tissue and allow urine to drain. A specific rate is not prescribed; the irrigation flow is adjusted so that the urine remains light pink without clots. Typically, the irrigation rate will gradually decrease during the first 24 hours. Incorrect Answers: A. Blood pressure 120/80 mm Hg, pulse 80/min: Vital signs within normal limits indicate hemodynamic stability but not patency of the draining catheter from irrigation. B. Intake 3200 mL, output 3000 mL: Overall, the intake and output should be equal (considering approximately 400-500 mL/day of insensible loss). If the negative balance is ≥500 mL, further assessment/intervention is needed. However, fluid balance is not the best indicator of irrigation infusion rate in these clients. C. Lack of bladder spasms: Painful bladder spasms are expected after TURP and catheter placement. The spasms are typically treated with belladonna-opium suppositories or other antispasmodics (eg, anticholinergics such as oxybutynin [Ditropan]). The nurse should remind the client to refrain from trying to void around the catheter as this can trigger the spasms. Educational objective:After transurethral resection of the prostate, the rate of continuous bladder irrigation is adjusted to keep urinary output light pink in color. Bladder spasms (treated with belladonna-opium suppositories) are expected after the procedure.

A client with a C3 spinal cord injury has a headache and nausea. The client's blood pressure is 170/100 mm Hg. How should the nurse respond initially? A. Administer PRN analgesic medication B. Administer PRN antihypertensive medication C. Lower the head of the bed D. Palpate the client's bladder

Correct Answer: D. Palpate the client's bladder Autonomic dysreflexia is an acute, life-threatening response to noxious stimuli, which clients with spinal cord injuries above T6 are unable to feel. Signs and symptoms include hypertension, bradycardia, a pounding headache, diaphoresis, and nausea. It is essential that the nurse assess for and remove noxious stimuli to prevent a stroke. Noxious stimuli may include: — Bladder distention (eg, obstructed urinary catheter, neurogenic bladder) — Fecal impaction — Tight clothing (eg, shoelaces, waistbands) Incorrect Answers: [A. Administer PRN analgesic medication] Hypertension, headache, and nausea due to uncontrolled sympathetic activity will resolve once the cause is identified and removed. [B. Administer PRN antihypertensive medication] Hypertension, headache, and nausea due to uncontrolled sympathetic activity will resolve once the cause is identified and removed. [C. Lower the head of the bed] Lowering the head of the bed would increase blood pressure. The head of the bed should be raised to lower the blood pressure. Educational objective:Autonomic dysreflexia is an acute, life-threatening response to noxious stimuli (eg, bladder distention, tight clothing) in clients with spinal cord injuries above T6. Signs and symptoms include hypertension, bradycardia, a pounding headache, diaphoresis, and nausea. The nurse must immediately identify and remove noxious stimuli to prevent a stroke and resolve symptoms.

A 62-year-old client admitted to the telemetry unit after an acute myocardial infarction 3 days ago reports that the left calf is very tender and warm to the touch. Which nursing intervention is the priority? A. Asking the client how long the leg has been tender and warm B. Checking the electrocardiogram for ectopic beats C. Obtaining vital signs, including pulse oximetry D. Performing a neurovascular check on the lower extremities

Correct Answer: D. Performing a neurovascular check on the lower extremities This client with a tender calf that is warm to the touch exhibits signs and symptoms of a possible deep venous thrombosis (DVT). The client also has several risk factors for DVT (eg, age >60, being hospitalized and in bed for 3 days). The nurse will need to notify the supervising registered nurse and health care provider (HCP) immediately after assessing the lower extremities. The priority action by the nurse should include a thorough neurovascular assessment of the extremities for presence and quality of dorsalis pedis and posterior tibial pulses, temperature of the extremities, capillary refill, and circumference measurements of both calves and thighs. Both extremities should be assessed for comparison. Incorrect Answers: [A. Asking the client how long the leg has been tender and warm] Determining duration of symptoms, assessing the electrocardiogram, and obtaining current vital signs are all interventions that the nurse should report to the HCP. However, they are not as high a priority or as relevant to the client's specific condition as a neurovascular assessment of the lower extremities. [B. Checking the electrocardiogram for ectopic beats] Determining duration of symptoms, assessing the electrocardiogram, and obtaining current vital signs are all interventions that the nurse should report to the HCP. However, they are not as high a priority or as relevant to the client's specific condition as a neurovascular assessment of the lower extremities. [C. Obtaining vital signs, including pulse oximetry] Determining duration of symptoms, assessing the electrocardiogram, and obtaining current vital signs are all interventions that the nurse should report to the HCP. However, they are not as high a priority or as relevant to the client's specific condition as a neurovascular assessment of the lower extremities. Educational objective:The nurse who suspects deep venous thrombosis should perform a thorough neurovascular assessment of the client's extremities. The assessment should include the presence and quality of dorsalis pedis and posterior tibial pulses, temperature of extremities, capillary refill, and circumference measurements of both calves and thighs. Both extremities should be assessed for comparison. The findings should be reported immediately to the health care provider.

A hospitalized client is receiving chemotherapy. Based on today's blood laboratory results, what action should the nurse take? Click on the exhibit button for additional information. Laboratory results White blood cell count1,400/mm3 (1.4x109/L) Absolute neutrophil count500/mm3 (0.5x109/L) Hemoglobin10.5 g/dL (105 g/L) Platelets150,000/mm3 (150x109/L) Serum potassium3.4 mEq/L (3.4 mmol/L) A. Check for hematuria B. Check for peaked T waves C. Obtain prescription for epoetin alfa D. Place a mask on the client

Correct Answer: D. Place a mask on the client The normal range for a white blood cell (WBC) count is 4,000-11,000/mm3 (4.0-11.0 × 109/L). Clients with neutropenia (a reduction in WBCs) are predisposed to infection. The absolute neutrophil count (ANC) is determined by multiplying the total WBC count by the percentage of neutrophils. Neutropenia is an ANC <1,000/mm3 (1.0 × 109/L). An ANC <500/mm3 (0.5 × 109/L) is defined as severe neutropenia and is a critical emergency. This client's neutropenia is probably a result of bone marrow suppression from the chemotherapy. The client needs reverse or protective isolation from microorganisms, on people or objects, to which the client lacks resistance. A hospitalized client needs to be in a private room, and the room may need to be equipped with HEPA (high-efficiency particulate air) filtration (or positive pressure airflow). Until the room can be readied, this client should be protected with a mask and separated from infectious clients. Additional neutropenic precautions include avoiding raw fruits/vegetables, standing water, and undercooked meat. In addition, no infectious health care providers (eg, with colds) should care for the client. Incorrect Answers: [A. Check for hematuria] Thrombocytopenia (low platelets) can result from bone marrow suppression caused by chemotherapy. This client's platelets are at the low end of the normal range (150,000-400,000/mm3 [150-400 × 109/L]). Spontaneous or surgical bleeding from thrombocytopenia rarely occurs with a platelet count of >50,000/mm3 (50 × 109/L). [B. Check for peaked T waves] This client's potassium level is slightly low (normal 3.5-5.0 mEq/L [3.5-5.0 mmol/L]). Low potassium, if it affects the cardiac tracing, causes flattened T waves. Peaked or tented T waves on a cardiac tracing are related to hyperkalemia. [C. Obtain prescription for epoetin alfa] Epoetin alfa (human recombinant erythropoietin) is a hematopoietic growth factor. The erythropoietin is produced in the kidney and stimulates bone marrow production of red blood cells (RBCs), a process called erythropoiesis. Epoetin alfa is used to stimulate RBC production but is not typically prescribed unless the client has symptomatic anemia with hemoglobin of <10 g/dL (100 g/L). Educational objective:Neutropenia (absolute neutrophil count [ANC] <1,000/mm3 [1.0 × 109/L]) and severe neutropenia (ANC <500/mm3 [0.5 × 109/L]) result in immunosuppression and require protective (reverse) precautions to be taken first. Place the client in a private room with HEPA filtration. Ensure that the client avoids raw fruits/vegetables, standing water, and undercooked meat.

The nurse assesses a client with hyperparathyroidism who had a parathyroidectomy 4 hours ago. Which assessment technique should the nurse use to check for complications in this client? A. Ask client to place backs of the hands against each other to provide hyperflexion of the wrist while the elbows remain flexed B. Instruct client to lie down and run the heel of one foot down the shin of the other leg C. Perform Romberg test by asking the client to stand with eyes closed and feet together D. Place blood pressure (BP) cuff on arm, inflate to pressure > than systolic BP, and hold in place for 3 minutes

Correct Answer: D. Place blood pressure (BP) cuff on arm, inflate to pressure > than systolic BP, and hold in place for 3 minutes Normal serum calcium is 8.6-10.2 mg /dL (2.15-2.55 mmol/L). Hypocalcemia (serum calcium <8.6 mg/dL [2.15 mmol/L]) is a potential complication of parathyroidectomy because the parathyroids regulate calcium levels in the blood. When one or more parathyroids are removed, it may take some time for others that have been dormant during hyperparathyroidism (which causes an increase in serum calcium) to begin regulating serum calcium. Trousseau's sign may indicate hypocalcemia before other signs and symptoms of hypocalcemia, such as tetany, occur. Trousseau's sign can be elicited by placing the BP cuff on the arm, inflating to a pressure > than systolic BP, and holding in place for 3 minutes. This will occlude the brachial artery and induce a spasm of the muscles of the hand and forearm when hypocalcemia is present. Chevostek's sign another early indicator of hypocalcemia, should also be assessed. It may be elicited by tapping the face at the angle of the jaw and observing for contraction on the same side of the face. Incorrect Answers: [A. Ask client to place backs of the hands against each other to provide hyperflexion of the wrist while the elbows remain flexed] Phalen's maneuver is used to diagnose carpal tunnel syndrome. [B. Instruct client to lie down and run the heel of one foot down the shin of the other leg] The heel-to-shin test is another means of assessing cerebellar function. An abnormal examination is evident when the client is unable to keep the foot on the shin. [C. Perform Romberg test by asking the client to stand with eyes closed and feet together] The Romberg test is a component of a neurological examination to assess vestibular function (perception of head position in space), proprioception (perception of the body in space), and vision. Educational objective:Normal serum calcium is 8.6-10.2 mg/dL (2.15-2.55 mmol/L). Hypocalcemia (serum calcium <8.6 mg/dL [2.15 mmol/L]) is a potential complication of parathyroidectomy. The nurse should check for Trousseau's and Chevostek's signs as early indications of hypocalcemia.

The nurse collaborates with the registered nurse to care for a client who developed diabetes insipidus 48 hours after a trans-sphenoidal hypophysectomy. Which prescription should the nurse question? A. Administer desmopressin B. Check urine osmolarity daily C. Obtain blood specimen for serum sodium D. Place the client in Trendelenburg position

Correct Answer: D. Place the client in Trendelenburg position Diabetes insipidus (DI) occurs either when the pituitary gland cannot secrete enough antidiuretic hormone (ADH) (ie, neurogenic DI) or when the kidneys do not respond to ADH (ie, nephrogenic DI). Neurogenic DI typically occurs after injury to or removal of the pituitary gland. Clinical manifestations include polydipsia (increased thirst), profound polyuria (increased urine output), and low urine osmolality and specific gravity (dilute urine). Clients with DI can become seriously dehydrated, so oral or IV rehydration must be given to replace lost fluids. Clients with neurogenic DI require lifelong supplementation of ADH with desmopressin, an ADH analog. The nurse should monitor urine output, urine specific gravity and osmolality, and serum sodium of clients treated for acute DI. With treatment, the client's urine specific gravity and osmolality should increase as serum sodium gradually normalizes. Dilutional hyponatremia may occur from fluid retention after the client begins receiving desmopressin. The client recovering from neurosurgery (eg, trans-sphenoidal hypophysectomy) should be positioned with the head of the bed elevated 15-30 degrees (eg, semi-Fowler, reverse Trendelenburg). This prevents cerebrovascular congestion and promotes clearance of sinus/incisional drainage. Educational objective:Diabetes insipidus (DI) can occur when the pituitary gland is injured or removed and not enough antidiuretic hormone is secreted. DI causes polydipsia and profound polyuria with dilute urine. Clients with DI following a hypophysectomy should receive desmopressin and be positioned with the head of the bed at 15-30 degrees.

The home health nurse visits a 72-year-old client with pneumonia who was discharged from the hospital 3 days ago. The client has less of a productive cough at night but now reports sharp chest pain with inspiration. Which finding is most important for the nurse to report to the supervising registered nurse? A. Bronchial breath sounds B. Increased tactile fremitus C. Low-pitched wheezing (rhonchi) D. Pleural friction rub

Correct Answer: D. Pleural friction rub Pleurisy is characterized by stabbing chest pain that usually increases on inspiration or with cough. It is caused by inflammation of the visceral pleura (over the lung) and the parietal pleura (over the chest cavity). The pleural space (between the 2 layers) normally contains about 10 mL of fluid to help the layers glide easily with respiration. When inflamed, they rub together, causing pleuritic pain. A pleural friction rub is auscultated in the lateral lung fields over the area of inflammation. The sound is produced by the 2 layers rubbing together and can indicate pleurisy, a complication of pneumonia. It is characterized by squeaking, crackling, or the sound heard when the palm is placed over the ear and the back of the hand is rubbed with the fingers. Complications of pneumonia are more prevalent in elderly clients with underlying chronic disease. Incorrect Answers: [A. Bronchial breath sounds] Clients with consolidative lung processes (pneumonia) may also have bronchial breath sounds due to over-transmission of sound over the chest wall. Breath sounds are diminished or absent over a pleural effusion or pneumothorax. [B. Increased tactile fremitus ] Palpable vibration felt on the chest wall is known as fremitus. Sound travels faster in solids (consolidation) than in an aerated lung, resulting in increased fremitus in pneumonia. It is an expected finding in clients with pneumonia. It is concerning if decreased because the client may be developing pleural effusion. [C. Low-pitched wheezing (rhonchi)] Low-pitched wheezing (rhonchi) is a continuous adventitious breath sound heard over the large airways, usually during expiration. It indicates the presence of secretions in the larger airways and is an expected finding as pneumonia resolves. Educational objective:Pleurisy is characterized by stabbing pleuritic chest pain that increases on inspiration. It is a complication of pneumonia caused by inflamed parietal and visceral pleurae rubbing together.

A client with an implantable cardioverter defibrillator (ICD) develops ventricular tachycardia (VT) with a pulse while admitted to the medical-surgical unit. The ICD fires multiple times without successfully stopping the VT, causing the client to become confused and difficult to rouse. Which action by the nurse is appropriate? A. Attempt to stimulate a vagal response by having the client cough B. Deactivate the client's implantable cardioverter defibrillator with an external magnet C. Obtain a STAT 12-lead ECG to verify the cardiac rhythm D. Prepare for synchronized cardioversion with the external defibrillator

Correct Answer: D. Prepare for synchronized cardioversion with the external defibrillator An implantable cardioverter defibrillator (ICD) is a medical device that is surgically implanted underneath the skin that can sense life-threatening arrhythmia and discharge electrical shocks directly into the cardiac muscle to correct the arrhythmia. Clients typically receive ICDs after a history of sustained or recurrent ventricular tachycardia (VT), including personal or family history of sudden cardiac death and severe heart failure. When caring for a client with an ICD, it is critical that the nurse monitor for ICD firings (eg, client report, observation on cardiac monitors). After firings, the nurse should monitor for resolution of the arrythmia, indications of hemodynamic compromise (eg, hypotension, chest pain, altered mentation), and additional ICD discharges. Occasionally, an ICD may be unable to convert the arrythmia to a hemodynamically stable rhythm and will repeatedly shock the client. If the client experiences repeated ICD shocks without dysrhythmia resolution, the nurse should promptly obtain a manual external defibrillator and initiate measures to prevent hemodynamic instability and cardiac arrest. Incorrect Answers: [A. Attempt to stimulate a vagal response by having the client cough] Stimulating a vagal response (eg, coughing) is an appropriate intervention for supraventricular tachycardia. [B. Deactivate the client's implantable cardioverter defibrillator with an external magnet] Deactivating the ICD and obtaining an ECG are not appropriate at this time because they delay lifesaving interventions. [C. Obtain a STAT 12-lead ECG to verify the cardiac rhythm] Deactivating the ICD and obtaining an ECG are not appropriate at this time because they delay lifesaving interventions. Educational objective:An implantable cardioverter defibrillator (ICD) is a device used to sense life-threatening arrhythmia and discharge electrical shocks to correct the arrhythmia. If a client experiences repeated ICD shocks without dysrhythmia resolution, the nurse should obtain a manual external defibrillator and initiate cardiac life support.

The practical nurse is collaborating with the registered nurse to form a care plan for a client with a possible diagnosis of Guillain-Barré syndrome. The nurse should give priority to which client assessment? A. Orthostatic blood pressure changes B. Presence or absence of knee reflexes C. Pupil size and reaction to light D. Rate and depth of respirations

Correct Answer: D. Rate and depth of respirations Guillain-Barré syndrome (GBS) is an acute, immune-mediated polyneuropathy that is most often accompanied by ascending muscle paralysis and absence of reflexes. Lower-extremity weakness progresses over hours to days to involve the thorax, arms, and cranial nerves (CNs). Neuromuscular respiratory failure is the most life-threatening complication; therefore, rate and depth of respirations should be monitored. Measurement of serial bedside forced vital capacity (spirometry) is the gold standard for assessing early ventilation failure. Incorrect Answers: [A. Orthostatic blood pressure changes] Autonomic dysfunction is common in GBS and usually results in orthostatic hypotension, paralytic ileus, urinary retention, and diaphoresis. These complications need to be assessed but are not a priority. [B. Presence or absence of knee reflexes] Absence of knee reflexes is expected early in the course of GBS due to the ascending nature of the disease. An absent gag reflex indicates GBS progression. [C. Pupil size and reaction to light] PERRLA (Pupils are Equal, Round, and Reactive to Light with Accommodation) evaluation assesses CNs II, III, IV, and VI. CN abnormalities are expected after involvement of thoracic (respiratory) muscles due to the ascending nature of GBS. Educational objective:The most serious complication to monitor for in new-onset Guillain-Barré syndrome is respiratory compromise from the paralysis ascending into the thoracic region. Monitoring the rate/depth of respirations and measuring serial bedside vital capacity (spirometry) help to detect this early in the disease course.

After a prolonged surgical procedure, the client reports unilateral leg pain. Which client assessment finding is most concerning? A. Client rates leg pain as "7" B. Negative Homan sign C. Prominent varicose veins bilaterally D. Right calf is 4 cm larger than left calf

Correct Answer: D. Right calf is 4 cm larger than left calf Deep venous thrombosis (DVT) is a major concern in clients with unilateral leg pain after prolonged immobilization (eg, air travel, surgery) or those with obesity, pregnancy, or other hypercoagulable states (eg, cancer). Eighty percent of DVTs start in the veins of the calf and move into the popliteal and femoral veins. Classic symptoms include unilateral leg edema, local warmth, erythema, and low-grade fever. Therefore, the swelling in one leg is highly concerning. Incorrect Answers: [A. Client rates leg pain as "7"] The level of pain is not as sensitive a sign as the edema, and severe pain is not a classic sign. Leg discomfort described as a sense of fullness, dullness, tightness, aching, or heaviness is more typical. [B. Negative Homan sign] Homan sign is pain in the calf with dorsiflexion. It is no longer recommended as an assessment tool for DVT as it is not very sensitive. However, a negative Homan sign would be a normal finding. [C. Prominent varicose veins bilaterally] Varicose veins can be a risk factor for DVT, but their presence alone is not as significant as the unilateral edema. Educational objective:Classic signs of deep venous thrombosis are unilateral leg swelling, local warmth, erythema, and low-grade fever in a client with obesity or immobility.

An elderly client with a history of stable chronic obstructive pulmonary disease, alcohol abuse, and cirrhosis has a serum theophylline level of 25.8 mcg/mL (143 µmol/L). Which clinical manifestation associated with theophylline toxicity should worry the nurse most? A. Alterations in color vision B. Gum (gingival) hypertrophy C. Hyperthermia D. Seizure activity

Correct Answer: D. Seizure activity Theophylline has narrow therapeutic index and plasma concentrations >20 mcg/mL (111 µmol/L) are associated with theophylline drug toxicity. Toxicity can be acute or chronic. Conditions associated with chronic toxicity include advanced age (>60), drug interactions (eg, alcohol, macrolide and quinolone antibiotics), and liver disease. Acute toxicity is associated with intentional or accidental overdose. Symptoms of toxicity usually manifest as central nervous system stimulation (eg, headache, insomnia, seizures), gastrointestinal disturbances (eg, nausea, vomiting), and cardiac toxicity (eg, arrhythmia). Incorrect Answers: [A. Alterations in color vision] Alteration in color perception and visual changes are commonly seen with digoxin toxicity. [B. Gum (gingival) hypertrophy] Gum hypertrophy is seen with phenytoin toxicity. [C. Hyperthermia] Hyperthermia and tinnitus are often seen with aspirin overdose. Educational objective:Theophylline plasma concentrations >20 mcg/mL (111 µmol/L) are associated with theophylline drug toxicity. Seizures (central nervous system stimulation) and cardiac arrhythmias are the most serious and lethal consequences.

The nurse is reviewing the telemetry strips of assigned clients. The rhythm strip displayed in the exhibit is given to the nurse by the telemetry technician. The nurse recognizes it as which rhythm? Click on the exhibit button for additional information. A. Atrial Fibrillation B. First-degree atrioventricular block C. Sinus bradycardia D. Sinus rhythm

Correct Answer: D. Sinus rhythm A standard electrocardiogram (ECG) traces the electrical activity of the heart at 25 mm/second on tracing paper (small boxes = 0.04 second, large boxes = 0.20 second). On a 6-second strip, the heart rate is determined by counting the number of QRS complexes on the strip and multiplying by 10. If the strip is not 6 seconds and the R-R interval is regular, the rate can be determined by counting the large boxes between QRS complexes and dividing that number into 300 (number of large boxes in 1 minute of strip). After determining the rate, the nurse should analyze the P waves, QRS complexes, and T waves (if present) and determine the rhythm. A regular heart rate of 60-100/min with normal PR intervals, QRS complexes, and QT intervals indicates that the client is in normal sinus rhythm Incorrect Answers: [A. Atrial Fibrillation] In atrial fibrillation, there are absent P waves and fine, fibrillatory waves, indicating disorganized atrial electrical activity. The ventricular rate is usually irregular. [B. First-degree atrioventricular block] In first-degree atrioventricular block, there is a prolonged (>20 seconds), regular PR interval. [C. Sinus bradycardia] Sinus bradycardia is characterized by a rate <60/min. Educational objective:To analyze electrocardiogram (ECG) strips, the nurse should measure the R-R interval to determine regularity and heart rate and then analyze the PR interval, QRS complex, and QT interval. A heart rate of 60-100/min and normal PR intervals, QRS complexes, and QT intervals indicate a normal sinus rhythm.

A client with chest pain is diagnosed with acute pericarditis by the health care provider. The nurse reinforces teaching to the client that the pain will improve with which of the following? A. Coughing and deep breathing B. Left lateral position C. Pursed-lip breathing D. Sitting up and leaning forward

Correct Answer: D. Sitting up and leaning forward The most common cause of acute pericarditis is a recent viral infection. It is an inflammation of the visceral and/or parietal pericardium. Pericarditis is characterized by sharp, pleuritic chest pain. It is aggravated during inspiration and coughing. Pain is typically relieved by sitting up and leaning forward. This position reduces pressure on the inflamed parietal pericardium, especially during lung inflation. The pain is different than that experienced during myocardial infarction. Assessment shows a pericardial friction rub (scratchy or squeaking sound). Treatment includes a combination of nonsteroidal anti-inflammatory drugs (NSAIDS) or aspirin plus colchicine. Incorrect Answers: [A. Coughing and deep breathing] Pericarditis causes pain on inspiration, not expiration. This pleuritic-type pain also increases with coughing. [B. Left lateral position] The supine or other lying-down position worsens pericarditis pain. [C. Pursed-lip breathing] The pursed-lip breathing technique helps to decrease shortness of breath by preventing airway collapse, promoting carbon dioxide elimination, and reducing air trapping in clients with chronic obstructive pulmonary disease. Educational objective:Pericarditis is characterized by sharp, pleuritic chest pain. It is aggravated during inspiration and coughing. Pain is typically relieved by sitting up and leaning forward. Treatment includes a combination of nonsteroidal anti-inflammatory drugs or aspirin plus colchicine.

The nurse is reinforcing education to a client with irritable bowel syndrome who is experiencing diarrhea. Which of these meals selected by the client indicates an understanding of diet management? A. Beans, yogurt, and a fruit cup B. Beef, broccoli, and a glass of wine C. Eggs, a bagel, and black coffee D. Steak, tomato basil soup, and cornbread

Correct Answer: D. Steak, tomato basil soup, and cornbread Irritable bowel syndrome (IBS) is a common, chronic bowel condition caused by altered intestinal motility. Peristaltic action is affected, causing diarrhea, constipation, or a combination of both. Management focuses on reducing diarrhea or constipation, abdominal pain, and stress. Clients can manage symptoms with diet, medications, exercise, and stress management. To manage IBS, clients should restrict gas-producing foods (eg, bananas, cabbage, onions); caffeine; alcohol; fermentable oligo-, di-, and monosaccharides and polyols (FODMAPs) (eg, honey, high-fructose corn syrup, wheat); and other gastrointestinal (GI) irritants (eg, spices, hot/cold food or drink, dairy products, fatty foods). Clients should gradually increase fiber intake (eg, whole grains, legumes, nuts, fruits, vegetables) as tolerated. Foods that are generally well tolerated include proteins, breads, and bland foods. Incorrect Answers: [A. Beans, yogurt, and a fruit cup] Although they are a great source of fiber, beans are gas-producing and should be avoided. Most dairy products are GI irritants; however, yogurt is often better tolerated and may be included in the diet. [B. Beef, broccoli, and a glass of wine] Gas-producing cruciferous vegetables (eg, broccoli, cabbage) should be avoided. Alcohol exacerbates IBS symptoms. [C. Eggs, a bagel, and black coffee] Hot beverages and caffeine (eg, coffee) irritate the GI tract. Bagels are gas-producing. Educational objective:Irritable bowel syndrome is a chronic condition characterized by altered intestinal motility, causing abdominal discomfort with diarrhea and/or constipation. Clients can manage symptoms by avoiding gas-producing foods (eg, broccoli), caffeine, alcohol, and gastrointestinal irritants (eg, high-fructose corn syrup, spices, dairy products) and by increasing fiber.

The client was diagnosed 6 months ago with hypertension and had a recent emergency department visit for a transient ischemic attack. The client's blood pressure today is 170/88 mm Hg. What teaching topic is a priority for the nurse to reinforce with this client? A. Decreasing sodium intake B. Decreasing stress levels at work and home C. Increasing activity level D. Taking blood pressure mediations as prescribed

Correct Answer: D. Taking blood pressure mediations as prescribed The priority teaching topic for this client is taking blood pressure medications as prescribed. A major problem with long-term management of hypertension is poor adherence to the treatment plan. Blood pressure medications can have unpleasant side effects, including fatigue, dizziness, and erectile dysfunction. Clients may stop taking the medications when they believe their blood pressure has returned to normal range or if medications are expensive. The nurse should determine whether the client has been taking the medications consistently. There may be a need for a dosage change or addition of another medication. This client's blood pressure is not well controlled, and the transient ischemic attack places the client at high risk for a stroke. Incorrect Answers: [A. Decreasing sodium intake] Decreasing sodium intake and stress levels, plus increasing activity level, are helpful in managing hypertension. The client should be doing all these, and the teaching topics need to be reinforced. However, they are a lower priority than taking blood pressure medications as prescribed. [B. Decreasing stress levels at work and home] Decreasing sodium intake and stress levels, plus increasing activity level, are helpful in managing hypertension. The client should be doing all these, and the teaching topics need to be reinforced. However, they are a lower priority than taking blood pressure medications as prescribed. [C. Increasing activity level] Decreasing sodium intake and stress levels, plus increasing activity level, are helpful in managing hypertension. The client should be doing all these, and the teaching topics need to be reinforced. However, they are a lower priority than taking blood pressure medications as prescribed. Educational objective:A major problem with long-term management of hypertension is poor adherence to the treatment plan. The nurse should teach the client the importance of taking blood pressure medications as prescribed.

The nurse is monitoring a client who had an esophagogastroduodenoscopy 2 hours ago. Which finding requires an immediate report to the registered nurse? A. Blood pressure drop from 122/88 mm Hg to 106/72 mm Hg B. Gag reflex has not returned C. Sore throat when swallowing D. Temperature spike to 101.2 F (38.4 C)

Correct Answer: D. Temperature spike to 101.2 F (38.4 C) A sudden temperature spike 1-2 hours after an esophagogastroduodenoscopy (EGD) could be a sign of perforation or a developing infection. The practical nurse should notify the registered nurse immediately. Incorrect Answers: [A. Blood pressure drop from 122/88 mm Hg to 106/72 mm Hg] This blood pressure drop could be due to several things (sedation, blood loss, sepsis), but without any other symptoms indicating an emergency condition, it is still within the normal range. [B. Gag reflex has not returned] The gag reflex may take a few hours to return as the EGD involves applying a topical anesthetic to the throat. Absent gag reflex after a prolonged period (6 hours) would require reporting to the health care provider. [C. Sore throat when swallowing] A sore throat is expected after certain procedures (EGD, intubation) due to local irritation. Warm saline gargles could provide some relief. Educational objective:Fever after an esophagogastroduodenoscopy (EGD) or colonoscopy could be a sign of infection from perforation and should be reported.

The nurse is caring for a client recently diagnosed with an active deep vein thrombosis (DVT). Which action by the client would require an immediate intervention by the nurse? A. The client has a temperature of 100 F (37.7 C) B. The client is ambulating up and down the hallways C. The client is breathing at a rate of 16/min D. The client is massaging the leg at the site of inflammation

Correct Answer: D. The client is massaging the leg at the site of inflammation Clients with active DVT are at risk for developing a pulmonary embolism (PE). In the case of active DVT, the clot may become dislodged by massage or use of sequential compression devices on the affected extremity. The nurse would intervene immediately if a client was observed massaging the site, as this may actually trigger an embolism. Incorrect Answers: [A. The client has a temperature of 100 F (37.7 C)] Mild fever would be expected with a diagnosis of DVT. [B. The client is ambulating up and down the hallways] The client should not be kept immobilized out of a fear of dislodging the clot. Further immobility creates venous stasis and risks clot formation. Early ambulation is not shown to increase the risk for PE as previously believed. Ambulation is strongly encouraged after a full medical evaluation finds no risk of impending embolization. Bedrest with limb elevation may be prescribed initially for clients with severe pain and edema. [C. The client is breathing at a rate of 16/min] This is a normal respiratory rate in an adult. A respiratory rate of more than 20/min and symptoms of dyspnea could indicate a PE. Educational objective:DVT is best prevented by avoiding extended bed rest and encouraging ambulation even after the diagnosis. Massaging the site of inflammation is highly discouraged.

The nurse reviews laboratory data for a client admitted to the emergency department with chest pain. Which serum value requires the most immediate action by the nurse? A. Glucose 200 mg/dL (11.1 mmol/L) B. Hematocrit 38% (0.38) C. Potassium 3.4 mEq/L (3.4 mmol/L) D. Troponin 0.7 ng/mL (0.7 mcg/L)

Correct Answer: D. Troponin 0.7 ng/mL (0.7 mcg/L) Serum cardiac markers are proteins released into the bloodstream from necrotic heart tissue after a myocardial infarction (MI). Troponin is a highly specific cardiac marker for the detection of MI. It has greater sensitivity and specificity for myocardial injury than creatine kinase (CK) MB. Serum levels of troponin increase 4-6 hours after the onset of MI, peak at 10-24 hours, and return to baseline after 10-14 days. A troponin value of 0.7 ng/mL (0.7 mcg/L) indicates cardiac muscle damage and should be the priority and immediate focus of the nurse. Normal values: troponin I <0.5 ng/mL (<0.5 mcg/L); troponin T <0.1 ng/mL (<0.1 mcg/L). Incorrect Answers: [A. Glucose 200 mg/dL (11.1 mmol/L)] The glucose is elevated (normal random glucose 70-110 mg/dL [3.9-6.1 mmol/L]) but is not the priority in this situation. The nurse will need to assess whether the client has a history of diabetes and time of the last meal. [B. Hematocrit 38% (0.38)] Normal hematocrit for a male is 39%-50% [0.39-0.50] and 35%-47% [0.35-0.47] for a female. The hematocrit value is not the priority. [C. Potassium 3.4 mEq/L (3.4 mmol/L)] The potassium is slightly below normal (3.5-5.0 mEq/L [3.5-5.0 mmol/L]). This should be the nurse's second priority. A low potassium level can precipitate dysrhythmias. Educational objective:An elevated troponin value holds the highest priority for intervention when a client is experiencing chest pain. Positive troponin levels are indicative of myocardial injury and require immediate attention by the nurse. Normal values are <0.5 ng/mL (<0.5 mcg/L) for troponin I and <0.1 ng/mL (<0.1 mcg/L) for troponin T.

The nurse is caring for a client with left-sided weakness from a stroke. When assisting the client to a chair, what should the nurse do? A. Bend at the waist B. Keep the feet close together C. Pivot on the foot proximal to the chair D. Use a transfer belt

Correct Answer: D. Use a transfer belt When transferring a client from bed to chair the following are recommended for client safety: 1. Clients should wear nonskid shoes (first step) 2. Make sure the bed and chair (wheelchair) brakes are locked 3. Use a transfer belt. A transfer belt worn around the client's waist allows the nurse to assist the client while maintaining proper body mechanics and safety. 4. Transfer the client toward the stronger (not the weaker) side. If the client is weak on the left side, ask the client to pivot on the right side. Incorrect Answers: [A. Bend at the waist] Bending at the waist often requires the nurse to use the back for lifting, making for poor body mechanics. [B. Keep the feet close together] The nurse should provide a wide body stance for more stability. Keeping the feet close together would not be good body mechanics and could cause injury. [C. Pivot on the foot proximal to the chair] The nurse using proper body mechanics would pivot on the foot distal to the chair. Educational objective:A transfer belt worn around the client's waist allows the nurse to assist the client while maintaining proper body mechanics and safety.

The nurse prepares to instill dialysate for a client receiving peritoneal dialysis. Which nursing action is the priority? A. Ensuring that the drainage collection bag is below the level of the abdomen B. Placing the client in the semi-Fowler position C. Recording the characteristics of output dialysate D. Using sterile technique when spiking and attaching the bag of dialysate

Correct Answer: D. Using sterile technique when spiking and attaching the bag of dialysate In peritoneal dialysis (PD), the abdominal lining (peritoneum) is used as a semipermeable membrane to dialyze clients with decreased kidney function. A catheter is placed in the peritoneal cavity for infusing and draining dialysate (dialysis fluid). Dialysate is infused and dwells in the abdomen, which allows waste products and electrolytes to cross the peritoneum into the dialysate for removal. After the prescribed dwell time, the dialysate, electrolytes, and wastes are drained via gravity. When administering PD, it is essential to use sterile technique when spiking and attaching bags of dialysate to the client's PD catheter to prevent contamination and infection. Bacterial peritonitis, an infection of the peritoneum, is a potential complication of PD that may lead to sepsis. Signs of peritonitis should be reported to the health care provider. Incorrect Answers: [A. Ensuring that the drainage collection bag is below the level of the abdomen] Proper positioning of the catheter drainage bag (ie, below the abdomen) and the client (eg, Fowler or semi-Fowler position) promotes effluent outflow but is not a priority over infection prevention. [B. Placing the client in the semi-Fowler position] Proper positioning of the catheter drainage bag (ie, below the abdomen) and the client (eg, Fowler or semi-Fowler position) promotes effluent outflow but is not a priority over infection prevention. [C. Recording the characteristics of output dialysate] Cloudy effluent may indicate infection, whereas bloody or brown effluent may indicate bowel perforation. Documenting effluent characteristics is important but not a priority over maintaining asepsis. Educational objective:Peritoneal dialysis (PD) uses the peritoneum as a semipermeable membrane to dialyze clients with decreased kidney function. Bacterial peritonitis is a potential complication of PD. Using sterile technique when spiking or changing bags of dialysate is a priority to avoid contamination and reduce the risk of peritonitis.

Which of the following diets would place a client at the highest risk for macrocytic anemia? A. Lacto-ovo-vegetarian B. Lacto-vegetarian C. Macrobiotic D. Vegan

Correct Answer: D. Vegan Megaloblastic anemia is caused by vitamin B12 or folic acid deficiency. Vitamin B12 deficiency can also result in peripheral neuropathy and cognitive impairment. Vitamin B12 is formed by microorganisms and found only in animal foods; some plant foods may contain minimal amounts of vitamin B12 only if they accidentally contain animal particles. Natural sources of vitamin B12 include meat, fish, poultry, eggs, and milk; some breads and cereals may be fortified with vitamin B12 as well as some nutritional yeasts. Vegans are strict vegetarians; they exclude all animal products, including eggs, milk, and milk products, from the diet. They also may avoid foods that are processed or not organically grown, thereby eliminating potentially fortified food sources of vitamin B12. Individuals who practice any form of vegetarianism are at risk for vitamin B12 deficiency. A vegan diet, with its elimination of all animal products, poses the highest risk. A vitamin B12 supplement is recommended when dietary intake is inadequate. Incorrect Answers: [A. Lacto-ovo-vegetarian] Lacto-ovo-vegetarian — eggs, milk, and milk products are included, but no meat is consumed. [B. Lacto-vegetarian] Lacto-vegetarian — milk and milk products are included in the diet; eggs and meats are excluded. [C. Macrobiotic] Macrobiotic — whole grains, vegetables, fruits, and seaweeds are emphasized; fish and seafood may be included in the diet up to several times a week. Educational objective:Individuals who follow a plant-based diet, especially vegans, are at risk for vitamin B12 deficiency and the resulting macrocytic anemia.

The nurse reinforces teaching on preventive measures against hepatitis A transmission to a group of clients who are homeless. Which measure would the nurse state is the priority precaution to prevent transmission? A. Do not share needles when injecting drugs B. Practice safe sex by using condoms C. Receive the hepatitis A vaccine D. Wash hands after bowel movements and before eating

Correct Answer: D. Wash hands after bowel movements and before eating Transmission of hepatitis A occurs most commonly via the fecal-oral route and is due to poor hand hygiene and improper food handling by infected persons. It is seen primarily in developing countries. After infection, the hepatitis A virus reproduces in the liver and is secreted in bile. Therefore, hand hygiene (especially after toileting and before meals) is the most important intervention to reduce incidence of hepatitis A. Vaccination against hepatitis A is recommended for all children at age 1 and for adults at risk of contracting the virus (health care workers, men who have sex with men, IV drug users, persons who travel to high prevalence areas, and those with clotting disorders or liver disease). Incorrect Answers: [A. Do not share needles when injecting drugs] Hepatitis A virus is secreted in bile and generally transmitted via the fecal-oral route. However, the virus can also spread via unsafe sexual practices and needle sharing between IV drug users. Such behaviors should be discouraged and hand hygiene encouraged as the most important intervention for prevention. [B. Practice safe sex by using condoms] Hepatitis A virus is secreted in bile and generally transmitted via the fecal-oral route. However, the virus can also spread via unsafe sexual practices and needle sharing between IV drug users. Such behaviors should be discouraged and hand hygiene encouraged as the most important intervention for prevention. [C. Receive the hepatitis A vaccine] Vaccination is important for preventing infection. However, hygienic measures (eg, hand washing, sanitation, cleanliness, avoiding sharing personal items) should be readily implemented by all clients regardless of means. Educational objective:Hepatitis A virus is transmitted via the fecal-oral route. Therefore, hygienic practices (eg, hand hygiene, sanitation) are the fastest and most readily available interventions to prevent spread of the virus. Needle sharing and unprotected sex should be discouraged, and all children at least 1 year old should receive the hepatitis A vaccine.

The nurse cares for a client diagnosed with Addison disease. Which clinical finding would the nurse anticipate? A. Acanthosis nigricans B. Hirsutism C. Truncal obesity D. Weight loss

Correct Answer: D. Weight loss Addison disease (chronic adrenal insufficiency) occurs when the adrenal glands produce inadequate amounts of steroid hormones (eg, mineralocorticoids, glucocorticoids, androgens). It is characterized by: · Hypoglycemia, weight loss, and muscle weakness: A deficit of glucocorticoids (eg, cortisol) decreases the amount of liver glycogen and prevents gluconeogenesis. · Hyperpigmented skin, especially on the knees, elbows, buccal area, and palmar creases: Low cortisol triggers the production of excess ACTH from the pituitary gland. · Hypotension: A deficit of aldosterone prevents the retention of water and sodium. · Hyperkalemia: Potassium is retained when sodium is lost in urine. Treatment for Addison disease consists of replacement therapy with oral mineralocorticoids and corticosteroids. Incorrect Answers: [A. Acanthosis nigricans] Acanthosis nigricans, a skin condition related to insulin resistance from obesity and diabetes, appears as velvet-like patches of darkened, thick skin, typically on the back of the neck and in the groin and armpits. [B. Hirsutism] Cushing syndrome (overproduction of steroid hormones) produces "opposite" symptoms from Addison disease. Clients have truncal obesity, "buffalo hump" (fat deposits in the shoulders), hyperglycemia from excess glucocorticoids, and hypertension from excess aldosterone. Increased sex hormones cause hirsutism (male-pattern hair growth in females). Immune suppression from increased glucocorticoids increases the risk of infection and delays healing. [C. Truncal obesity] Cushing syndrome (overproduction of steroid hormones) produces "opposite" symptoms from Addison disease. Clients have truncal obesity, "buffalo hump" (fat deposits in the shoulders), hyperglycemia from excess glucocorticoids, and hypertension from excess aldosterone. Increased sex hormones cause hirsutism (male-pattern hair growth in females). Immune suppression from increased glucocorticoids increases the risk of infection and delays healing. Educational objective:Addison disease occurs when the adrenal glands produce inadequate amounts of steroid hormones (eg, mineralocorticoids, glucocorticoids). It is characterized by hyperpigmented skin, hypoglycemia, weight loss, muscle weakness, hypotension, and hyperkalemia.

A client with type 2 diabetes, coronary artery disease, and peripheral arterial disease developed hospital-acquired pneumonia (HAP) and has been receiving intravenous (IV) antibiotics for 4 days. Which parameter monitored by the nurse best indicates the effectiveness of treatment? A. Color of sputum B. Lung sounds C. Saturation level D. White blood cell count (WBC)

Correct Answer: D. White blood cell count (WBC) HAP is a bacterial infection acquired in a health care facility that was not present on admission. Almost all clients with bacterial pneumonia develop leukocytosis (WBC >11,000/mm3). Antibiotic therapy is the first-line treatment, but antibiotic resistance frequently occurs in HAP. If antibiotic therapy is effective, clinical improvement usually occurs within 3-4 days of initiation of IV antibiotics. The nurse monitors WBC as the best indicator of treatment effectiveness as antibiotics cause bacterial lysis or hinder bacterial DNA reproduction. The reduced number of bacteria and the resulting decrease in inflammation cause a decrease in the number of white blood cells needed to fight the infection. Other indicators of treatment effectiveness include improvement of infiltrates on chest x-ray, oxygenation, and signs and symptoms (cough, fever, sputum production). Incorrect Answers: [A. Color of sputum] The color of sputum (eg, clear, yellow, green, grey, rusty, blood-tinged) can vary with different types of pneumonia; it is not the best indicator of treatment effectiveness. [B. Lung sounds] Adventitious/abnormal lung sounds (crackles, low-pitched wheeze, bronchial breath sounds) can be present as the pneumonia resolves or can be a sign of further complication (pleural effusion). However, these are not the best indicators of treatment effectiveness. [C. Saturation level] Saturation is an indicator of oxygenation but can be affected by many other factors, such as coexisting disease, peripheral circulation, and drugs. It is not the best indicator of treatment effectiveness. Educational objective:Indicators of treatment effectiveness for HAP include decreased WBC on complete blood count with differential and improvement of infiltrates on chest-x-ray, oxygenation, and signs and symptoms (cough, fever, sputum production).

The nurse is caring for a client with Bell's palsy. The nurse most likely expects which finding(s) on assessment? Select all that apply. A. Change in tear production on the affected side B. Electric shock-like pain in the lips and gums C. Flaccidity of forehead muscles D. Inability to smile symmetrically E. Severe facial pain along the cheekbone

Correct Answers: A, C, and D Bell's palsy is peripheral, unilateral facial paralysis characterized by inflammation of the facial nerve (cranial nerve VII) in the absence of a stroke or other causative agent/disease. Paralysis of the motor fibers innervating the facial muscles results in flaccidity on the affected side. Symptoms include the following: — Inability to completely close the eye on the affected side — Flattening of the nasolabial fold on the side of the paralysis — Inability to smile or frown symmetrically — Alteration in tear production (eg, decreased tearing with extreme dryness, excessive tearing) due to lower eyelid muscle weakness Alteration in the sensory fibers can cause loss of taste on the anterior two-thirds of the tongue. Incorrect Answers [B. Electric shock-like pain in the lips and gums] Severe facial pain along the cheekbone and electric shock-like pain in the lips and gums are symptoms of trigeminal neuralgia (cranial nerve V). The trigeminal nerve may become hypersensitive and cause facial pain in Bell's palsy, but this is uncommon. [E. Severe facial pain along the cheekbone] Severe facial pain along the cheekbone and electric shock-like pain in the lips and gums are symptoms of trigeminal neuralgia (cranial nerve V). The trigeminal nerve may become hypersensitive and cause facial pain in Bell's palsy, but this is uncommon. Educational objective:Bell's palsy is a facial paralysis characterized by facial droop, unsymmetrical smile or frown, changes in tear production, and inability to close the affected eye appropriately. Facial pain with Bell's palsy is related to hypersensitivity of the trigeminal nerve and is uncommon.

The emergency department nurse is caring for a 70-year-old client with a history of type 2 diabetes mellitus who reports sudden-onset nausea, sweating, dizziness, and fatigue. The nurse should anticipate the initiation of which protocol? A. Food poisoning B. Influenza C. Myocardial infarction D. Stroke

Correct Answer: Myocardial infarction Early recognition and treatment of heart attack are critical. Women, the elderly, and clients with a history of diabetes may not have the classic heart attack symptoms of dull chest pain with radiation down the left arm. Instead, they can have "atypical" symptoms such as nausea, vomiting, belching, indigestion, diaphoresis, dizziness, and fatigue. Incorrect Answers: [A. Food poisoning] Taking a careful history and evaluating for any sick contacts would be helpful in identifying food poisoning, but a more important initial step is to assess for a heart attack. [B. Influenza] A viral infection is a possibility, but fever and myalgia are usually present during an episode of influenza. [D. Stroke] Early intervention in stroke is also critical, and a neurologic assessment would take place after the acute coronary syndrome algorithm, especially with negative ECG and serum heart enzyme levels. Educational objective:Myocardial infarctions in women, the elderly, and clients with diabetes may present with gastrointestinal distress as the main symptom; this should be evaluated with the institutional protocol for acute coronary syndrome.

The nurse evaluates the results of laboratory tests completed on a client admitted for a non-healing wound. Which of the following values would be a priority for the nurse? A. Blood urea nitrogen 15 mg/dL (5.4 mmol/L) B. Serum albumin 3.7 g/dL (37 g/L) C. Serum potassium 4.5 mEq/L (4.5 mmol/L) D. Serum sodium 153 mEq/L (153 mmol/L)

Correct Answer: Serum sodium 153 mEq/L (153 mmol/L) Nutritional deficiencies (eg, zinc, protein, vitamin C) and dehydration can impair wound healing. Dehydration (loss of free water) can increase serum sodium levels. The normal value for serum sodium is 135-145 mEq/L (135-145 mmol/L). The value listed, 153 mEq/L (153 mmol/L), is high. Increased serum sodium level (hypernatremia) has an osmotic action that causes water to be pulled from the interstitial spaces into the vascular system. Remember that "water goes where salt is." This action decreases wound healing at a cellular level, reducing the nutrients cells need for repair. Incorrect Answers: [A. Blood urea nitrogen 15 mg/dL (5.4 mmol/L)] Normal blood urea nitrogen (BUN) values are 6-20 mg/dL (2.1-7.1 mmol/L). Elevated BUN may indicate dehydration and could impair wound healing. [B. Serum albumin 3.7 g/dL (37 g/L)] Malnutrition can impair wound healing. Serum albumin and prealbumin levels are obtained to assess nutritional status. The normal value for albumin is 3.5-5.0 g/dL (35-50 g/L). [C. Serum potassium 4.5 mEq/L (4.5 mmol/L)] The normal value for serum potassium is 3.5-5.0 mEq/L (3.5-5.0 mmol/L) Educational objective:Nutritional deficiencies (eg, zinc, protein, vitamin C) and dehydration (hypernatremia, elevated BUN) can impair wound healing.

The nurse working in the intensive care unit hears an alarm coming from a client's room. On entering the room, the nurse sees the rhythm displayed in the exhibit on the monitor. The nurse recognizes it as which rhythm? A. Asystole B. Atrial fibrillation C. Ventricular fibrillation D. Ventricular tachycardia

Correct Answer: Ventricular fibrillation VF is characterized on the ECG by irregular waveforms of varying shapes and amplitudes. This represents the firing of multiple ectopic foci originating in the ventricle. Mechanically, the ventricle is quivering with no effective contraction or cardiac output. VF is considered a lethal dysrhythmia. It results in an unresponsive, pulseless, apneic state. If not treated rapidly, the client will not recover. VF commonly occurs in acute myocardial infarction and myocardial ischemia and in chronic heart diseases such as heart failure and cardiac myopathy. It may occur in cardiac pacing or catheterization procedures due to catheter stimulation of the ventricle. Treatment consists of rapid initiation of CPR, defibrillation, and the use of drug therapy (eg, epinephrine, vasopressin, amiodarone). Incorrect Answers [A. Asystole] Asystole is the total absence of ventricular electrical activity. [B. Atrial fibrillation] Atrial fibrillation is characterized by total disorganization of atrial, not ventricular, activity. QRS complexes are usually normal in morphology. P waves are not seen. [D. Ventricular tachycardia] Ventricular tachycardia has a ventricular rate of 150-250/min and originates from foci firing repeatedly in the ventricle. Educational objective:The nurse should recognize VF, a potentially lethal dysrhythmia. The ECG shows irregular waveforms of varying shapes and amplitudes. The client is unresponsive, pulseless, and apneic. Rapid treatment should include CPR, defibrillation, and drug therapy (eg, epinephrine, vasopressin, amiodarone).

An unlicensed assistive personnel (UAP) is aiding a client recovering from a right-sided cerebrovascular accident with resulting mild oropharyngeal dysphagia. The client has been placed on a dysphagia diet. Which actions require intervention by the nurse? Select all that apply. A. The UAP adds milk to mashed potatoes to make them thinner B. The UAP encourages the client to occasionally turn the head to the left C. The UAP helps the client sit in an upright position D. The UAP places food on the strong side of the client's mouth E. The UAP puts a straw in a fruit smoothie to prevent spilling

Correct Answers: A & E Clients with dysphagia are at risk for aspiration and aspiration pneumonia. Dietary modifications and swallowing rehabilitation measures can reduce the risk of aspiration in clients who can tolerate oral feedings. Specific techniques include the following: — Modification of food consistency (pureed, mechanically altered, soft) — Thickened liquids — Having the client sit upright at a 90-degree angle — Placing food on the stronger side of the mouth to aid in bolus formation — Tilting the neck slightly to assist with laryngeal elevation and closure of the epiglottis Some clients who have suffered a cerebrovascular accident (CVA) are also left with visual impairment such as hemianopsia; in this condition, a person sees only a portion of the visual field from each eye. A client with a right-sided CVA may have left-sided hemianopsia. Having the client turn the head during a meal will help the client see everything on the plate. [A. The UAP adds milk to mashed potatoes to make them thinner] Adding milk to mashed potatoes will alter the consistency; if the consistency is too thin, the client will be at increased risk of aspiration. [E. The UAP puts a straw in a fruit smoothie to prevent spilling] Using a straw for drinking liquids might cause increased swallowing difficulty and choking. Controlling liquid intake through a straw is more difficult than drinking straight from a cup or glass. Educational objective:Measures for reducing the risk of aspiration for clients with dysphagia include diet modification (pureed, mechanically altered, soft), thickened liquids, positioning the client in an upright position, placing food on the stronger side of the mouth, and flexing the chin slightly downward. A client with visual impairment should be reminded to turn the head from time to time while eating.

The nurse is reinforcing education about home and lifestyle alterations to a client recently diagnosed with HIV. Which of the following statements by the client indicates a need for further education? Select all that apply. A. "I don't have to use protection if my sexual partner is also HIV positive." B. "I have to make sure my family knows not to borrow my razors." C. "I need to avoid eating raw or undercooked meats and eggs." D. "I started to use lambskin condoms during sex, as I have a latex allergy." E. "I won't reuse or share any needles or syringes that I use to inject heroin."

Correct Answers: A and D. Human immunodeficiency virus (HIV) is a viral infection of the CD4+ (helper T) cells, resulting in progressive immune system impairment. When educating clients with HIV, the nurse should discuss health promotion and infection transmission prevention strategies, particularly safe sex practices. Unprotected sex increases the risk of transmitting HIV and other sexually transmitted infections (STIs). Protected sex is important even with HIV-positive partners as HIV has multiple strains and coinfection results in HIV superinfection, which may hasten progression to AIDS Clients with HIV should use latex or synthetic condoms and/or dental dams during sexual activity involving mucous membrane exposure (ie, oral, vaginal, anal) to semen or vaginal secretions. Natural barriers (eg, lambskin) do not prevent transmission of STIs due to the presence of small pores. Incorrect Answers: [B. "I have to make sure my family knows not to borrow my razors."] Sharing personal hygiene devices that may have been exposed to blood (eg, toothbrushes, razors) increases HIV transmission risk and should be avoided. [C. "I need to avoid eating raw or undercooked meats and eggs."] Immunosuppressed clients should be educated to avoid raw or undercooked foods (eg, eggs, meats, seafood) to avoid foodborne illnesses. [E. "I won't reuse or share any needles or syringes that I use to inject heroin."] To prevent transmission of HIV, hepatitis B virus, and other bloodborne diseases, IV drug users should be taught to avoid reusing or sharing needles or syringes. Educational objective:Clients with HIV are educated to use latex or synthetic barriers during all sexual encounters (ie, oral, vaginal, anal) in which nonintact skin or mucous membranes are exposed to semen or vaginal secretions. Unprotected sex increases the risk of transmitting HIV and other sexually transmitted infections, as well as HIV coinfection/superinfection.

The practical nurse is reviewing serum laboratory values for an 80-year-old client with an IV vancomycin prescription for treatment of methicillin-resistant Staphylococcus aureus infection. Which laboratory results should be reported to the registered nurse before the client receives the dose? Select all that apply. A. Blood urea nitrogen is 60 mg/dL (21.4 mmol/L) B. Creatinine is 2.1 mg/dL (185.6 µmol/L) C. Glucose is 140 mg/dL (7.7 mmol/L) D. Hemoglobin is 15 g/dL (150 g/L) E. Magnesium is 1.5 mEq/L (0.75 mmol/L) F. White blood cell count is 14,000/mm3 (14.0 × 109/L)

Correct Answers: A, B Vancomycin is an antibiotic excreted by the kidneys that is used to treat serious gram-positive bacterial infections (eg, methicillin-resistant Staphylococcus aureus) and the diarrhea associated with Clostridium difficile infection. Blood urea nitrogen (BUN) and creatinine levels are monitored regularly (usually 2-3 times/week) in clients receiving vancomycin due to increased risk of nephrotoxicity, especially in those age >60, with impaired renal function, and who are receiving other nephrotoxic medications (eg, aminoglycoside antibiotics). The nurse should be familiar with the client's baseline levels of BUN and creatinine to monitor trends (mainly an increase in serum levels). The health care provider can decrease the drug dose and administration frequency or discontinue vancomycin if needed. Serum vancomycin trough level is monitored before the fourth dose (optimal levels: 15-20 mg/L [10.4-13.8 µmol/L]). The normal range for BUN is 6-20 mg/dL (2.1-7.1 mmol/L) and for creatinine is 0.6-1.3 mg/dL (53-115 µmol/L). The nurse should notify the registered nurse (RN) that the client's BUN (60 mg/dL [21.4 mmol/L]) and creatinine (2.1 mg/dL [185.6 µmol/L]) are both increased. Incorrect Answers [C. Glucose is 140 mg/dL (7.7 mmol/L)] An elevated glucose level of 140 mg/dL (7.7 mmol/L) is expected in a client with an infection due to physiological stress and gluconeogenesis; this does not need to be reported to the RN. [D. Hemoglobin is 15 g/dL (150 g/L)] A hemoglobin level of 15 g/dL (150 g/L) is normal (13.2-17.3 g/dL [132-173 g/L] in adult men; 11.7-15.5 g/dL [117-155 g/L] in adult women) and does not need to be reported to the RN. [E. Magnesium is 1.5 mEq/L (0.75 mmol/L)] A magnesium level of 1.5 mEq/L (0.75 mmol/L) is normal (1.5-2.5 mEq/L [0.75-1.25 mmol/L]) and does not need to be reported to the RN. [F. White blood cell count is 14,000/mm3 (14.0 × 109/L)] A white blood cell count of 14,000/mm3 (14 × 109/L) is elevated and expected in a client with a serious infection; this does not need to be reported to the RN. Educational objective:Due to the possibility of nephrotoxicity, monitoring of vancomycin trough level to maintain optimal drug level and blood urea nitrogen and creatinine to assess renal function is indicated, especially in clients with impaired renal function and who are age >60.

The nurse identifies which of the following clients as being at high risk for developing colorectal cancer? Select all that apply. A. Client who consumes a diet high in red meat and low in fiber B. Client who is morbidly obese C. Client with a 15-year history of ulcerative colitis D. Client with a 40-year history of cigarette smoking E. Client with a family history of colorectal cancer

Correct Answers: A, B, C, D, E Colorectal cancer is the third most common cancer and the second leading cause of cancer deaths; it affects both genders equally. Various risk factors for colorectal cancer include: — Personal or family (first-degree relative) history of colorectal cancer or polyps — Personal history of inflammatory bowel disease, Crohn disease, or ulcerative colitis —History of hereditary nonpolyposis colorectal cancer (Lynch syndrome) — Lifestyle factors such as a diet high in red meat and saturated fat and low in fiber, obesity (eg, body mass index >30 kg/m2), cigarette smoking, and alcohol consumption Educational objective:Medical risk factors for colorectal cancer include a personal or family (first-degree relative) history of colorectal cancer or polyps and personal history of inflammatory bowel disease. Lifestyle risk factors include a diet high in red meat and saturated fat and low in fiber, obesity, cigarette smoking, and alcohol consumption.

An older client comes to the outpatient clinic for a routine physical examination and health screening. Which findings does the nurse recognize as possible indications of colorectal cancer? Select all that apply. A. Abdominal pain B. Blood in the stools C. Change in bowel habits D. Low hemoglobin level E. Unexplained weight loss

Correct Answers: A, B, C, D, and E Colorectal cancer occurs most often in adults over age 50. Risk factors include history of colon polyps; family history of colorectal cancer; inflammatory bowel disease (eg, Crohn disease, ulcerative colitis); and history of other cancers (eg, gastric, ovarian). Symptoms of colorectal cancer may include: · Blood in the stool (eg, positive occult blood, melena) from fragile, bleeding polyps or tumors · Abdominal discomfort and/or mass (not common) · Anemia due to intestinal bleeding, which may result in fatigue and dyspnea with exertion · Change in bowel habits (eg, diarrhea, constipation) due to obstruction by polyps or tumors · Unexplained weight loss due to impaired nutrition from altered intestinal absorption Colorectal cancer often goes unnoticed, as many of the symptoms are painless and nonspecific. Clients should be assessed for these symptoms and receive regular routine colorectal cancer screening tests (eg, occult blood test every year, colonoscopy every 10 years). Educational objective:Clients over age 50 should receive routine colorectal cancer screening for symptoms such as blood in the stool, anemia, abdominal discomfort, change in bowel habits, and weight loss. Symptoms result from intestinal polyps or tumors that cause intestinal bleeding, obstruction, and impaired intestinal absorption.

A client with chronic bronchitis tells the home health nurse of being exhausted all day due to coughing all night and being unable to sleep. The client can feel thick mucus in the chest and throat. Which teaching can the nurse reinforce to help the client mobilize secretions and improve sleep? Select all that apply. A. Increase fluids to at least 8 glasses (2-3 L) of water a day B. Sleep with a cool mist humidifier C. Take prescribed guaifenesin cough medicine before bedtime D. Use abdominal breathing and the huff cough technique at bedtime E. Use pursed-lip breathing during the night

Correct Answers: A, B, C, and D Chronic bronchitis is characterized by excessive mucus production, chronic cough, and recurrent respiratory tract infections. Interventions to help reduce viscosity of mucus, facilitate secretion removal, and promote comfort include the following: — Increasing oral fluids to 2-3 L/day if not contraindicated prevents dehydration and keeps secretions thin — Cool mist humidifier increases room humidity of inspired air — Guaifenesin (Robitussin) is an expectorant that reduces the viscosity of thick secretions by increasing respiratory tract fluid; drinking a full glass of water after taking the medication is recommended. — Abdominal breathing with the huff, a forced expiratory cough technique, is effective in mobilizing secretions into the large airways so that they can be expectorated — Chest physiotherapy (postural drainage, percussion, vibration) — Airway clearance handheld devices, which use the principle of positive expiratory pressure to help loosen secretions when the client exhales through the mouthpiece Incorrect Answers: [E. Use pursed-lip breathing during the night] Pursed lip breathing prolongs exhalation, reduces air trapping in the lungs, and decreases dyspnea. It does not help to thin secretions. Educational objective:Interventions to help reduce viscosity of mucus, facilitate secretion removal, and promote comfort in clients with chronic bronchitis include increasing oral fluids to 2-3 L/day if not contraindicated, cool mist humidification, cough expectorants (eg, guaifenesin), and huff coughing.

Which instructions should the nurse include when reinforcing discharge teaching to a client with peptic ulcer disease due to Helicobacter pylori infection? Select all that apply. A. "Avoid foods that may cause epigastric distress such as spicy or acidic foods." B. "It is best if you refrain from consuming alcohol products." C. "Report black tarry stools to your health care provider immediately." D. "Take your amoxicillin, clarithromycin, and omeprazole for the next 14 days." E. "You may take over-the-counter drugs such as aspirin if you have mild epigastric pain."

Correct Answers: A, B, C, and D Client teaching related to peptic ulcer disease (PUD) includes lifestyle changes (eg, dietary modifications, stress reduction), PUD complications, and medication administration. Helicobacter pylori infection and treatment with nonsteroidal anti-inflammatory drugs (NSAIDs) are risk factors for complicated PUD. H pylori treatment includes antibiotics and proton-pump inhibitors for acid suppression. The recommended initial treatment is 7-14 days of triple-drug therapy with omeprazole (Prilosec), amoxicillin, and clarithromycin (Biaxin). Incorrect Answer: [E. "You may take over-the-counter drugs such as aspirin if you have mild epigastric pain."] Clients with PUD should avoid NSAIDs [eg, aspirin, ibuprofen (Motrin)] as they inhibit prostaglandin synthesis, increase gastric secretion, and reduce the integrity of the mucosal barrier. Educational objective:Clients with peptic ulcer disease should avoid NSAIDs, smoking, and excess use of alcohol or caffeine.

The nurse assists with data collection during a screening event at a gastroenterology clinic. Which of the following client statements include risk factors for esophageal cancer? Select all that apply. A. "A few years ago, I switched from smoking cigarettes to smoking cigars 1 or 2 times a week." B. "I am proud that I was able to lose 10 lb, but I'm still considered obese for my height." C. "I drink 3 or 4 beers nightly to relax, but I did switch to light beer recently." D. "I have struggled with daily episodes of acid reflux for years, especially at nighttime." E. "I snack on a lot of salted foods like popcorn and peanuts."

Correct Answers: A, B, C, and D Esophageal cancer is a rare, rapidly growing malignancy of the esophageal lining with a low 5-year survival rate. Squamous cell carcinoma usually develops in the upper part of the esophagus, whereas adenocarcinoma usually develops in the lower part. Major risk factors include smoking (eg, cigarettes, pipes, cigars) and excessive alcohol consumption (ie, approximately >15 drinks/week for men, >8 drinks/week for women). Barrett esophagus is also a significant risk factor for esophageal cancer; this condition occurs when the distal portion of the esophagus develops precancerous changes. Obesity (which allows stomach acid to flow upward into the esophagus due to increased abdominal pressure) and uncontrolled gastroesophageal reflux disease contribute to the development of Barrett esophagus; they are both closely linked with esophageal cancer. Incorrect Answers: [E. "I snack on a lot of salted foods like popcorn and peanuts."] Consumption of salty foods is not associated with an increased risk of esophageal cancer but increases the risk for gastric cancer. Dietary factors that may increase a client's risk of esophageal cancer include high intake of nitrosamine-containing foods (eg, pickled foods, beer), frequent ingestion of extremely hot beverages (thermal injury), and deficient intake of fruits and vegetables. Educational objective:Esophageal cancer is a rapidly growing malignancy of the esophageal lining. Risk factors for esophageal cancer include smoking, excessive alcohol consumption, obesity, and gastroesophageal reflux disease.

The practical nurse is collaborating with the registered nurse to create a care plan for a client experiencing exophthalmos as a complication of Graves disease. Which interventions should be included in the client's plan of care? Select all that apply. A. Administer artificial tears to moisten the conjunctiva B. Lightly tape eyelids shut if they do not close during sleep C. Recommend the use of dark glasses to prevent irritation D. Teach about the importance of smoking cessation E. Teach avoidance of eye movement to prevent further damage

Correct Answers: A, B, C, and D Exophthalmos is a complication of hyperthyroidism (hypermetabolic state due to thyroid hormone overproduction) from Graves disease. It is defined as protrusion of the eyeballs caused by increased orbital tissue (connective, adipose, muscular) expansion and can be irreversible. The exposed cornea is at risk for dryness, injury, and infection. Nursing care for a client with exophthalmos includes the following: — Maintaining the head of the bed in a raised position to facilitate fluid drainage from the periorbital area — Using artificial tears or similar products to moisten the eyes to prevent corneal drying (causes abrasions/ulcers) — Taping the client's eyelids shut during sleep if they do not close on their own — Teaching the client the following: · Regular visits to the ophthalmologist are necessary to measure eyeball protrusion and evaluate the condition. · If recommended, anti-thyroid drugs should be used to prevent further exacerbation of exophthalmos. · Smoking cessation is necessary as smoking increases the risk of Graves disease and associated eye problems (Option 4). · Restrict salt intake to decrease periorbital edema. · Use dark glasses to decrease glare and prevent external irritants and infection (Option 3). Incorrect Answers: [E. Teach avoidance of eye movement to prevent further damage] The client should perform intraocular muscle exercises (turning the eyes using complete range of motion) to maintain flexibility. Educational objective:Exophthalmos is a complication of hyperthyroidism from Graves disease that leads to increased orbital tissue (connective, adipose, muscular) expansion that can be irreversible. Nursing care to keep the eyes moist and protected is necessary to prevent corneal ulcers and infection.

The nurse is reinforcing teaching on oral care and symptom management to a client with head and neck cancer who has developed mouth sores related to external radiation therapy. What should the nurse encourage the client to do? Select all that apply. A. Apply a water-soluble lubricating agent to moisturize mouth tissue B. Avoid hot liquids and foods that are spicy or acidic C. Brush teeth with a soft-bristle toothbrush D. Cleanse the mouth with normal saline after meals and at bedtime E. Rinse with alcohol-based antiseptic mouthwash to decrease mouth odor F. Use palifermin as prescribed to alleviate oral pain

Correct Answers: A, B, C, and D Oral mucositis (inflammation or ulceration of the oral mucosa) results from chemotherapy or radiation therapy. Oral hygiene practices that minimize oral mucositis and promote comfort include the following: — Use of water-soluble lubricating agents to moisten mouth tissues that may become dry due to radiation therapy — Avoidance of hot liquids and spicy/acidic foods, which can cause oral discomfort — Use of a soft-bristle toothbrush to decrease gum irritation — Cleansing the mouth with normal saline after meals and at bedtime to promote oral health — Application of prescribed viscous lidocaine HCl (xylocaine) to alleviate oral pain Incorrect Answer: [E. Rinse with alcohol-based antiseptic mouthwash to decrease mouth odor] Clients with mucositis should avoid antiseptic mouthwashes with alcohol as they are irritating to mucous membranes. [F. Use palifermin as prescribed to alleviate oral pain] Administration of palifermin (Kepivance), a recombinant human keratinocyte growth factor, can shorten the duration of oral mucositis in clients diagnosed with hematologic malignancies. However, it does not help with pain. Educational objective:The nurse can reinforce measures to minimize oral mucositis from chemotherapy and radiation therapy, including rinsing the mouth with normal saline, brushing with a soft-bristle toothbrush, using a water-soluble lubricating agent, avoiding hot liquids and spicy/acidic foods, and applying prescribed viscous lidocaine.

The nurse is reinforcing home care education to a client newly diagnosed with von Willebrand disease. Which of the following client statements demonstrate correct understanding of the education? Select all that apply. A. "I can use a humidifier to help prevent nosebleeds." B. "I need to avoid contact sports such as soccer or hockey." C. "I should use a soft-bristled toothbrush and electric razor." D. "I will call my health care provider if I soak a menstrual pad in an hour." E. "I will take naproxen to decrease pain and inflammation if I am injured."

Correct Answers: A, B, C, and D Von Willebrand disease is a genetic bleeding disorder caused by a deficiency of von Willebrand factor (vWF), which plays an important role in coagulation. Intranasal desmopressin or topical therapies (eg, thrombin) may be prescribed to stop minor bleeding, whereas major bleeding may require replacement of vWF. Clients should wear medical identification bracelets in case of emergency. Client teaching includes: — Notify the health care provider of signs of bleeding (eg, severe joint pain or swelling, headache [especially after injury], blood in urine/stool, uncontrollable nosebleed) — Use a humidifier or nasal spray to keep the mucosa moist, reducing the risk of nosebleeds — Avoid aspirin and NSAIDs — Avoid activities with a higher risk for injury (eg, contact sports) — Avoid gum injury (eg, use soft-bristled toothbrush, perform gentle flossing) and use an electric razor for shaving to minimize bleeding potential — Report heavy menstrual bleeding (eg, soaking a pad in <3 hours) Incorrect Answers: [E. "I will take naproxen to decrease pain and inflammation if I am injured."] Clients should avoid medications that can exacerbate bleeding, including aspirin and NSAIDs (eg, ibuprofen, naproxen, ketorolac). Clients can instead use rest, ice, compression, and elevation (RICE), as well as acetaminophen, to help with pain and inflammation. Educational objective:In von Willebrand disease, a genetic bleeding disorder, a deficiency in von Willebrand factor prevents effective coagulation. Clients can decrease bleeding risk by avoiding high-risk activities (eg, contact sports) and NSAIDs, keeping nasal mucosa moist, and maintaining gum integrity (eg, soft-bristled toothbrush).

The nurse reinforces education about safety modifications in the home for the spouse of a client diagnosed with Alzheimer disease. What instructions should the nurse include? Select all that apply. A. Arrange furniture to allow for free movement B. Keep frequently used items within easy reach C. Lock doors leading to stairwells and outside areas D. Place an identifying symbol on the bathroom door E. Provide a dark room free of shadows for sleeping

Correct Answers: A, B, C, and D When a client with Alzheimer disease is being cared for in the home, the caregiver should be instructed regarding safety modifications to ease the burden of caregiving and promote the client's independence and dignity. Injury-prevention modifications include: — Arrange furniture to allow for free movement to prevent falls — Place frequently used items within easy, visible reach of the client — Place locks on stairwells and outside doors to decrease the client's risk of falls and becoming lost during periods of wandering — Label the doors to the bathroom and other commonly used rooms to assist with environment interpretation and promote independent functioning Incorrect Answer: [E. Provide a dark room free of shadows for sleeping] Providing a night light in the sleeping area can prevent falls, aid in orientation, and decrease illusions. Educational objective:Caregivers of clients with Alzheimer disease should be taught safety modifications for the home, such as placing frequently used items within reach, arranging furniture to allow for free movement, labeling doors to commonly used rooms, providing a night light, and locking stairwell and outside doors.

A client comes to the emergency department for the second time with shortness of breath and substernal pressure that radiates to the jaw. The nurse understands that angina pectoris may be precipitated by which of these factors? Select all that apply. A. Amphetamine use B. Cigarette smoking C. Cold exposure D. Deep sleep E. Sexual intercourse

Correct Answers: A, B, C, and E Angina pectoris is defined as chest pain brought on by myocardial ischemia (decreased blood flow to the heart muscle). Any factor that increases oxygen demand or decreases oxygen supply to cardiac muscle may cause angina, including the following: — Physical exertion (eg, exercise, sexual activity): Increases heart rate and reduces diastole (time of maximum blood flow to the myocardium) — Intense emotion (eg, anxiety, fear): Initiates the sympathetic nervous system and increases cardiac workload — Temperature extremes: Usually cold exposure and hypothermia (vasoconstriction); occasionally hyperthermia (vasodilation and blood pooling) — Tobacco use and second-hand smoke inhalation: Replaces oxygen with carbon monoxide; nicotine causes vasoconstriction and catecholamine release — Stimulants (eg, cocaine, amphetamines): Increase heart rate and cause vasoconstriction — Coronary artery narrowing (eg, atherosclerosis, coronary artery spasm): Decreases blood flow to myocardium Incorrect Answer: [D. Deep sleep] Deep sleep doesn't increase oxygen demand. Educational objective:Angina pectoris is chest pain caused by myocardial ischemia. Any factor that increases oxygen demand or decreases oxygen supply may deprive the myocardium of necessary oxygen needed to function effectively.

The nurse is caring for a client with an exacerbation of asthma following a viral respiratory illness. When collecting data, the nurse expects to find which clinical characteristics of a severe asthma exacerbation? Select all that apply. A. Accessory muscle use B. Chest tightness C. High-pitched expiratory wheeze D. Prolonged inspiratory phase E. Tachypnea

Correct Answers: A, B, C, and E Asthma is an obstructive lung disease characterized by hyperreactive airways and chronic inflammation. Asthma exacerbations occur due to various triggers (eg, allergens, respiratory infection, exercise, cold air), resulting in edema, hypersecretion of mucus, and bronchospasm. Narrowing of the airways culminates in air trapping, lung hyperinflation, and increased airway resistance. In severe asthma, breath sounds may be diminished due to closure of bronchioles. Absent breath sounds in a client with asthma are a medical emergency. Clinical manifestations of an asthma exacerbation include: — Accessory respiratory muscle use related to increased work of breathing and diaphragm fatigue — Chest tightness related to air trapping — Cough from airway inflammation and increased mucus production — Diminished breath sounds related to hyperinflation — High-pitched expiratory wheezing caused by narrowing airways. As asthma worsens, wheezing may be heard on both inspiration and expiration. — Tachypnea related to inability to take a full, deep breath. Incorrect Answers: [D. Prolonged inspiratory phase] Clients with obstructive lung disease (eg, asthma, chronic obstructive pulmonary disease) develop a prolonged expiratory phase as a physiologic response to hyperinflation and trapped air. Educational objective:Asthma is an obstructive lung disease characterized by hyperreactive airways and chronic inflammation. Clinical manifestations of an asthma exacerbation include accessory muscle use, chest tightness, diminished breath sounds, high-pitched wheezing on expiration, prolonged expiratory phase, tachypnea, and cough.

The nurse is reviewing a client's health history during a primary care visit. Which of the following findings should the nurse identify as risk factors for developing hypertension? Select all that apply. A. African American ethnicity B. Diabetes mellitus type 2 C. Frequent stress at work D. LDL of 94 mg/dL (2.43 mmoI/L) E. Smoking of 1 pack of cigarettes daily

Correct Answers: A, B, C, and E Risk factors for hypertension Nonmodifiable — Increasing age — Positive family history — African American ethnicity — Diabetes mellitus type 1 Modifiable — Excessive alcohol intake — Smoking — Obesity — Excessive sodium intake — Sedentary lifestyle — Increased stress — Hyperlipidemia — Diabetes mellitus type 2 Hypertension is referred to as the "silent killer" as many clients are asymptomatic. Untreated chronic hypertension can result in damage of various organs and tissues and increases the risk for renal failure, coronary artery disease, stroke, and heart failure. Appropriate client screening based on risk factors is key to preventing complications. This client has both nonmodifiable (eg, African American ethnicity) and modifiable (eg, diabetes mellitus type 2, chronic stress, smoking) risk factors. To prevent future comorbidities, the nurse should educate the client on smoking cessation, appropriate diabetes management, and therapeutic strategies for stress management at work. Incorrect Answers: [D. LDL of 94 mg/dL (2.43 mmoI/L)] Clients should be screened for potential hyperlipidemia. An LDL laboratory value of 94 mg/dL (2.43 mmol/L) is within recommended parameters (<100 mg/dL [<2.6 mmol/L]). Educational objective:Key risk factors for developing hypertension include African American ethnicity, increasing age, positive family history, smoking, excessive sodium and alcohol use, diabetes mellitus, obesity, hyperlipidemia, chronic stress, and sedentary lifestyle. Untreated hypertension increases client risk for coronary artery disease, stroke, heart failure, and renal failure.

A client diagnosed with cirrhosis is experiencing pruritus. Which strategies are appropriate for the nurse to teach the client to promote comfort and skin integrity? Select all that apply. A. Apply cool, moist washcloths to the affected areas B. Keep the fingernails trimmed short to minimize skin scratching C. Take a hot bath or shower to alleviate itching sensations D. Use skin protectant or moisturizing cream over unbroken skin E. Wear cotton gloves or long-sleeved clothing to avoid scratching

Correct Answers: A, B, D, and E A client with cirrhosis may experience pruritus (itching) due to buildup of bile salts beneath the skin. Clients with cirrhosis are also at an increased risk for skin breakdown due to the development of edema, which increases skin fragility and impedes wound healing, and the loss of muscle and fat tissue from pressure points (eg, heels, sacrum). The nurse encourages the client to cut the nails short, wear cotton gloves, and wear long-sleeved shirts to avoid injury to the skin from scratching. Other comfort measures include baking soda baths; calamine lotion; and cool, wet cloths, which cool and soothe irritated skin. Cholestyramine (Questran) may be prescribed to increase the excretion of bile salts in feces, thereby decreasing pruritus. It is packaged in powdered form, must be mixed with food (applesauce) or juice (apple juice), and should be given 1 hour after all other medications. Incorrect Answers: [C. Take a hot bath or shower to alleviate itching sensations] Temperature extremes (eg, hot baths/showers) may intensify pruritus. The nurse should instruct the client to bathe with tepid water until the pruritus has subsided. Educational objective:A client with cirrhosis may experience pruritus (itching) due to the buildup of bile salts beneath the skin. Comfort measures include encouraging the client to cut nails short and wear long-sleeved cotton shirts and cotton gloves. Baking soda baths, calamine lotion, and cool, wet cloths also help. Cholestyramine increases the excretion of bile salts through feces, thereby decreasing itching.

The nurse reinforces teaching about self-management strategies for a client with urge incontinence. Which of the following statements indicate that teaching has been effective? Select all that apply. A. "I am going to join a walking program to lose excess weight." B. "I may have dry mouth as a side effect from the oxybutynin." C. "I really need caffeine to get myself going in the morning." D. "I should perform Kegel exercises several times daily." E. "I will void every 2 hours until I am having fewer accidents."

Correct Answers: A, B, D, and E Urge incontinence (UI), also known as overactive bladder, occurs when the bladder contracts randomly, causing a strong, sudden urge to urinate that is followed by urine leakage. UI may occur without cause or may result from spinal cord injury and impairment of the bladder (eg, interstitial cystitis) or neurological system (eg, Parkinson disease, stroke). Interventions for clients with UI include: — Loss of excess weight to reduce pressure on the pelvic floor — Anticholinergic medications (eg, oxybutynin, tolterodine) to decrease bladder spasms. Dry mouth (xerostomia) is a frequent adverse effect — Avoidance of bladder irritants (eg, artificial sweeteners, caffeine, citrus juices, alcohol, carbonated drinks, nicotine) — Pelvic floor exercises (eg, Kegel) to strengthen the muscles and help prevent urinary leakage — Bladder training (eg, voiding every 2 hours while awake) and gradually lengthening intervals between voiding Educational objective:Management of urge incontinence includes loss of excess weight, anticholinergic medications (eg, oxybutynin), avoidance of bladder irritants, pelvic floor exercises, and bladder training. Dry mouth is a common adverse effect of anticholinergic medications.

The nurse is reviewing discharge teaching for a client who had surgical repair of a retinal detachment. Which of the following instructions are appropriate for the nurse to include in the teaching? Select all that apply. A. Avoid rubbing or scratching the affected eye B. Avoid straining when having a bowel movement C. Expect occasional flashes of light during recovery D. Report any sudden pain to the health care provider E. Rest the eyes by refraining from reading and writing

Correct Answers: A, B, D, and E. Retinal detachment is separation of the sensory retina from the underlying pigment epithelium. Clients experiencing retinal detachment may report a gradual, curtain-like loss of the visual field. Traumatic retinal detachment may also result in abrupt vision loss. Retinal detachment requires emergency surgery to attempt to restore vision. Surgical repair involves rebinding the choroid and retina. After repair, interventions focus on promoting retinal reattachment. Postoperative teaching should include: — Avoiding activities that increase intraocular pressure (eg, rubbing the eye, straining) — Reporting sudden pain, flashes of light, vision loss, or bleeding, which may indicate detachment or infection, to the health care provider — Avoiding focused activities (eg, reading, writing, sewing), which can cause rapid eye movements and increase the risk for detachment — Wearing an eye patch or shield as directed to prevent rubbing/scratching of the eye and minimize eye movement — Ensuring appropriate positioning as instructed by the surgeon because clients may receive intravitreal oil or gas, which holds the retina in a specific position to allow healing Incorrect Answer: [C. Expect occasional flashes of light during recovery] Signs of retinal detachment include floaters, sudden flashes of light, and loss of vision. If signs of detachment occur, the surgeon should be notified immediately. Educational objective:After retinal detachment repair, clients should avoid activities that increase intraocular pressure (eg, rubbing the eye, straining); report pain, flashes of light, or floaters; wear an eye patch; avoid focused activities that may cause eye strain; and minimize eye movement.

The nurse is reviewing medical histories with several clients during a community health screening event. Which of the following client statements indicate a risk factor for cervical cancer? Select all that apply. A. "I have had four sexual partners during my lifetime." B. "I have smoked cigarettes for many years." C. "I never used birth control pills because my partners wore condoms." D. "I received treatment for chlamydia when I was younger." E. "I tested positive for human papillomavirus a few years ago."

Correct Answers: A, B, D, and E. The most important risk factor for cervical cancer is persistent human papillomavirus (HPV) infection, a common, transient, and often asymptomatic sexually transmitted infection (STI) that can be identified in almost all clients with cervical cancer. Most other risk factors are related to acquiring, clearing, or increasing the cancer-causing effects of HPV infection, including: — Having multiple sexual partners (ie, >1 lifetime partner), which increases the chance of HPV exposure — Smoking tobacco, which is believed to promote cell mutation and increase the likelihood of HPV infection — Being infected with other STIs (eg, gonorrhea, chlamydia), which increases the likelihood of HPV infection Incorrect Answers: [C. "I never used birth control pills because my partners wore condoms."] Condoms help to prevent HPV transmission between partners, and not taking oral contraceptives is associated with a decreased risk for cervical cancer. Educational objective:Cervical cancer is a malignancy of the cervix associated with persistent human papillomavirus (HPV) infection. Most other risk factors for cervical cancer relate to increasing risk for or ability to clear HPV infections, including multiple sexual partners, history of other sexually transmitted infections, and smoking.

The nurse reinforces safety precautions of home oxygen use to a client with chronic obstructive pulmonary disease (COPD) being discharged with a nasal cannula and portable oxygen tank. Which client statement indicates the need for further teaching? Select all that apply. A. "I can apply Vaseline to my nose when my nostrils feel dry from the oxygen." B. "I can cook on my gas stove as long as I have a fire extinguisher in the kitchen." C. "I can increase the liter flow from 2 to 6 liters a minute whenever I feel short of breath." D. "I should not polish my nails when using my oxygen." E. "I should not use a wool blanket on my bed."

Correct Answers: A, B, and C Oxygen is a colorless, odorless gas that supports combustion and makes up about 21% of the atmosphere. Oxygen is not combustible itself, but it can feed a fire if one occurs. When using home oxygen, safety precautions are imperative. 1. Vaseline is an oil-based, flammable product and should be avoided. A water-soluble lubricant may be used instead. 2. Oxygen canisters should be kept at least 5-10 feet away from gas stoves, lighted fireplaces, wood stoves, candles, or other sources of open flames. Clients should use precautions as cooking oils and grease are highly flammable. 3. The prescribed concentration of oxygen, usually 24%-28% for clients with COPD, should be maintained. Oxygen is prescribed to raise the PaO2 to 60-70 mm Hg and the saturations from 90%-93%. A flow rate of 2 L/min provides approximately 28% oxygen concentration, and 6 L/min provides approximately 44%. Higher rates usually do not help and can even be dangerous in clients with COPD as they can decrease the drive to breathe. The client should notify the care provider about excessive shortness of breath as additional treatment may be indicated. Incorrect Answers [D. "I should not polish my nails when using my oxygen." ] The client understands that nail polish remover and nail polish contain acetone, which is highly combustible. [E. "I should not use a wool blanket on my bed."] Clients should avoid synthetic and wool fabrics because they can cause static electricity, which may ignite a fire in the presence of oxygen. Clients should use cotton blankets and wear cotton fabrics. Educational objective:Safety precautions for home oxygen use include the following: no smoking; keeping electrical devices in good condition and plugs grounded; avoiding volatile, flammable products and materials that generate static electricity; staying at least 5-10 feet away from open sources of flame; keeping fire extinguishers readily available; and regularly testing smoke detectors.

A client with ST segment elevation myocardial infarction (MI) is due for 9:00 AM medications. Based on the data shown in the exhibit, which medications should the nurse administer? Select all that apply. Vital Signs Temperature: 98.4 F (36.8 C) Blood Pressure: 110/72 Heart Rate: 52/min Respirations: 16/min Laboratory Results Hematocrit: 40% Hemoglobin: 14.0 g/dL Platelets: 200,000/mm*3 Potassium: 4.0 mEq/L Medications [Time] Aspirin: 81 mg by mouth, daily [0900] Docusate sodium: 100 mg by mouth [0900] Lisinopril: 5 mg by mouth [0900] Metoprolol: 100 mg by mouth, twice daily [0900 and 1700] Simvastatin: 20 mg by mouth, daily [2000] A. Aspirin B. Docusate Sodium C. Lisinopril D. Metoprolol E. Simvastatin

Correct Answers: A, B, and C The client should receive aspirin, docusate sodium, and lisinopril. Antiplatelet agents (eg, aspirin, clopidogrel) are administered to clients with heart disease to reduce inflammation and inhibit platelet aggregation. The client's platelet count (normal 150,000-400,000/mm*3 [150-400 X 10*9/L]), hemoglobin, and hematocrit are normal, so it is safe to administer this medication. Docusate sodium is a stool softener. The client who has had an MI should not strain during bowel movements due to the risk of producing a vagal response, putting the client at risk for bradycardia and other dysrhythmias. Angiotensin-converting enzyme (ACE) inhibitors (eg, lisinopril, benazepril, captopril) are commonly given to clients post MI to prevent the progression of heart failure. Because ACE inhibitors have the potential to cause hyperkalemia and hypotension, the nurse should assess the potassium level and blood pressure (BP) prior to administration. This client's BP and potassium (normal 3.5-5.0 mEq/L [3.5-5.0 mmol/L]) are within normal range. Incorrect Answers: [D. Metoprolol] Beta blockers (eg, metoprolol, propranolol, atenolol) are given to clients post MI to reduce the risk of reinfarction and the occurrence of heart failure. A side effect of this medication is bradycardia (<60/min). This client is experiencing bradycardia with a heart rate of 52/min. The nurse should hold this medication and report findings to the registered nurse. [E. Simvastatin] Simvastatin is a lipid-lowering medication prescribed to clients to reduce triglycerides and LDL cholesterol. This medication is not due at 9:00 AM. Educational objective:BP and serum potassium levels are checked prior to administration of ACE inhibitors. Heart rate should be checked prior to administration of beta blockers. Aspirin is given to clients with normal platelet counts. Stool softeners reduce straining during defecation which helps prevent a vagal response.

The nurse is caring for a client with Cushing syndrome. Which of the following clinical manifestations should the nurse expect? Select all that apply. A. Hyperglycemia B. Hypertension C. Hyponatremia D. Truncal Obesity E. Weight Loss

Correct Answers: A, B, and D Cushing syndrome is caused by prolonged exposure to excess corticosteroids, especially glucocorticoids. The most common cause is the administration of corticosteroids, such as prednisone or hydrocortisone. However, pituitary adenomas (tumors) can secrete adrenocorticotropic hormone, which stimulates the adrenal glands to produce too much cortisol. Clinical manifestations include: · Androgen excess from adrenal gland stimulation, causing acne, hirsutism (facial, chest, or back hair in women), or menstrual irregularities (eg, oligomenorrhea) · Metabolic complications such as hyperglycemia (ie, excess cortisol stimulating gluconeogenesis), hypertension, and truncal obesity (leading to insulin resistance); fat accumulation in the face ("moon face") and the back of the neck ("buffalo hump"), which is common (Options 1, 2, and 4) · Skin changes from loss of collagen, such as easy bruising, purple striae, and skin thinning · Muscle wasting and bone loss (ie, osteoporosis) due to steroid-induced catabolism, if the syndrome is not treated Incorrect Answers: [C. Hyponatremia] Hyponatremia and weight loss are signs of adrenocortical insufficiency or Addison disease. [E. Weight Loss] Hyponatremia and weight loss are signs of adrenocortical insufficiency or Addison disease. Educational objective:Clinical manifestations of Cushing syndrome include weight gain, truncal obesity, moon face, buffalo hump, skin thinning, easy bruising, purple striae, muscle wasting, bone loss, hypertension, and hyperglycemia. The associated excess of androgen can result in acne, hirsutism, and menstrual irregularities.

A client with a history of diverticular disease is being discharged after an episode of acute diverticulitis. Which instructions should the nurse reinforce to reduce the risk of future episodes? Select all that apply. A. Drink plenty of fluids B. Exercise regularly C. Follow a low-fiber diet D. Include whole grains, fruits, and vegetables in the diet E. Increase intake of red meat

Correct Answers: A, B, and D Diverticular disease of the colon is a condition in which there are sac-like protrusions in the large intestine (diverticula). Diverticulosis is characterized by the presence of these protrusions; the client is asymptomatic and may not even be aware of the condition. Diverticulitis occurs when diverticula become infected and inflamed. Complications of diverticulitis include abscess, fistula formation, intestinal obstruction, peritonitis, and sepsis. Diverticular bleeding occurs when a blood vessel next to one of these pouches bursts; this may cause blood in the stool. The etiology of diverticular disease has been linked to chronic constipation, a major cause of excess intracolonic pressure. Preventing constipation may help reduce the risk of diverticula forming and becoming inflamed. Measures to prevent constipation include a diet high in fiber (whole grains, fruits, vegetables), daily intake of at least 8 glasses of water or other fluids, and exercise. A fiber supplement such as psyllium or bran may be advised. In the past, clients have been taught to avoid consuming seeds, nuts, and popcorn; however, current evidence does not indicate that avoidance of these foods will prevent an episode of diverticulitis. Incorrect Answers: [C. Follow a low-fiber diet] A high-fiber diet is recommended to reduce the risk of future diverticulitis episodes. [E. Increase intake of red meat] Increased consumption of red meat and other high-fat or high-iron containing foods can lead to constipation and increase the risk of diverticulitis. Educational objective:Clients with diverticulosis should take measures to prevent constipation (eg, high-fiber diet, increased fluid intake, regular exercise), which may help prevent recurring episodes of acute diverticulitis.

The nurse is preparing to administer 40 mg of oral furosemide. Prior to administering the medication, the nurse should evaluate which parameters? Select all that apply. A. Blood pressure B. Blood urea nitrogen C. Liver enzymes D. Potassium E. White blood cell count

Correct Answers: A, B, and D Loop diuretics (furosemide, torsemide, bumetadine) are used to treat fluid retention, such as that found in clients with heart failure or cirrhosis. When administering loop diuretics, the nurse can expect the client's kidneys to excrete a significant amount of water and potassium. When potassium is excreted at a fast rate, the client could develop hypokalemia, a medical emergency that can result in other life-threatening complications such as heart arrythmias, as well as muscle cramps and weakness. Blood pressure should also be assessed prior to administration of loop diuretics as excess diuresis may cause intravascular volume depletion that results in low blood pressure. A client with baseline hypotension may develop a critically low blood pressure. Excess diuresis can also affect kidneys, and the blood urea nitrogen and creatinine levels can become elevated as well. Therefore, these levels should be assessed. Incorrect Answers: [C. Liver enzymes] Loop diuretics typically do not cause abnormalities in white blood cell counts or liver function tests, so these do not need to be assessed routinely. [E. White blood cell count] Loop diuretics typically do not cause abnormalities in white blood cell counts or liver function tests, so these do not need to be assessed routinely. Educational objective:When administering furosemide, it is important to closely monitor the client's vital signs, serum electrolytes (potassium), and kidney function tests (blood urea nitrogen, creatinine) prior to administration to prevent side effects such as hypokalemia, hypotension, and kidney injury.

The nurse is caring for a child newly diagnosed with cystic fibrosis. What interventions does the nurse expect to be included in the client's multidisciplinary plan of care? Select all that apply. A. Chest physiotherapy B. Genetic counseling C. Low-calorie diet D. Oral fluid restriction E. Spiritual support

Correct Answers: A, B, and E Cystic fibrosis (CF) is a genetic disorder involving the cells that line the respiratory, gastrointestinal (GI), and reproductive tracts. A defective protein responsible for transporting sodium and chloride causes the secretions from exocrine glands in these areas to be thicker and stickier than normal. These abnormal secretions plug smaller airway passages and ducts in the GI tract. Secretions of impaired digestive enzymes in the GI tract result in ineffective absorption of essential nutrients. The sticky respiratory secretions lead to an inability to clear the airway and a chronic cough. The client eventually develops chronic lung disease (bronchiectasis). As a result of these changes, the client's life span is shortened; most affected individuals live only into their 30s. Chest physiotherapy helps remove sticky secretions that cause ineffective airway clearance. Clients and parents should receive genetic testing and counseling as CF is transmitted in an autosomal recessive inheritance pattern. Spiritual support should be offered as clients must deal with the impact of CF on life span and future pregnancies. Incorrect Answers: [C. Low-calorie diet] Adequate nutrition is not possible due to defective digestive enzymes and impaired nutrient absorption. A diet high in fat and calories is recommended. [D. Oral fluid restriction] Oral fluids are not restricted, and liberal intake is recommended to assist with thinning the sticky respiratory secretions. Educational objective:Clients with cystic fibrosis should adhere to a diet high in fat and calories to combat nutrient malabsorption. Liberal fluid intake is encouraged to thin the thick secretions. Spiritual support should be offered to cope with the impact of a shortened life span.

The nurse reinforces disease management teaching to a group of clients with type 1 diabetes mellitus. Which of the following should the nurse include as signs or symptoms associated with hypoglycemia? Select all that apply. A. Diaphoresis B. Flushing C. Pallor D. Polyuria E. Trembling

Correct Answers: A, B, and E Hypoglycemia (low blood glucose <70 mg/dL [3.9 mmol/L]) is an acute and potentially serious complication that occurs when insulin levels exceed the proportion of glucose. Epinephrine is one of the major hormones released during a hypoglycemic reaction and may cause early symptoms such as trembling, palpitations, anxiety/arousal, and restlessness. Diaphoresis and pallor are present on examination. Confusion, seizures, or coma can occur when the brain is deprived of glucose (neuroglycopenia) due to prolonged and severe hypoglycemia. Incorrect Answers [B. Flushing] Flushing (red skin) is commonly seen with fever, polycythemia vera, and sexual intercourse. Flushing is not seen with hypoglycemia. [D. Polyuria] Polyuria and weight loss are usually associated with hyperglycemia, not hypoglycemia. Educational objective:Hypoglycemia (blood glucose <70 mg/dL [3.9 mmol/L]) is an acute and potentially serious complication. Signs and symptoms include trembling, palpitations, anxiety/arousal, restlessness, diaphoresis, and pallor.

A client who developed heart failure after a myocardial infarction is scheduled to be discharged this afternoon. Based on the discharge data, the nurse plans to reinforce which home care instructions? Select all that apply. A. How to take own pulse B. Monitoring daily weight C. Need for monthly International Normalized Ratio testing D. Need to increase foods high in potassium E. Reduction of sodium in diet F. Use of home oxygen

Correct Answers: A, B, and E This client with heart failure would need to measure weight daily, restrict sodium and fluid intake, and know how to take a pulse. Incorrect Answers: [C. Need for monthly International Normalized Ratio testing] This client is not taking warfarin, so monthly testing of International Normalized Ratio is not indicated. [D. Need to increase foods high in potassium] Spironolactone is a potassium-sparing diuretic, so there is no need to increase dietary potassium. Angiotensin-converting enzyme inhibitors such as captopril can cause hyperkalemia. [F. Use of home oxygen] The client's SpO2 was 96% on room air and home oxygen has not been prescribed. Educational objective:The client being discharged with heart failure should understand weight monitoring, diet, medication regimen, activity, and symptoms to report.

Which teaching instructions should the nurse reinforce to a client with advanced chronic obstructive pulmonary disease? Select all that apply. A. Follow a low-calorie diet B. Obtain a pneumococcal vaccine C. Report increased sputum D. Take iron to improve anemia E. Use an incentive spirometer

Correct Answers: B & C The client with chronic obstructive pulmonary disease (COPD) is very susceptible to pulmonary infections. Most exacerbations are caused by bacterial or viral infection. Clients with COPD typically have some mucus production and cough at baseline. However, the presence of increased dyspnea, increased sputum volume, or sputum purulence indicates bacterial infection and would benefit from antibiotics. It is important for the client to treat an exacerbation as soon as possible. The current recommendations are that clients with COPD, regardless of age, should receive the pneumococcal vaccine. Clients should also receive a yearly influenza virus vaccine. Incorrect Answers: [A. Follow a low-calorie diet] Clients with advanced COPD are underweight. The loss of muscle mass is attributed to aging, steroid use (catabolic effect), and the effort to eat with dyspnea. Extra intake of protein and calories is encouraged. [D. Take iron to improve anemia] Clients with COPD are at risk for polycythemia. The kidney is stimulated to make erythropoietin and increase red blood cells due to hypoxia. Anemia is not expected in this condition. [E. Use an incentive spirometer] Incentive spirometry gives the client visual feedback and encouragement to take deep breaths. It is used routinely for postoperative atelectasis. The pathophysiology of COPD involves structural changes and permanent airflow limitations that result in trapped air. The client needs help to expel the air, not get more air into the lungs. Educational objective:Clients with chronic obstructive pulmonary disease (COPD) need to report any signs of infection (including change in their normal sputum) as infection is a primary cause of exacerbation. Any client with COPD should have a pneumococcal vaccine and an annual influenza virus vaccine.

The nurse is reinforcing teaching to a client diagnosed with Raynaud phenomenon about ways to prevent recurrent episodes. Which instructions should the nurse include? Select all that apply. A. Avoid excessive caffeineB. Immerse hands in cold waterC. practice yoga or tai chiD. Refrain from using tobacco productsE. Wear gloves when handling cold objects

Correct Answers: A, C, D, and E Raynaud phenomenon is a vasospastic disorder resulting in an episodic vascular response related to cold temperatures or emotional stress. It most commonly affects women age 15-40. Vasospasms induce a characteristic color change in the appendages (eg, fingers, toes, ears, nose). When vasoconstriction occurs, the affected appendage initially turns white from decreased perfusion, followed by a bluish purple appearance due to cyanosis. Clients usually report numbness and coldness during this stage. When blood flow is subsequently restored, the affected area becomes reddened, and clients experience throbbing or aching pain, swelling, and tingling. Acute vasospasms are treated by immersing the hands in warm water. Client teaching regarding prevention of vasospasms includes: — Wear gloves when handling cold objects — Dress in warm layers, particularly in cold weather. — Avoid extremes and abrupt changes in temperature. — Avoid vasoconstricting drugs (eg, cocaine, amphetamines, ergotamine, pseudoephedrine). — Avoid excessive caffeine intake — Refrain from use of tobacco products — Implement stress-management strategies (eg, yoga, tai chi) If conservative management is unsuccessful, calcium channel blockers may be prescribed to relax arteriole smooth muscle and prevent recurrent episodes. Incorrect Answer: [B. Immerse hands in cold water] Cold water causes vasoconstriction and worsens the condition. Educational objective:Raynaud phenomenon is a vasospastic disorder triggered by exposure to cold or stress. Key elements of client teaching include management of acute attacks, avoidance of vasoconstrictive substances (eg, tobacco, cocaine, caffeine), stress reduction, and appropriate clothing (eg, gloves, warm layers).

A client admitted 3 days ago with upper gastrointestinal bleeding underwent an endoscopic procedure to stop the bleeding. The client is started on a clear liquid diet today. Which foods are appropriate for the nurse to offer the client? Select all that apply. A. Apple juice B. Cherry popsicle C. Chicken broth D. Frozen yogurt E. Unsweetened tea F. Vanilla ice cream

Correct Answers: A, C, E A client recovering from abdominal surgery first consumes ice chips after demonstrating adequate bowel function (return of bowel sounds and passing flatus). After ice chips, postoperative diet progression continues to clear liquids, full liquids, soft diet, and then regular diet. Unsweetened tea, chicken broth, and apple juice are appropriate food choices for a client on a clear liquid diet. Incorrect Answers: [B. Cherry popsicle] Popsicles are part of a clear liquid diet. However, red dyes in clear liquids (eg, cherry popsicles, red gelatin) should not be given to clients with recent gastrointestinal bleeding. If a client vomits, the vomitus may appear red and falsely lead the nurse to believe that the client is bleeding. It is important to implement prudent nursing judgment and fully consider the client's condition when making care decisions; for this client, a green or yellow popsicle would be more appropriate. [D. Frozen yogurt] Frozen yogurt and vanilla ice cream are appropriate food choices for a client on a full liquid diet. [F. Vanilla ice cream] Frozen yogurt and vanilla ice cream are appropriate food choices for a client on a full liquid diet. Educational objective:A postoperative diet begins with ice chips and progresses to clear liquids, full liquids, soft diet, and then regular diet. Clear liquids with red dyes should not be given to clients with recent gastrointestinal bleeding.

The nurse is caring for a client after a motor vehicle accident. The client's injuries include 2 fractured ribs and a concussion. The nurse notes which findings as expected neurological changes for the client with a concussion? Select all that apply. A. Amnesia B. Asymmetrical pupillary constriction C. Brief loss of consciousness D. Headache E. Loss of vision

Correct Answers: A, C, and D A concussion is considered a minor traumatic brain injury and results from blunt force or an acceleration/deceleration head injury. Typical signs of concussion include: 1. A brief disruption in level of consciousness 2. Amnesia regarding the event (retrograde amnesia) 3. Headache These clients should be observed closely by family members and not participate in strenuous or athletic activities for 1-2 days. Rest and a light diet are encouraged during this time. Incorrect Answers [B. Asymmetrical pupillary constriction] & [E. Loss of vision] The following manifestations indicate more serious brain injury and are not expected with simple concussion: · Worsening headaches and vomiting (indicate high intracranial pressure) · Sleepiness and/or confusion (indicate high intracranial pressure) · Visual changes · Weakness or numbness of part of the body Educational objective:Expected neurological changes with a concussion include brief loss of consciousness, amnesia regarding the event, and headache. These clients should be observed closely by family members and not participate in strenuous or athletic activities for 1-2 days.

A client 4 days post colostomy is preparing to be discharged home. Which findings are concerning and should be further investigated? Select all that apply. A. Client states, "I will need home health to empty the pouch." B. Client states, "There is a little gas in the colostomy bag." C. No bowel sounds are present and the client reports nausea D. Skin surrounding the stoma is red and excoriated E. Stoma is red, edematous, and smaller than the previous day

Correct Answers: A, C, and D After a colostomy, the stoma should be beefy red and edematous but will begin to shrink over the course of a few days as inflammation subsides. There should be no mucocutaneous separation (eg, separation of the stoma from the abdominal wall), unusual bleeding (eg, moderate to large amounts of blood in the ostomy pouch), or signs of inadequate circulation, including stoma ischemia (eg, pale, dusky) and necrosis (eg, dark red, purple, black). Appliances should be resized during the first several weeks to ensure proper fit, preventing skin breakdown (eg, excoriation) due to stool coming into contact with the skin. Within 24 hours of surgery, the client should demonstrate signs of returning gastrointestinal motility, including resolution of nausea, active bowel sounds, and flatus. Nausea and absent bowel sounds may indicate postoperative ileus and should be reported to the health care provider. Clients should change the pouch according to the manufacturer's instructions (every 5-10 days) and if the skin surrounding the stoma becomes irritated (eg, burning). The nurse should also assess the client with a new ostomy for body image disturbance and ineffective coping (eg, client unwilling to care for the ostomy). Educational objective:Careful assessment of clients with new ostomies should include the stoma site (eg, perfusion, approximation to the skin), gastrointestinal function (eg, bowel sounds, flatus, stool), and self-care and body image. Appliances must be properly fitted to prevent skin breakdown (eg, excoriation).

A client with emphysema comes for a routine follow-up visit. The nurse assisting with the initial assessment knows that which manifestations are characteristic of emphysema? Select all that apply. A. Barrel chest B. Bilateral coarse crackles C. Decreased activity tolerance D. Diminished breath sounds E. Increased sputum production

Correct Answers: A, C, and D Chronic obstructive pulmonary disease (COPD) includes emphysema and chronic bronchitis; a component of each can be present. Emphysema is characterized by destruction of alveolar walls and obstruction to airflow. As a result, there is loss of lung tissue elasticity (recoil), enlargement of air spaces, air trapping, impaired gas exchange, and increased work of breathing. The major contributing factor to the development of emphysema is smoking tobacco (95%). Characteristic manifestations of emphysema include the following: 1. The anteroposterior to transverse diameter ratio is increased from 5:7 (normal) to 1:1 in the presence of barrel chest. This increase results in a "barrel" appearance, with a more horizontal rather than vertical slant to the ribs. It is associated with chronic emphysema as a result of hyperinflation of the lungs. 2. Decreased activity tolerance occurs due to worsening dyspnea and fatigue related to inadequate oxygen supply to meet the client's increased oxygen demand. 3. Diminished breath sounds with prolonged expiration result from hyperinflation, air trapping, and decreased gas exchange. The normal inspiration to expiration ratio of 1:2 decreases to 1:3 or 1:4. Additional findings may include hyperresonance on percussion and muffled heart tones due to over-distension of lungs. 4. Pursed-lip breathing, use of accessory muscles, and use of tripod position (leaning forward) are seen in advanced stages. Incorrect Answers: [B. Bilateral coarse crackles] Bilateral coarse crackles indicate pulmonary edema (fluid in the lungs). COPD exacerbation with underlying emphysema is more commonly associated with wheezing. [E. Increased sputum production] Cough with increased sputum production is more commonly associated with chronic bronchitis. Educational objective:Characteristic manifestations of emphysema include shortness of breath, exercise intolerance, tachypnea, diminished breath sounds with a prolonged expiratory phase, use of tripod position (leaning forward), and presence of barrel chest.

The nurse is reviewing discharge instructions on home management for a client with peripheral arterial disease. Which statements indicate a correct understanding of the instructions? Select all that apply. A. "I will apply moisturizing lotion on my legs every day." B. "I will elevate my legs at night when I am sleeping." C. "I will keep my legs below heart level when sitting." D. "I will start walking outside with my neighbor." E. "I will use a heating pad to promote circulation."

Correct Answers: A, C, and D Peripheral arterial disease (PAD) is a chronic, atherosclerotic disease caused by buildup of plaque within the arteries. PAD commonly affects the lower extremities and can lead to tissue necrosis (gangrene). Home management instructions for PAD include: — Lower the extremities below the heart when sitting and lying down - improves arterial blood flow — Engage in moderate exercise (eg, 30- to 45-minute walk, twice daily) - promotes collateral circulation and distal tissue perfusion — Perform daily skin care, including application of lotion - prevents skin breakdown from dry skin — Maintain mild warmth (eg, lightweight blankets, socks) - improves blood flow and circulation — Stop smoking - prevents vessel spasm and constriction — Avoid tight clothing and stress - prevents vasoconstriction — Take prescribed medications (eg, vasodilators, antiplatelets) - increases blood flow and prevents blood clot development Incorrect Answers: [B. "I will elevate my legs at night when I am sleeping."] Elevating the legs promotes venous return, but does not promote arterial circulation. [E. "I will use a heating pad to promote circulation."] Heating pads should not be used in clients with altered perfusion or sensation due to the increased risk for burns. Educational objective:Peripheral artery disease increases the risk of tissue necrosis and limb loss. Management focuses on improving blood flow and circulation to the extremities through lifestyle changes and medications.

The nurse is reinforcing discharge instructions with a client following a partial gastrectomy. Which of the following instructions should the nurse include to prevent dumping syndrome? Select all that apply. A. Add high-protein foods to diet B. Consume high-carbohydrate meals C. Eat small, frequent meals D. Increase intake of fluids with meals E. Lie down after eating

Correct Answers: A, C, and E Following a partial gastrectomy, many clients experience dumping syndrome, which occurs when gastric contents empty too rapidly into the duodenum, causing a fluid shift into the small intestine. This results in hypotension, abdominal pain, nausea/vomiting, dizziness, generalized sweating, and tachycardia. The symptoms usually diminish over time. Recommendations to delay gastric emptying include: · Consume meals high in fat, protein, and fiber, which take more time to digest and remain in the stomach longer than carbohydrates. These foods also help meet the body's energy needs. · Avoid consuming fluids with meals because this causes stomach contents to pass faster into the jejunum, which worsens symptoms. Fluid intake should occur up to 30 minutes before or after meals. · Slowly consume small, frequent meals to reduce the amount of food in the stomach. · Avoid meals high in simple carbohydrates (eg, sugar, syrup) because these may trigger symptoms when the carbohydrates break down into simple sugars. · Avoid sitting up after a meal because gravity increases gastric emptying. Instead, lying down after meals is encouraged. Incorrect Answers: [B. Consume high-carbohydrate meals] Avoid meals high in simple carbohydrates because these may trigger dumping syndrome. [D. Increase intake of fluids with meals] Avoid consuming fluids with meals to reduce the risk of dumping syndrome. Educational objective:Dumping syndrome is a complication of gastrectomy. To delay gastric emptying and reduce the risk of dumping syndrome, clients should consume meals low in carbohydrates and high in fiber, proteins, and fats; avoid fluids during meals; eat small, frequent meals; and lie down after eating.

The nurse is reinforcing teaching to a group of clients diagnosed with diabetes mellitus. Which lessons regarding foot care should be included? Select all that apply. A. Cut toenails straight across and file along the curves of the toes B. Rub feet vigorously with a towel after bathing to ensure dryness C. Use a mild foot powder on perspiring feet D. Use an over-the-counter kit for corns or calluses E. Use cotton or lamb's wool to separate overlapping toes

Correct Answers: A, C, and E Individuals with type 1 or 2 diabetes mellitus (DM) are at increased risk for developing an infection or ulcer on their feet. This is due to peripheral neuropathy, a chronic complication of DM that results from nerve damage in the extremities. Instructions for diabetic foot care include the following: 1. Wash feet daily with warm water and mild soap; test water temperature with a thermometer beforehand. Gently pat feet dry, particularly between the toes. Use lanolin to prevent dry and cracked skin, but do not apply between the toes. 2. Inspect for abrasions, cuts, or sores. Have others inspect the feet if eyesight is poor. 3. To prevent injury, use cotton or lamb's wool to separate overlapping toes. Cut toenails straight across and use a nail file to file along the curves of the toes. Avoid going barefoot and wear sturdy leather shoes. Use mild foot powder to absorb perspiration and wear clean absorbent socks with seams aligned 4. Avoid using over-the-counter products (eg, iodine, alcohol, strong adhesives) on cuts or abrasions 5. To improve circulation, do not sit with legs crossed or for extended periods, avoid tight-fitting garments, and exercise daily. 6. Report other types of problems such as infections or athlete's foot immediately. Educational objective:Individuals with type 1 or 2 diabetes mellitus are at increased risk for developing an infection or ulcer on their feet due to the chronic complication of peripheral neuropathy. Clients should keep their feet clean, dry, and free from irritation.

A client with throat cancer receives radiation therapy to the head and neck. Which strategies are appropriate to increase oral intake? Select all that apply. A. Avoid irritants such as acidic, spicy foods B. Discourage the use of topical analgesics C. Encourage liquid nutritional supplements D. Perform oral hygiene once a day E. Use artificial saliva to control dryness

Correct Answers: A, C, and E Radiation therapy to the head and neck can decrease a client's oral intake due to the development of mucositis (ie, inflammation of the mouth, esophagus, and oropharynx) and xerostomia (ie, dry mouth). These adverse side effects affect speech, taste, and ability to swallow and can have a significant impact on the client's nutritional status. The nurse teaches the client to: — Avoid irritants such as spicy, acidic, dry, or crumbly foods; coffee; and alcohol — Consume supplemental nutritional drinks (eg, Ensure), which are often easier to swallow — Use artificial saliva to manage xerostomia and the production of thick saliva due to altered salivary gland function. Sipping water throughout the day is equally effective and less expensive. Incorrect Answers: [B. Discourage the use of topical analgesics] Topical anesthetics (eg, lidocaine) have been found to increase comfort and improve oral intake in clients with mucositis due to radiation therapy. [D. Perform oral hygiene once a day] Clients on radiation therapy need to maintain more frequent (eg, before and after meals, at bedtime) oral hygiene (eg, using soft toothbrush, rinsing with baking soda solution) due to the drying effects of mucositis. Educational objective:Radiation therapy to the head and neck can cause mucositis (ie, inflammation of the mouth, esophagus, and oropharynx) and xerostomia (ie, dry mouth), leading to decreased nutrition. Care includes avoiding irritants, consuming supplements, using artificial saliva or sipping water, and performing frequent oral hygiene.

In the intensive care unit, the nurse cares for a client admitted with a head injury who develops syndrome of inappropriate antidiuretic hormone. Which data should the nurse expect with the onset of this condition? Select all that apply. A. Decreased serum osmolality B. High serum osmolality C. High urine specific gravity D. Increased urine output E. Low serum sodium

Correct Answers: A, C, and E Syndrome of inappropriate antidiuretic hormone (SIADH) is potential complication of head injury. In SIADH, the extra ADH leads to excessive water absorption by the kidneys. Low serum osmolality and low serum sodium are the result of increased total body water (dilution). As ADH is secreted and water is retained, urine output is decreased and concentrated, resulting in a high specific gravity. Incorrect Answers: [B. High serum osmolality] Increased urine output is associated with diabetes insipidus (DI). In DI, ADH is suppressed, causing polyuria, severe dehydration, and high serum osmolality if the client is unable to drink enough to maintain a fluid balance. [D. Increased urine output] Increased urine output is associated with diabetes insipidus (DI). In DI, ADH is suppressed, causing polyuria, severe dehydration, and high serum osmolality if the client is unable to drink enough to maintain a fluid balance. Educational objective:Syndrome of inappropriate antidiuretic hormone (SIADH) is a condition that results in increased ADH. Too much ADH causes increased total body water, resulting in a low serum osmolality and low serum sodium. As ADH is secreted and water is retained, urine output is decreased and concentrated, resulting in a high specific gravity.

The nurse understands that which of these body substances are modes of transmission for hepatitis B? Select all that apply. A. Blood B. Feces C. Semen D. Urine E. Vaginal secretions

Correct Answers: A, C, and E. Viral hepatitis is a disease of the liver characterized by inflammation, necrosis, and cirrhosis. One of the most common viral strains that causes hepatitis is hepatitis B. The transmission of hepatitis B is primarily through contact with blood, semen, and vaginal secretions (mnemonic: B for body fluids), commonly through unprotected sexual intercourse and intravenous illicit drug use. Infants born to infected mothers are also at risk for vertical transmission of hepatitis B. Although kissing, sneezing, sharing drinks/utensils, and breastfeeding are not known routes of transmission, hepatitis B could possibly be transmitted through saliva entering the bloodstream via sharing a toothbrush or receiving a bite. Hepatitis B has an insidious onset of illness, and clients may be asymptomatic carriers. Early symptoms are often nonspecific (eg, malaise, nausea, vomiting, abdominal pain). Hepatitis B may produce jaundice, weight loss, clay-colored stools, and thrombocytopenia in late stages of illness. An effective vaccine is widely available for hepatitis B. Incorrect Answers: [B. Feces] The transmission of hepatitis A occurs through the fecal-oral route via poor hand hygiene and improper food handling. Therefore, this infection is seen primarily in developing countries. Hepatitis B is not transmitted through feces. [D. Urine] Urine is not known to be a mode of transmission for any form of hepatitis. Educational objective:The transmission of hepatitis B occurs through parenteral or sexual contact with body fluids such as blood, semen, or vaginal secretions (mnemonic: B for body fluids).

A healthy 50-year-old client asks the nurse, "What must I do in preparation for my screening colonoscopy?" Which instructions should the nurse reinforce to correctly answer the client's question? Select all that apply. A. "No food or drink is allowed 8 hours prior to the test." B. "Prophylactic antibiotics are taken as prescribed." C. "Smoking must be avoided after midnight." D. "The day prior to the procedure your diet will be clear liquids." E. "You will drink polyethylene glycol as directed the day before."

Correct Answers: A, D, and E Colonoscopy is performed to evaluate the mucosa of the colon. Clients should follow instructions to keep the colon clean and free of stool for better visualization during the procedure. These instructions include the following: 1. Consume a clear liquid diet the day before. 2. Drink the prescribed bowel-cleansing agent, such as a cathartic, enema, or polyethylene glycol (GoLYTELY), the day before the test. The type of preparation depends on the health care provider's preference and client health status. 3. Have nothing by mouth 8-12 hours prior to the examination. Incorrect Answers: [B. "Prophylactic antibiotics are taken as prescribed."] Healthy clients screened for colon disease do not require antibiotics prior to the procedure. [C. "Smoking must be avoided after midnight."] The instructions prior to a nuclear gastric emptying scan include avoiding smoking the day of the examination as delay of emptying occurs with tobacco use. Smoking cessation per se has no role in colonoscopy, but it is good for general health. Educational objective:Instructions for clients scheduled for a colonoscopy include consuming a clear liquid diet the day before the procedure, taking the bowel-cleansing agent as prescribed, and having nothing by mouth 8-12 hours prior to the examination.

The nurse is reviewing lifestyle and nutritional strategies to help reduce symptoms in a client with newly diagnosed gastroesophageal reflux disease. Which strategies should the nurse include? Select all that apply. A. Choose foods that are low in fat B. Do not consume any foods containing dairy C. Eat three large meals a day and minimize snacking D. Limit or eliminate the use of alcohol and tobacco E. Try to avoid caffeine, chocolate, and peppermint

Correct Answers: A, D, and E Gastroesophageal reflux disease (GERD) occurs when chronic reflux of stomach contents causes inflammation of the esophageal mucosa. The lower esophageal sphincter (LES) normally prevents stomach contents from entering the esophagus. Any factor that decreases the tone of the LES (eg, caffeine, alcohol), delays gastric emptying (eg, fatty foods), or increases gastric pressure (eg, large meals) can precipitate GERD. Lifestyle and dietary measures that may prevent GERD and associated symptoms include: · Weight loss, as excessive abdominal fat may increase gastric pressure · Small, frequent meals with sips of water or fluids to help facilitate the passage of stomach contents into the small intestine and prevent reflux from becoming overly full during meals · Avoiding GERD triggers such as caffeine, alcohol, nicotine, high-fat foods, chocolate, spicy foods, peppermint, and carbonated beverages · Chewing gum to promote salivation, which may help neutralize and clear acid from the esophagus · Sleeping with the head of the bed elevated · Refraining from eating at bedtime and/or lying down immediately after eating Incorrect Answers: [B. Do not consume any foods containing dairy] Clients with GERD generally do not need to minimize or eliminate dairy products from the diet; however, they should choose low-fat or nonfat products. [C. Eat three large meals a day and minimize snacking] Small, frequent meals with sips of water or fluids to help facilitate the passage of stomach contents into the small intestine and prevent reflux from becoming overly full during meals Educational objective:Lifestyle and dietary measures that help prevent or minimize symptoms of gastroesophageal reflux disease include avoiding dietary triggers such as alcohol, caffeine, chocolate, peppermint, and high-fat foods. Clients should consume small, frequent meals and discontinue the use of tobacco products.

The nurse is caring for a client admitted for a seizure disorder. The nurse witnesses the client having a tonic-clonic seizure with increasing salivation. Which actions should the nurse take? Select all that apply. A. Call for help B. Hold down the client's arms C. Insert a tongue depressor to move the tongue D. Prepare for suctioning E. Turn the client on the side

Correct Answers: A, D, and E Safety is the immediate priority in a client experiencing a seizure. Nursing interventions include: — Remain at the client's bedside while noting duration and symptoms of the seizure — Call for help so that other team members can assist with care of the client — Protect client from hitting hard surfaces by padding the side rails — Turn client on the side if possible to allow for drainage of secretions and prevent the tongue from occluding the airway — Loosen clothing around the neck and chest to promote ventilation — Use suction equipment after the seizure subsides as needed to maintain a patent airway Incorrect Answers: [B. Hold down the client's arms] The client should not be restrained as this could cause injury. [C. Insert a tongue depressor to move the tongue] Oral airways should be kept at the bedside for postical airway management and recovery, but during an active seizure it is dangerous to attempt to insert anything in the client's mouth, especially if the teeth are clenched. Educational objective:During an active seizure, the nurse should call for additional help, turn the client on the side if possible, and have suction equipment ready to clear any excessive secretions that may block the airway. The nurse should not restrain the client or force anything into the client's mouth.

A client diagnosed with trichomonal vaginal infection (trichomoniasis) is prescribed metronidazole. Which directions are essential for the nurse to reinforce? Select all that apply. A. Avoid alcohol while taking this medication. B. Perform vaginal douche for 7 - 10 days C. Use birth control pills to prevent infection recurrence D. Your partner(s) must be treated simultaneously E. Your urine can change to a deep red-brown color

Correct Answers: A, D, and E Trichomoniasis is a sexually transmitted infection (STI). Many women are asymptomatic but can have profuse frothy gray or yellow-green vaginal discharge with a fishy odor. Small red lesions (strawberry color) may be present in the vagina or cervix. Pruritus is common. Metronidazole (Flagyl) is the initial drug of choice. Clients should avoid alcohol while taking metronidazole and for 24 hours after completion of the therapy due to the risk of a reaction that includes flushing, nausea/vomiting, and abdominal pain. The medication can have a metallic taste and turn the urine a deep red-brown color. It is essential to treat the affected client's partner(s) simultaneously to avoid reinfection. Clients should abstain from sexual intercourse until the infection is cleared, usually about 1 week after treatment. Incorrect Answers: [B. Perform vaginal douche for 7 - 10 days] Routine vaginal douching (with a mixture of water and vinegar) is not recommended as it increases the risk of infections such as bacterial vaginosis. [C. Use birth control pills to prevent infection recurrence] Birth control pills do not protect against STIs. However, condom use can help prevent the spread of infection. Educational objective:Trichomoniasis is a sexually transmitted infection. Expected symptoms include yellow-green, frothy discharge with a fishy odor and an accompanying itch. Metronidazole is the initial drug of choice; common side effects include darker urine and a metallic taste to the medication. Partners must be treated simultaneously. Clients taking metronidazole should be taught to avoid alcohol.

The nurse is caring for a client with community-acquired pneumonia. When collecting client data, the nurse should anticipate which findings? Select all that apply. A. Crackles B. High-pitched wheezing C. Hyperresonance D. Pleuritic chest pain E. Productive cough

Correct Answers: A, D, and E Pneumonia is an acute infection of the lung. Findings in a client with pneumonia include the following: 1. Fever, chills, productive cough, dyspnea, and pleuritic chest pain 2. Bronchial breath sounds in peripheral lung fields: High-pitched, harsh sounds (ie, bronchial sounds) are considered abnormal when heard in areas other than the trachea. Such sounds can indicate conduction through consolidated lung tissue and be an early sign of pneumonia. 3. Crackles: Coarse inspiratory crackles on auscultation are discontinuous, adventitious low-pitched sounds caused by air passing through airways that are intermittently obstructed with mucus 4. Increased vocal/tactile fremitus: Transmission of palpable vibrations (fremitus) is increased when transmitted through consolidated versus normal lung tissue. 5. Unequal chest expansion: Decreased expansion of affected lung is noted on palpation. 6. Dullness: Percussion of medium-pitched sounds occurs over consolidated lung tissue (pneumonia) or a fluid-filled space (ie, pleural effusion, a complication of pneumonia). Incorrect Answers: [B. High-pitched wheezing] High-pitched wheezing is auscultated when air is forced through a narrowed airway; wheezing is a finding in clients with asthma and anaphylactic allergic reactions. [C. Hyperresonance] Hyperresonance is percussed over a hyperinflated lung (eg, asthma, emphysema) or air in the pleural space (pneumothorax). Educational objective:Clinical findings in a client with pneumonia include fever, productive cough, dyspnea, and pleuritic chest pain. Auscultation of the lungs reveals bronchial breath sounds and crackles. Additional findings include dullness to percussion and increased tactile fremitus.

The nurse reinforces teaching about the use of the peak flow meter to evaluate airflow to a client newly diagnosed with asthma. Which statement by the client indicates an understanding of the correct technique? A. "I will exhale as quickly and forcibly as possible through the mouthpiece of the device to obtain a peak flow reading." B. "I will move the indicator to the desired reading on the numbered scale before using the device." C. "I will record my personal best reading, which is the average of 3 consecutive peak flow readings." D. I will remember to use the device after taking my fluticasone metered-dose inhaler."

Correct Answers: A. "I will exhale as quickly and forcibly as possible through the mouthpiece of the device to obtain a peak flow reading." The peak flow meter is a handheld device used to measure peak expiratory flow rate (PEFR) and is most helpful for clients with moderate to severe asthma. The device evaluates the degree of airway narrowing by measuring the volume of air that a client can exhale in one breath. Exhaling as quickly and forcibly as possible through the mouthpiece of the device causes the indicator to rise to the volume expired. This action is performed 3 consecutive times, and the highest volume is recorded as the peak flow value. Peak flow values provide key information to guide and evaluate treatment and allow for effective self-management of asthma. Incorrect Answers: [B. "I will move the indicator to the desired reading on the numbered scale before using the device."] The client moves the indicator on the numbered scale to 0 or to the lowest number on the scale before using the device. [C. "I will record my personal best reading, which is the average of 3 consecutive peak flow readings."] The personal best reading is the highest peak flow reading that the client whose asthma is in good control attains and is usually calculated over a 2-week period. [D. I will remember to use the device after taking my fluticasone metered-dose inhaler."] In addition to measuring premedicated PEFR, the peak flow meter is used to evaluate response to a short-acting bronchodilator rescue metered-dose inhaler (MDI). The device is not used to measure response to a corticosteroid MDI. Educational objective:The peak flow meter is used to measure peak expiratory flow rate and is most helpful for clients with moderate to severe asthma. A reading is obtained by exhaling as quickly and forcibly as possible through the mouthpiece of the device.

A nursing diagnosis of "ineffective airway clearance related to pain" is identified for a client who had open abdominal surgery 2 days ago. Which intervention should the nurse implement first? A. Administer prescribed analgesic medication for incisional pain B. Encourage use of incentive spirometer every 2 hours while awake C. Offer an additional pillow to splint the incision while coughing D. Promote increased oral fluid intake

Correct Answers: A. Administer prescribed analgesic medication for incisional pain Postoperative clients are at risk for atelectasis and possibly for pneumonia following surgery as a result of retained secretions. Effective coughing is essential to prevent these complications. The nurse can promote many client actions that will facilitate effective coughing. These include splinting the incision while coughing, changing position every 1-2 hours, ambulating early, using an incentive spirometer, and hydrating adequately to thin the secretions. However, all of these interventions are less effective if the client is in pain. The nurse should instruct the client to request pain medication before the pain becomes intense. Pain relief should be addressed prior to implementing coughing exercises and ambulation. Incorrect Answers [B. Encourage use of incentive spirometer every 2 hours while awake] These are appropriate interventions but will be more effective if pain is managed first. [C. Offer an additional pillow to splint the incision while coughing] These are appropriate interventions but will be more effective if pain is managed first. [D. Promote increased oral fluid intake] These are appropriate interventions but will be more effective if pain is managed first. Educational objective:The nurse should ensure that the postoperative client has effective pain relief before performing coughing exercises.

The client with malignant left pleural effusion undergoes a thoracentesis and 900 mL of excess pleural fluid is removed. Which of these manifestations, if noted on the post-procedure assessment, should the nurse report to the health care provider immediately? A. Asymmetrical chest expansion and decreased breath sounds on the left B. Blood pressure 100/65 mm Hg (mean arterial pressure 77 mm Hg) C. Client complains of 6/10 pain at the needle insertion site D. Respiratory rate 24/min, pulse oximetry 94% on oxygen 2 L/min

Correct Answers: A. Asymmetrical chest expansion and decreased breath sounds on the left A thoracentesis involves the insertion of a large-bore needle through an intercostal space to remove excess fluid. The procedure has the following advantages: 1. Diagnostic - analysis of fluid to diagnose the underlying cause of the pleural effusion (eg, infection, malignancy, heart failure), including cytology, bacterial culture, and related testing 2. Therapeutic - removal of excess fluid (>1 L) improves dyspnea and client comfort Complications from insertion of the needle and removal of large amounts of fluid include iatrogenic pneumothorax, hemothorax, pulmonary edema, and infection. After the procedure, the nurse assesses for pain and difficulty breathing; monitors vital signs and oxygen saturation; and observes for changes in respiratory rate and depth, symmetry of chest expansion, and breath sounds. If any abnormalities are noted, a post-procedure chest x-ray is obtained. Decreased chest expansion with inspiration and breath sounds on the affected side, tachypnea, tracheal deviation to the opposite side, and hyperresonance (air) on the affected side are manifestations of a pneumothorax. These should be reported immediately. Incorrect Answers: [B. Blood pressure 100/65 mm Hg (mean arterial pressure 77 mm Hg)] Hypotension, pulmonary edema, and tachycardia can occur as the result of removal of large amounts of pleural fluid (>1.5 L). This client's blood pressure is adequate (mean arterial pressure 77 mm Hg), and the nurse should continue to monitor. However, this blood pressure does not need to be reported immediately. [C. Client complains of 6/10 pain at the needle insertion site] Mild to moderate pain is common after the procedure. It does not need to be reported immediately. [D. Respiratory rate 24/min, pulse oximetry 94% on oxygen 2 L/min] Difficulty breathing, tachypnea, and hypoxemia are pulmonary complications that can occur after thoracentesis. Saturation (94%) and respiratory rate (24/min) are adequate and do not need to be reported immediately. Educational objective:Complications of thoracentesis include iatrogenic pneumothorax, hemothorax, and infection. Post-procedure, the nurse assesses for pain and difficulty breathing; monitors vital signs and oxygen saturation; and observes for changes in respiratory rate and depth, symmetry of chest expansion, and breath sounds.

The nurse is teaching about the importance of dietary fiber at a community health fair. Which health benefits of consuming a fiber-rich diet should the nurse include in the teaching plan? Select all that apply. A. Helps prevent colorectal cancer B. Improves glycemic control C. Promotes weight loss D. Reduces risk of vascular disease E. Regulates bowel movements

Correct Answers: A. B, C, D, and E Dietary fiber is composed of indigestible complex carbohydrates that absorb and retain water, which increases stool bulk and makes stool softer and easier to pass. Consuming a diet high in fiber-rich foods (eg, fruits, vegetables, legumes, whole grains) improves stool elimination, which helps prevent constipation and decreases the risk of colorectal cancer. Fiber-rich foods tend to have a low glycemic load (less sugar per serving) and are nutrient dense, yet they have lower caloric density. Clients may also experience increased satiety as fiber absorbs water and produces fullness. This may help reduce caloric intake, improve blood glucose control, and promote weight loss. Fiber binds to cholesterol in the intestines, which reduces serum cholesterol levels by decreasing the amount of dietary cholesterol that enters the bloodstream. Decreasing serum cholesterol levels helps reduce vascular plaque buildup and atherosclerosis. A high intake of fiber-rich foods directly correlates with a reduced risk of vascular diseases, including coronary artery disease and stroke. Educational objective:Dietary fiber increases stool bulk and makes stool softer and easier to pass. A fiber-rich diet helps prevent constipation; decreases risk of colorectal cancer; promotes weight loss; improves blood glucose control; and decreases serum cholesterol levels, which reduces the risk of coronary artery disease and stroke.

The nurse is reinforcing teaching to an overweight 54-year-old client about ways to decrease symptoms of obstructive sleep apnea. Which interventions would be most effective? Select all that apply. A. Eating a high-protein snack at bedtime B. Limiting alcohol intake C. Losing weight D. Taking a mild sedative at bedtime E. Taking a nap during the day F. Taking modafinil at bedtime

Correct Answers: B & C Obstructive sleep apnea (OSA) is characterized by partial or complete airway obstruction during sleep that occurs from relaxation of the tongue and soft palate. The result is repeated episodes of apnea (≥10 seconds) and hypopnea (≤50% normal ventilation), which cause hypoxemia (decreased PaO2) and hypercapnia (increased PaCO2). Common symptoms include frequent periods of sleep disturbance, snoring, morning headache, daytime sleepiness, difficulty concentrating, forgetfulness, mood changes, and depression. Interventions for OSA include: — Using a continuous positive airway pressure (CPAP) device at night to keep the tongue from collapsing backward — Limiting alcohol intake at bedtime as it can cause muscles of the oral airway to relax, leading to airway obstruction — Weight loss and exercise may reduce snoring and sleep apnea-associated airway obstruction. Obesity contributes to the development of OSA — Avoiding sedating medications (eg, benzodiazepines, certain antidepressants, antihistamines, opiates) as they may exacerbate OSA and worsen daytime sleepiness Incorrect Answers: [A. Eating a high-protein snack at bedtime] Eating before bedtime can interfere with sleep and contribute to excess weight. [D. Taking a mild sedative at bedtime] Sedatives at bedtime can relax the muscles of the oral airway and lead to airway obstruction. [E. Taking a nap during the day] Napping during the day can make it more difficult to sleep through the night. [F. Taking modafinil at bedtime] Stimulants such as modafinil may be prescribed for daytime sleepiness but should be avoided at bedtime as they can cause insomnia. Educational objective:Obstructive sleep apnea is characterized by partial or complete airway obstruction during sleep. Interventions to relieve symptoms include using a continuous positive airway pressure device during sleep and making lifestyle changes (eg, weight loss; exercise; avoiding food, alcohol, and sedatives at bedtime).

During morning rounds, the nurse notices that a client who was admitted 3 days ago with hepatic encephalopathy is sleepy and confused. The client is scheduled for discharge later today. Which interventions are appropriate for the nurse to implement? Select all that apply. A. Compare current mental status to previous findings B. Encourage the client to ambulate in the hallway C. Hold the client's morning dose of lactulose D. Monitor the client's ammonia level E. Observe the client's hand movements with the arms extended

Correct Answers: B and C Hepatic encephalopathy is a serious complication of end-stage liver disease (ESLD) that results from inadequate detoxification of ammonia from the blood. Symptoms include lethargy, confusion, and slurred speech; coma can occur if this condition remains untreated. Asterixis, described as a flapping tremor of the hands when the arms are extended with the hands facing forward, may also be noted in the client with hepatic encephalopathy. The client with ESLD exhibiting confusion and lethargy should be evaluated for worsening encephalopathy by assessing for asterixis and comparing the current mental status and ammonia level to previous findings. If encephalopathy worsens, medical treatment should include higher doses of lactulose and rifaximin, and discharge should be delayed until the client is stable. Incorrect Answers: [B. Encourage the client to ambulate in the hallway] The client with lethargy and confusion is at risk for falling. Ambulation is not an appropriate intervention at this point. [C. Hold the client's morning dose of lactulose] Lactulose is the primary drug used for treating hepatic encephalopathy. It helps to excrete ammonia through the bowels as soft or loose stools. Lactulose should not be held if the client's hepatic encephalopathy continues to worsen. Educational objective:Hepatic encephalopathy is a serious complication of end-stage liver disease caused by high levels of ammonia in the blood. Assessment findings include confusion, lethargy, and asterixis; coma and death can occur if this condition remains untreated. Pharmacologic treatments include lactulose and antibiotics (eg, rifaximin). The client with worsening encephalopathy is not stable enough for discharge.

The nurse is caring for a client with partial hearing loss. Which interventions would be appropriate to promote effective communication? Select all that apply. A. Dim lights to prevent overstimulation B. Post a hearing impairment sign on the client's door C. Raise voice to speak more loudly D. Speak directly facing the client E. Tell family to take hearing aids home so they will not be lost

Correct Answers: B and D Effective communication is the key to ensuring the safety of clients with hearing impairment. To avoid startling the client, the nurse should approach the client from the front and visibly gain the client's attention before speaking. The nurse should stand directly facing the client so that the speaker's face can be seen clearly. Facial expressions and gestures can help make the meaning clear. If clients communicate with sign language, a professional sign language interpreter should be used when needed. The nurse may post a hearing impairment sign at the head of the bed or on the door to inform all caregivers of the safety concern. Incorrect Answers: [A. Dim lights to prevent overstimulation] Many clients with hearing impairment will lip-read. The room lights should be on so that the speaker's lips and face are well illuminated. [C. Raise voice to speak more loudly] When speaking to a client with hearing loss, speech should be directed toward the least-affected ear and should be at a normal volume. Raising the voice to speak loudly creates a higher pitch that is harder to understand. [E. Tell family to take hearing aids home so they will not be lost] The nurse should ensure that any hearing aids are functional and in place before attempting to speak to the client. Educational objective:When speaking to a client with hearing impairment, the nurse should have the room lights on, directly face the client, speak at a normal volume toward the least-affected ear, and ensure that any hearing aids are functional and in place. Hearing impairment signs may be posted to promote safety.

The nurse is reinforcing information for a client with chronic obstructive pulmonary disease. Which statements by the client indicate an understanding of the pursed-lip breathing technique? Select all that apply. A. "I exhale for 2 seconds through pursed lips." B. "I exhale for 4 seconds through pursed lips." C. "I inhale for 2 seconds through my mouth." D. "I inhale for 2 seconds through my nose, keeping my mouth closed." E. "I inhale for 4 seconds through my nose, keeping my mouth closed."

Correct Answers: B and D The pursed-lip breathing technique helps to decrease shortness of breath by preventing airway collapse, promoting carbon dioxide elimination, and reducing air trapping in clients with chronic obstructive pulmonary disease (COPD). Those with COPD are taught to use this technique when experiencing dyspnea as it increases ventilation and decreases the work of breathing. Regular practice (eg, 5-10 minutes 4 times daily) enables the client to use the technique when short of breath without conscious effort. Clients are taught the following steps: — Relax the neck and shoulders. — Inhale deeply for 2 seconds through the nose with the mouth closed. — Exhale for 4 seconds through pursed lips, as if blowing through a straw. If unable to exhale for this long, practice regularly until you can exhale for twice as long as you inhale. Incorrect Answers: [A. "I exhale for 2 seconds through pursed lips."] Exhalation through pursed lips is done for 4 seconds (or twice as long as inhalation), not 2 seconds. [C. "I inhale for 2 seconds through my mouth."] The client should inhale through the nose, not the mouth. [E. "I inhale for 4 seconds through my nose, keeping my mouth closed."] The client should inhale for 2 seconds, not 4 seconds. Educational objective:The pursed-lip breathing technique helps to decrease shortness of breath by preventing airway collapse, promoting carbon dioxide elimination, and reducing air trapping in clients with chronic obstructive pulmonary disease. Clients are taught to relax the shoulders and neck, inhale through the nose for 2 seconds with the mouth closed, and exhale through pursed lips for 4 seconds (or twice as long as inhalation).

A client is in suspected shock state from major trauma. Which parameters best indicate the adequacy of peripheral perfusion? Select all that apply. A. Apical pulse B. Capillary refill C. Lung sounds D. Pupillary response E. Skin color and temperature

Correct Answers: B and E Shock is a life-threatening syndrome characterized by decreased perfusion and impaired cellular metabolism. A lack of perfusion at both the tissue and cellular level (anaerobic metabolism) occurs due to decreased cardiac output, ineffective blood flow, and inability to meet the body's demand for increased oxygen. Sustained hypoperfusion activates compensatory mechanisms (eg, neural, hormonal, biochemical) to maintain homeostasis and reverse the consequences of anaerobic metabolism. Shock will progress through 4 stages (initial, compensatory, progressive, irreversible). Early identification and intervention help to prevent stage progression. Adequacy of tissue perfusion in a client with shock syndrome and possible organ dysfunction is assessed by the level of consciousness, urine output, capillary refill, peripheral sensation, skin color, extremity temperature, and peripheral pulses. Capillary refill indicates adequacy of blood flow to the peripheral tissues. It is measured by the time taken for color (pink) to return to an external capillary bed (nail bed) after pressure is applied to cause blanching. In an adult, color should return in less than 3 seconds. Normal skin color and temperature are indicators of the adequacy of peripheral blood flow; these are usually within normal limits during the initial and compensatory stages of shock. Incorrect Answers: [A. Apical pulse] Apical pulse is a central pulse and does not indicate adequacy of peripheral tissue perfusion. [C. Lung sounds] Lung sounds indicate the adequacy of ventilation and gas exchange, not peripheral tissue perfusion. [D. Pupillary response] Pupillary response is an indicator of cerebral function, not peripheral tissue perfusion. Educational objective:The adequacy of blood flow to peripheral tissues is determined by measuring capillary refill and assessing skin color and temperature; these are usually within normal limits during the initial and compensatory stages of shock.

The nurse is counseling a client with obesity who is starting a weight reduction diet. The client reports consuming 4-5 regular cola beverages daily. Which of the following beverages should the nurse recommend as healthier substitutes? Select all that apply. A. Commercial fruit juice B. Flavored club soda C. Fresh vegetable juice D. Sports beverages E. Unsweetened tea

Correct Answers: B, C, and D Sugary beverages, such as regular soft drinks, are key contributors to the excess consumption of calories and the obesity epidemic. Individuals who are attempting to lose weight should consume beverages with nutritional value and little-to-no caloric value, including: · Water · Club soda (flavored or unflavored) · Unsweetened tea and/or coffee · Fresh vegetable juice · Nonfat or low-fat milk (in limited amounts) A 12-oz (355-mL) serving in a typical can of regular cola-type beverage contains around 140 calories (kcal). For this client, the consumption of 5 cola beverages daily is contributing 255,500 kcal per year and accounts for 73 lb (33.2 kg) (3500 kcal/lb). This client could lose 73 lb (33.2 kg) in a year simply by substituting zero-calorie beverages for cola. Incorrect Answers: [A. Commercial fruit juice] Commercially available fruit juices and sports beverages often contain relatively high amounts of sugar and calories with little to no nutritional value (ie, "empty calories"), and should be limited or removed from the client's dietary intake. [D. Sports beverages] Commercially available fruit juices and sports beverages often contain relatively high amounts of sugar and calories with little to no nutritional value (ie, "empty calories"), and should be limited or removed from the client's dietary intake. Educational objective:Substituting low- or zero-calorie beverages such as water, unsweetened tea, fresh vegetable juice, and club soda for high-calorie, sugary beverages can have a significant impact on weight loss for clients who have habitually consumed them.

The nurse is caring for a client who had a laparoscopic cholecystectomy 3 days ago. The client's WBC count has increased from 11,200/mm*3 (11.2 × 10*9/L) to 14,600/mm*3 (14.6 × 10*9/L) over the last 24 hours. The nurse understands that which of the following assessment findings indicate potential infection? Select all that apply. A. Client rating left shoulder pain as 4 on a scale of 0-10 B. Greenish-gray drainage noted on surgical dressing C. Productive cough with thick, green sputum D. Stiff abdomen with rebound tenderness on palpation E. Warm, reddened area around the incision site*

Correct Answers: B, C, D, and E Cholecystectomy (removal of the gallbladder) is performed through laparoscopic or open surgery. Signs of postoperative infection typically appear 3-7 days after surgery. Systemic signs may include fever, elevated WBC count, and fatigue. — Pneumonia can occur when atelectasis (alveolar collapse) prevents clearing of secretions, promoting bacterial growth. Symptoms include cough with or without sputum, tachypnea, and shortness of breath. Postoperative incentive spirometry, ambulation, and cough/deep breathing exercises help keep alveoli open and prevent pneumonia — Surgical site infections present with localized redness, warmth, swelling, and purulent drainage. Proper wound care and sterile dressing changes help prevent infection — Urinary tract infections (UTIs), caused by the use of indwelling urinary catheters during surgery, can present with frequency, urgency, and dysuria. Prompt removal of catheters after surgery helps prevent UTIs. — Peritonitis (peritoneal infection) presents with rebound tenderness, boardlike abdominal rigidity, and shallow breathing related to abdominal distension. Peritonitis may lead to sepsis and death if untreated Incorrect Answer [A. Client rating left shoulder pain as 4 on a scale of 0-10] Clients recovering from laparoscopic surgery may experience referred left shoulder pain during the first few postoperative days. This is due to diaphragmatic nerve irritation caused by the carbon dioxide used to inflate the abdomen during laparoscopic surgery Educational objective:Some potential postoperative infections related to abdominal surgery include pneumonia, surgical site infection, and peritonitis. Signs of infection may include cough, tachypnea, and shortness of breath; warmth or redness around the incision; purulent incisional drainage; or rigid, painful abdomen.

The nurse is caring for a client with overflow urinary incontinence related to diabetic neuropathy. Which of the following interventions are appropriate? Select all that apply. A. Decrease fluid intake to 1 glass with each meal and at bedtime B. Encourage the client to bear down while attempting to void C. Inspect the perineal area for evidence of skin breakdown D. Measure postvoid residual volumes as prescribed E. Tell the client to wait 30 seconds after voiding and then attempt to void again

Correct Answers: B, C, D, and E. Overflow urinary incontinence occurs due to compression of the urethra (eg, uterine prolapse, prostate enlargement) or impairment of the bladder muscle (eg, spinal cord injury, diabetic neuropathy, anticholinergic medications). Both types involve incomplete bladder emptying and urinary retention, which lead to overdistension and overfilling of the bladder and frequent involuntary dribbling of urine. When caring for clients with overflow incontinence, the nurse should: — Implement a fixed voiding schedule (eg, every 2 hours) to prevent bladder overfilling. — Instruct the client to use the Valsalva maneuver (ie, "bearing down") and Credé maneuver (ie, gently applying pressure to the lower abdomen) to help facilitate bladder emptying — Assess the perineal area for skin breakdown related to incontinence — Measure postvoid residual volumes as prescribed to ensure that the client is not retaining large amounts of urine — Instruct the client to wait 20-30 seconds after voiding and then attempt to void a second time (ie, double voiding) to help empty residual urine Incorrect Answers: [A. Decrease fluid intake to 1 glass with each meal and at bedtime] Fluid restriction can lead to dehydration with concentrated urine, which irritates the bladder and increases the risk for urinary tract infection. Dehydration also contributes to constipation, which worsens incontinence by compressing the bladder. Educational objective:When caring for clients with overflow incontinence, the nurse should implement a fixed voiding schedule, teach the client techniques that assist with bladder emptying (eg, Valsalva maneuver, Credé maneuver, double voiding), monitor for perineal skin breakdown, and measure postvoid residual volumes as prescribed.

The nurse cares for a client with a pulmonary embolism. Which of the following clinical manifestations would the nurse anticipate? Select all that apply. A. Bradycardia B. Chest pain C. Dyspnea D. Hypoxemia E. Tachypnea F. Tracheal deviation

Correct Answers: B, C, D, and E. Pulmonary embolism (PE) is a potentially life-threatening medical emergency occurring when a blood clot, fat or air embolus, or tissue (eg, tumor) travels via the venous system into the pulmonary circulation and obstructs blood flow into the lung. This prevents deoxygenated blood from reaching the alveoli, which leads to hypoxemia due to impaired gas exchange and cardiac strain due to congested blood flow in the pulmonary arteries. Clinical manifestations of PE range from mild (eg, anxiety, cough) to severe (eg, heart failure, sudden death). However, many clients initially have mild, nonspecific symptoms that are often misdiagnosed and inadequately managed, greatly increasing the likelihood of progression to shock and/or cardiac arrest. Clinical manifestations of PE include: — Pleuritic chest pain (ie, sharp lung pain while inhaling) — Dyspnea and hypoxemia — Tachypnea and cough (eg, dry or productive cough with bloody sputum) — Tachycardia — Unilateral leg swelling, erythema, or tenderness related to deep vein thrombosis Incorrect Answers [A. Bradycardia] Tachycardia, rather than bradycardia, is expected with PE because the heart attempts to compensate for hypoxemia, right ventricular overfilling, and decreased left ventricular cardiac output. [F. Tracheal deviation] Tracheal deviation is a sign of tension pneumothorax (not PE), which occurs when pressure on the side of the collapsed lung pushes organs toward the unaffected lung. Educational objective:Pulmonary embolism is a potentially life-threatening medical emergency occurring when a pulmonary artery is obstructed. Common clinical manifestations include pleuritic chest pain, dyspnea, hypoxemia, tachypnea, cough, tachycardia, and unilateral leg swelling.

The nurse is reinforcing discharge instructions to a client who has had coronary artery bypass grafting. Which teachings are correct? Select all that apply. A. No sexual activity for at least 6 weeks postoperatively B. Notify the health care provider (HCP) of redness, swelling, or drainage at the incision site C. Refrain from lifting objects weighing >5 lb (2.26 kg) until approved by the HCP D. Take a shower daily without soaking chest and leg incisions E. Use lotion on incision sites when changing dressing if the areas are dry

Correct Answers: B, C, and D Discharge instructions for a client recovering from coronary artery bypass grafting should include the following guidelines: 1. Modify cardiac risk factors, including no smoking, losing weight, maintaining a healthy diet, and increasing activity levels through exercise. 2. Use a shower daily to bathe as a bath could introduce microorganisms into the surgical incision sites. Wash incisions gently with mild soap and water and pat dry; incisions should not be soaked or have lotions or creams applied as this could introduce pathogens. 3. May begin light housework in 2 weeks, but do not lift any object weighing >5 lb (2.26 kg) without health care provider (HCP) approval, which is given about 6 weeks after discharge. Lifting, carrying, and pushing heavy objects are isometric activities that rapidly increase heart rate and blood pressure. Gradually resume activity and possibly participate in a cardiac rehabilitation program. 4. Driving should be suspended for 4-6 weeks or until the HCP gives approval. 5. It is usually safe to resume sexual activity once able to walk 1 block or climb 2 flights of stairs without symptoms (eg, chest pain, shortness of breath, fatigue). 6. Notify the HCP if the following symptoms occur: x Chest pain or shortness of breath that does not subside with rest. x Fever >101F (38.3 C) x Redness, drainage, or swelling at the incision sites Educational objective:Discharge teaching for a client recovering from coronary artery bypass grafting should include instructions related to hygiene (showering instead of bathing, no soaking or applying lotions to incisions), medications, activity level (no lifting of objects >5 lb [2.26 kg], no driving for 4-6 weeks) sexual activity (resume when able to walk 1 block or climb 2 flights of stairs without symptoms), and symptoms to report to the health care provider (chest pain or shortness of breath during rest, signs of infection).

The nurse reinforces discharge instructions to a client who was hospitalized for deep venous thrombosis that has now resolved. Which instructions should the nurse include to prevent reoccurrence? Select all that apply. A. Do not travel by car or airplane for at least 3-4 weeks B. Drink plenty of fluids daily and limit caffeine and alcohol intake C. Elevate legs on a footstool when sitting and dorsiflex the feet often D. Resume the walking or swimming exercise program as soon as possible after getting home E. Sit in a cross-legged yoga position for 5-10 minutes as this benefits circulation

Correct Answers: B, C, and D Discharge teaching for a client who had deep venous thrombosis (DVT) emphasizes minimization of risk factors and interventions to promote blood flow and venous return and prevent reoccurrence. Teaching points include the following: — Drink plenty of fluids and limit caffeine and alcohol intake to avoid dehydration, which predisposes to blood hypercoagulability and venous thromboembolis — Elevate legs when sitting, and dorsiflex the feet often to reduce venous hypertension and edema and to promote venous return — Begin or resume a walking/swimming exercise program as soon as possible to promote venous return through contraction of the calf and thigh muscle — Change position frequently to promote venous return and circulation and prevent venous stasis. — Stop smoking to prevent endothelial damage and vasoconstriction as this promotes clotting. — Avoid wearing restrictive clothing (eg, Spanx, tight jeans) that interferes with circulation and promotes clotting. The nurse would suggest consultation with a nutritionist or enrollment in a weight-loss program to overweight/obese clients as excess weight contributes to venous insufficiency and hypertension by compressing large pelvic vessels. Incorrect Answers: [A. Do not travel by car or airplane for at least 3-4 weeks] Traveling does not need to be avoided. During extended travel periods (>4 hours), clients are instructed to use preventive measures (eg, wear knee-high compression stockings, exercise calf and foot muscles every 30 minutes, take frequent breaks and walk briefly every hour, recline in their seats, remove objects around the feet and legs to allow maximal movement, drink ample fluids to avoid dehydration). [E. Sit in a cross-legged yoga position for 5-10 minutes as this benefits circulation] Clients should avoid sitting in any cross-legged position and should never cross the legs at the knees or ankles as this compresses the veins and limits venous return. Educational objective:Discharge teaching for a client who had deep venous thrombosis emphasizes minimization of risk factors (eg, venous stasis, hypercoagulability of blood, endothelial damage) and interventions to promote blood flow and venous return and prevent reoccurrence (eg, adequate fluid intake, frequent position changes, elevation of the legs, regular exercise, smoking cessation).

The nurse is caring for a client following a transsphenoidal hypophysectomy. Which clinical findings would the nurse recognize as signs that the client may be developing diabetes insipidus? Select all that apply. A. Decreased serum sodium B. Excess oral water intake C. High urine output D. Increased serum osmolality E. Increased urine specific gravity

Correct Answers: B, C, and D Transsphenoidal hypophysectomy is the surgical removal of the pituitary gland, an endocrine gland that produces, stores, and excretes hormones (eg, antidiuretic hormone [ADH], growth hormone, adrenocorticotropic hormone). Clients undergoing hypophysectomies are at risk for developing neurogenic diabetes insipidus (DI), a metabolic disorder of low ADH levels. ADH promotes water reabsorption in the kidneys. Therefore, loss of circulating ADH results in massive diuresis of dilute urine. Clinical manifestations associated with DI include: — Decreased urine specific gravity (<1.003) — Elevated serum osmolality (>295 mOsm/kg [295 mmol/kg]) — Hypernatremia (>145 mEq/L [145 mmol/L]) — Hypovolemia and potential hypotension — Polydipsia — Polyuria (2-20 L/day) Educational objective:Diabetes insipidus (DI) is a metabolic disorder of decreased antidiuretic hormone, which is responsible for water retention in the kidneys. DI is often related to a preceding trauma, pituitary tumors, or neurosurgery (eg, hypophysectomy). Clinical manifestations of DI include polyuria, polydipsia, hypernatremia, hypovolemia, increased serum osmolality, and decreased urine specific gravity.

The practical nurse is assisting the registered nurse in preparing the room for a client with new-onset tonic-clonic seizures. It is important to ensure that what equipment is in the room? Select all that apply. A. Oral bite prevention device B. Oxygen delivery system C. Padding on the bed side rails D. Soft arm and leg restraints E. Suction equipment

Correct Answers: B, C, and E Client safety is a priority when caring for those with seizure activity. Protecting the airway and improving oxygenation include turning the client on the side and providing oxygen and oral suctioning as needed. Padding the metal bed side rails provides protection and decreases the potential for injury if the client hits the head or extremities during a seizure. Incorrect Answers: [A. Oral bite prevention device] During a seizure, nothing should be placed in the client's mouth. Placing objects in the mouth can result in injury to the client or health care provider. Maintaining an open airway is important and can be accomplished by turning the client on the side and providing oral suctioning to the inside of the cheeks as necessary. [D. Soft arm and leg restraints] A client should never be restrained during a seizure. Restraints could cause muscle or tissue injury. Educational objective:Turning the client on the side, providing oxygen and suctioning as needed, and padding the side rails or removing objects that are near the client can decrease the risk for injury during a seizure. Restraints should not be used.

A client is admitted to the medical surgical floor with a hemoglobin level of 5.0 g/dL (50 g/L). The nurse should anticipate which findings? Select all that apply. A. Coarse crackles B. Dyspnea C. Pallor D. Respiratory Depression E. Tachycardia

Correct Answers: B, C, and E. A normal hemoglobin level for an adult male is 13.2-17.3 g/dL (132-173 g/L) and female is 11.7-15.5 g/dL (117-155 g/L). A client with severe anemia will have tachycardia, which will maintain cardiac output. The cardiovascular system must increase the heart rate and stroke volume to achieve adequate perfusion. Shortness of breath (dyspnea) may occur due to an insufficient number of red blood cells. The respiratory system must increase the respiratory rate to maintain adequate levels of oxygen and carbon dioxide. Pallor (pale complexion) occurs from reduced blood flow to the skin. Incorrect Answers: [A. Coarse Crackles] Coarse crackles occur with fluid overload but not with anemia. [D. Respiratory Depression] Respiratory depression does not occur with anemia. Respiratory depression may occur post-administration of a narcotic or during oversedation. Educational objective:Cardiac and respiratory drive is increased to maintain cardiac output and oxygenation in the setting of anemia.

A client with ascites due to cirrhosis has increasing shortness of breath and abdominal pain. The practical nurse is assisting the registered nurse in preparing the client for a paracentesis. Which nursing actions should be implemented prior to the procedure? Select all that apply. A. Obtain informed consent for the procedure B. Place the client in high Fowler's position C. Place the client on npo status D. Request that the client empty the bladder E. Take baseline vital signs and weight

Correct Answers: B, D and E Paracentesis is performed to remove excess fluid from the abdominal cavity or to provide a specimen of ascitic fluid for diagnostic testing. Paracentesis is not a permanent solution for resolving ascites and is performed only if the client is experiencing impaired breathing or pain due to ascites. Nursing actions include the following: 1. Teach/reinforce teaching about the procedure, sensations, and expected results 2. Obtain baseline abdominal girth, weight, and vital signs. Instruct the client to void to prevent puncturing the bladder. Place the client in high Fowler's position and remain with the client during the procedure 3. After the procedure, reassess abdominal girth, weight, and vital signs 4. Monitor dressing at puncture site for bleeding Incorrect Answers: [A. Obtain informed consent for the procedure] Informed consent can be obtained only by a health care provider (HCP). The nurse can witness informed consent, verifying that it is given voluntarily, the signature is authentic, and the client appears competent to consent. [C. Place the client on npo status] An npo status is not required for this procedure. Paracentesis is often performed at the bedside or an HCP's office with only a local anesthetic. Educational objective:Paracentesis removes fluid from the abdominal cavity to improve symptoms or provide a specimen for testing. The client should be instructed to void prior to the procedure and be placed in high Fowler's position. Abdominal girth, weight, and vital signs should be recorded before and monitored after paracentesis.

The primary health care provider prescribes a 24-hour urine collection for a client with suspected Cushing syndrome. Which instructions should the nurse reinforce with the client regarding this test? Select all that apply. A. "A continuous urinary catheter must be inserted for this test and the urine will collect in an attached bag." B. "Keep the urine collection container in the refrigerator or a cooled ice chest when it is not in use." C. "Only daytime urine should be collected in the container as cortisol levels are higher in the morning." D. "Record the time the urine collection is started and then empty the bladder into the toilet so that the start time coincides with an empty bladder." E. "You will be given a dark plastic jug containing a powder that absorbs into the urine that you will collect in the jug."

Correct Answers: B, D, and E A 24-hour urine is collected to evaluate Cushing syndrome (a condition that results from chronic increased corticosteroids). The urine is tested for free cortisol, and results >80-120 mcg/24 hr indicate that Cushing syndrome is present. Instructions for collecting a 24-hour urine are as follows: — Use a dark jug containing a special powder (obtained from the lab) to protect the urine from light during collection. The powder helps preserve the urine and adjusts its acidity — Collection of the 24-hour urine should span over exactly 24 hours. It is important to first record the time and empty the bladder into the toilet so that the start time coincides with an empty bladder. At that exact time the next day, the bladder should be emptied for a final time and collected into the jug. All urine between the start time and end time should be collected into the container. The time for each urination between start and end does not need to be recorded — Keep the urine in a refrigerator or a cooled ice chest with the lid tightly screwed on for preservation Educational objective:A 24-hour urine is collected to test for increased cortisol levels when evaluating for Cushing syndrome. The client should be taught to collect the urine in a dark jug issued by the lab, start time and then empty the bladder and discard the 1st urine, and collect all the urine for 24 hours; it is kept in the refrigerator or ice chest with a secure lid. Exactly 24 hours after start time, empty bladder once more into the collection container.

The nurse is inspecting the legs of a client with a suspected lower-extremity deep venous thrombosis. Which of the following clinical manifestations should the nurse expect? Select all that apply. A. Blue, cyanotic toes B. Calf pain C. Dry, shiny, hairless skin D. Lower leg warmth and redness E. Unilateral leg edema

Correct Answers: B, D, and E A deep venous thrombosis (DVT) is a blood clot (ie, thrombus) formed in large veins, generally of the lower extremities. DVTs occur commonly as a result of decreased activity or mobility (eg, prolonged travels, bed rest) or as a complication of hospitalization or surgery. Although clients with a DVT may have no symptoms, typical clinical manifestations include unilateral edema, localized pain (eg, calf pain) or tenderness to touch, warmth, erythema, and occasionally low-grade fever. Recognition of a potential DVT is critical because the thrombus can dislodge from the vessel and cause life-threatening pulmonary embolism. Incorrect Answers: [A. Blue, cyanotic toes] Blue, cyanotic toes is an indicator of impaired arterial blood perfusion to the extremity, which may occur with acute arterial occlusion (eg, arterial embolism) or severely reduced blood flow (eg, vasopressor-induced vasoconstriction, atherosclerosis). [C. Dry, shiny, hairless skin] Dry, shiny, hairless skin are common clinical manifestations of chronic peripheral arterial disease. These characteristic skin alterations occur from long-term impairment of blood flow to the extremity. Educational objective:A deep venous thrombosis (DVT) is a blood clot formed in large veins, typically of the lower extremities, that occurs commonly from decreased activity or mobility. Clinical manifestations of a lower-extremity DVT include unilateral edema, calf pain or tenderness to touch, warmth, erythema, and low-grade fever.

The nurse is caring for a client who underwent a transsphenoidal hypophysectomy to remove a pituitary adenoma. Which intervention(s) should the nurse implement? Select all that apply. A. Encourage coughing frequently to prevent pneumonia B. Inspect the mouth and perform mouth care every 4 hours C. Maintain the head of the bed in a flat position D. Perform frequent neurological checks E. Remind the client to not use a toothbrush for 10 days

Correct Answers: B, D, and E A hypophysectomy is a surgical procedure that involves removal of part of the pituitary gland. A transsphenoidal approach involves insertion of an endoscope between the inner aspect of the upper lip and gingiva, through the sella turcica, bottom of the nose, and sphenoid sinuses to the pituitary gland. There is no external incision. The dural opening is closed with a patch of fat graft taken from the abdomen or outer thigh. As a result, the client should be prepared for an additional incision. Postoperative care focuses primarily on preventing disruption of the patch closure of the dura and cerebrospinal fluid (CSF) leak. This care includes the following: 1. Frequent mouth care with a soft sponge to prevent infection; this includes no use of a toothbrush for at least 10 days to prevent suture line disruption. 2. The nurse should teach the client to avoid coughing, sneezing, or straining. These actions can cause CSF leakage. Any clear nasal drainage should be tested for the presence of CSF, indicated by a glucose level >30 mg/dL in the fluid 3. The nurse should perform frequent neurological checks and report signs of increased intracranial pressure or bleeding to the health care provider. 4. The head of bed should be maintained at a 30-degree angle to decrease intracranial pressure Educational objective:A hypophysectomy using a transsphenoidal approach is a surgical procedure to remove part of the pituitary; it involves the insertion of an endoscope through the inner aspect of the upper lip all the way to the pituitary. Nursing care includes frequent mouth care with a soft sponge to avoid infection and disruption to the suture line, frequent neurological checks, and maintaining the head of the bed at a 30-degree angle.

A client with an asthma exacerbation has been using her albuterol rescue inhaler 10-12 times a day because she cannot take a full breath. What possible side effects of albuterol does the nurse anticipate the client will report? Select all that apply. A. Constipation B. Difficulty sleeping C. Hives with pruritus D. Palpitations E. Tremor

Correct Answers: B, D, and E Albuterol is a short-term beta-adrenergic agonist used as a rescue inhaler to treat reversible airway obstruction associated with asthma. Dosing in an acute asthma exacerbation should not exceed 2-4 puffs every 20 minutes x 3. If albuterol is not effective, an inhaled corticosteroid is indicated to treat the inflammatory component of the disease. Albuterol is a sympathomimetic drug. Expected side effects mimic manifestations related to stimulation of the sympathetic nervous system, and commonly include insomnia, nausea and vomiting, palpitations (from tachycardia), and mild tremor. Incorrect Answers: [A. Constipation] Constipation is not a common side effect of inhaled beta-agonist drugs. [C. Hives with pruritus] Hives can occur as a sign of an allergic reaction and are not a common anticipated side effect of an inhaled beta-agonist drug. Educational objective:Albuterol is a short-term beta-agonist rescue drug used to control symptoms of airway obstruction and promote bronchodilation. It is a sympathomimetic drug; common expected side effects include insomnia, nausea and vomiting, palpitations (tachycardia), and mild tremor.

The client is scheduled to have a cardiac catheterization. Which of the following findings would cause the nurse to question the safety of the test proceeding? Select all that apply. A. Elevated serum C-reactive protein level B. History of previous allergic reaction to IV contrast C. Prolonged PR interval on ECG D. Received metformin today for type 2 diabetes mellitus E. Serum creatinine of 2.5 mg/dL (221 µmol/L)

Correct Answers: B, D, and E Cardiac catheterization involves injection of IV iodinated contrast to assess for obstructed coronary arteries. Potential complications of IV iodinated contrast include: — Allergic reaction: Clients with a previous allergic reaction to iodinated contrast may require premedication (eg, corticosteroids, antihistamines) to prevent reaction or an alternative contrast medium — Lactic acidosis: When administered to clients taking metformin, IV iodinated contrast can cause an accumulation of metformin in the bloodstream, which can result in lactic acidosis. Therefore, health care providers may discontinue metformin 24-48 hours before administration of contrast and restart the medication after 48 hours, when stable renal function is confirmed — Contrast-induced nephropathy: Iodinated contrast can cause acute kidney injury in clients with renal impairment (eg, serum creatinine >1.3 mg/dL [115 µmol/L]). Therefore, clients with renal impairment should not receive iodinated contrast unless absolutely necessary. Incorrect Answers: [A. Elevated serum C-reactive protein level] C-reactive protein, produced during acute inflammation, may indicate elevated risk for coronary artery disease. However, it is not an indicator of an acute cardiac event and is not a safety concern for cardiac catherization. [C. Prolonged PR interval on ECG] First-degree atrioventricular block may precede more serious conditions. However, clients are usually asymptomatic and do not require treatment except for stopping the causative medication (eg, beta blocker, digoxin). This would not prevent the procedure from proceeding. Educational objective:Cardiac catheterization uses IV iodinated contrast to assess for obstructed coronary arteries. IV iodinated contrast is avoided in clients who had a previous allergic reaction to contrast agents; have renal impairment; or, in some cases, who recently received metformin.

The nurse is reinforcing teaching about constipation prevention to a client. Which of the following client statements indicate appropriate understanding of the teaching? Select all that apply. A. "Drinking more caffeinated drinks such as tea and soda helps to stimulate the bowel." B. "Having a routine for bowel movements is important, but I should not wait if I feel the urge." C. "I can use an over-the-counter laxative every other day if needed." D. "I should try to eat more fruits and vegetables every day." E. "Increasing my daily exercise level may help keep my bowel movements regular."

Correct Answers: B, D, and E Constipation is a symptom of many disease processes (eg, Parkinson disease, diabetic neuropathy, depression), procedures (eg, abdominal surgery, bowel manipulation), and medications (eg, anticholinergics, diuretics, opioids). Immobility, low-fiber diets, decreased fluid intake, and irregular bowel habits increase the likelihood of constipation. Educate clients to prevent constipation by: · Encouraging a healthy bowel regimen (eg, avoid delaying defecation if the urge is felt, defecate at the same time daily when possible, track bowel movements to identify changes in patterns) · Increasing consumption of fruits and vegetables to reach a daily fiber intake of at least 20 g (unless contraindicated) because fiber softens and increases the bulk of stool, which promotes defecation · Increasing daily exercise because activity stimulates peristalsis and promotes defecation · Drinking 2-3 L of noncaffeinated fluids daily (unless contraindicated), which prevents drying and hardening of stool in the colon Incorrect Answers: [A. "Drinking more caffeinated drinks such as tea and soda helps to stimulate the bowel."] Clients should avoid caffeinated beverages, which promote diuresis and dehydration and may lead to constipation. [C. "I can use an over-the-counter laxative every other day if needed."] Clients should avoid using laxatives and enemas unless prescribed by a health care provider because overuse can result in physical and psychological dependency. Educational objective:Constipation is a symptom of many disease processes, procedures, and medications. To prevent constipation, educate the client to increase daily fiber intake, drink 2-3 L of fluids daily, increase daily activity levels, and initiate a bowel regimen (eg, avoiding delay of defecation, defecating at the same time each day).

The intensive care nurse is caring for a client who has just been extubated. Which interventions are appropriate at this time? Select all that apply. A. Administer prescribed oral narcotics for throat pain B. Administer warmed, humidified oxygen via facemask C. Give the client ice chips to moisten the mouth D. Provide mouth care with oral sponges E. Start the client on incentive spirometer

Correct Answers: B, D, and E Recently extubated clients are at high risk for aspiration, airway obstruction (laryngeal edema and/or spasm), and respiratory distress. To prevent complications, clients are placed in high Fowler position to maximize lung expansion and prevent aspiration of secretions. Warmed, humidified oxygen is administered immediately after extubation to provide high concentrations of supplemental oxygen without drying out the mucosa. Oral care is provided to decrease bacteria and contaminants as well as promote comfort. Clients are instructed to frequently cough, deep breathe, and use an incentive spirometer to expand alveoli and prevent atelectasis. Incorrect Answers: [A. Administer prescribed oral narcotics for throat pain] Clients are kept NPO after extubation to prevent aspiration. They may have either a bedside swallow screen or a more formal swallow evaluation by a speech therapist prior to swallowing any food, drink, or medication. [C. Give the client ice chips to moisten the mouth] Clients are kept NPO after extubation to prevent aspiration. They may have either a bedside swallow screen or a more formal swallow evaluation by a speech therapist prior to swallowing any food, drink, or medication. Educational objective:Recently extubated clients are immediately placed on humidified oxygen and monitored for aspiration, airway obstruction, and respiratory distress. Clients should remain NPO until swallowing function has been evaluated. In addition, clients should be given routine oral care as well as instructions on coughing, deep breathing, and use of incentive spirometry.

The nurse cares for a client admitted to the hospital due to confusion. The client has a nonmetastatic lung mass and a diagnosis of syndrome of inappropriate antidiuretic hormone (SIADH). Which action(s) should the nurse expect to implement? Select all that apply. A. Fluid bolus (normal saline) B. Fluid restriction C. Salt restriction in the diet D. Seizure precautions E. Strict record of fluid intake and output

Correct Answers: B, D, and E SIADH is an endocrine condition in which antidiuretic hormone overproduction leads to water retention, increased total body water, and dilutional hyponatremia (low serum sodium). Hyponatremia can cause confusion, seizures, or other neurologic complications. It is important for the nurse to anticipate these problems and institute seizure precautions. SIADH treatment includes: 1. Fluid restriction to <1000 mL/day 2. Oral salt tablets to increase serum sodium 3. Hypertonic saline (3%) during the first few hours for clients with markedly decreased serum sodium and severe neurologic manifestations 4. Vasopressin receptor antagonists (eg, conivaptan) The nurse should also maintain a strict fluid intake and output chart and daily weights and carefully monitor neurologic status to evaluate for improvement or deterioration. Incorrect Answer: [A. Fluid bolus (normal saline)] Normal saline fluid bolus would worsen the hyponatremia as the client already has excess fluid volume. Symptoms are caused by a low sodium level. If the sodium level must be raised, the client will need hypertonic (3%) saline or salt tablets as these contain mainly sodium and little free fluid. [C. Salt restriction in the diet] Oral salt tablets would be administered to increase serum sodium Educational objective:SIADH can occur due to lung cancer and is characterized by water retention, increased total body water, and dilutional hyponatremia. Hyponatremia may cause neurologic complications (eg, confusion, seizures). SIADH treatment includes fluid restriction, oral salt tablets, and administration of 3% saline IV and/or vasopressin receptor antagonists.

The nurse is caring for a client with cirrhosis. Assessment findings include ascites, peripheral edema, shortness of breath, fatigue, and generalized discomfort. Which interventions would be appropriate for the nurse to implement to promote the client's comfort? Select all that apply. A. Encourage adequate sodium intake B. Place client in semi-Fowler position C. Place client in Trendelenburg position D. Provide alternating air pressure mattress E. Use music to provide a distraction

Correct Answers: B, D, and E In a client with cirrhosis and ascites, discomfort is often due to pressure of the fluid on the surrounding organs. Shortness of breath occurs due to the upward pressure exerted by the abdominal ascites on the diaphragm, which restricts lung expansion. Positioning the client in semi-Fowler or Fowler position can promote comfort, as this position can reduce the pressure on the diaphragm. In semi-Fowler position, the head of the bed is elevated 30-45 degrees; in Fowler position, elevation is 45-60 degrees. Side-lying with the head elevated can also be a position of comfort for the client with ascites as it allows the heavy, enlarged abdomen to rest on the bed, reducing pressure on internal organs and allowing for relaxation. Meticulous skin care is a priority due to the increased susceptibility of skin breakdown from edema, ascites, and pruritus. It is important to use a specialty mattress and implement a turning schedule of every 2 hours. A distraction can take the client's mind off the current symptoms and may also help promote comfort in many different situations. Some of these distractions include listening to music, watching television, playing video games, or taking part in hobbies. Incorrect Answers: [A. Encourage adequate sodium intake] This client has ascites and peripheral edema; higher levels of fluid or sodium intake can worsen these conditions. [C. Place client in Trendelenburg position] In Trendelenburg position, the bed is tilted with the head lower than the legs. This position is contraindicated in the client with ascites, as it may exacerbate shortness of breath by causing the abdominal ascites to push upward on the diaphragm, restricting lung expansion. Educational objective:The client with discomfort and shortness of breath due to ascites should be positioned in the semi-Fowler or Fowler position to promote comfort and lung expansion. Music and other methods of distraction may also promote comfort. Meticulous skin interventions (eg, specialty mattress, turning schedule) are important to prevent tissue breakdown.

A client is being discharged after receiving an implantable cardioverter defibrillator. Which statement by the client indicates that teaching has been effective? A. "I'm not worried about the device firing now because I know it won't hurt." B. "I will let my daughter fix my hair until my health care provider says I can do it." C. "I will look into public transportation because I won't be able to drive again." D. "I will notify my travel agent that I can no longer travel by plane."

Correct Answers: B. "I will let my daughter fix my hair until my health care provider says I can do it." An implantable cardioverter defibrillator (ICD) can sense and defibrillate life-threatening dysrhythmias. It also includes pacemaker capabilities such as overdrive pacing for rapid heart rhythms or back-up pacing for bradycardias that may occur after defibrillation. The ICD consists of a lead system placed into the endocardium via the subclavian vein. The pulse generator is implanted subcutaneously over the pectoral muscle. Postoperative care and teaching are similar to those for pacemaker implantation. Clients are instructed to refrain from lifting the affected arm above the shoulder (until approved by the health care provider) to prevent dislodgement of the lead wire on the endocardium. Incorrect Answers: [A. "I'm not worried about the device firing now because I know it won't hurt."] Firing of the ICD may be painful. Clients have described the feeling as a blow to the chest. [C. "I will look into public transportation because I won't be able to drive again."] Driving may be approved by the health care provider after healing has occurred. Long-term decisions are based on the ongoing presence of dysrhythmias, frequency of ICD firings, and state laws regarding drivers with ICDs. [D. "I will notify my travel agent that I can no longer travel by plane."] Travel is not restricted. The ICD may set off the metal detector in security areas. A hand-held wand may be used but should not be held directly over the ICD. The client should carry the ICD identification card and a list of medications while traveling. Educational objective:After placement of an implantable cardioverter defibrillator, clients are instructed to avoid lifting the arm on the side of the ICD above the shoulder (until cleared by the health care provider) to avoid dislodging the lead wire system.

The nurse is caring for a client with acute pericarditis. Which clinical finding would require immediate intervention by the nurse? A. Client reports chest pain that is worse with deep inspiration B. Distant heart tones and jugular venous distension C. ECG showing ST-segment elevations in all leads D. Pericardial friction rub auscultated at the left sternal border

Correct Answers: B. Distant heart tones and jugular venous distension Acute pericarditis is inflammation of the membranous sac (pericardium) surrounding the exterior of the heart, which can cause an increase in the amount of fluid in the pericardium (ie, pericardial effusion). Increased pericardial fluid places pressure on the heart, which impairs the heart's ability to contract and eject blood. This complication (ie, cardiac tamponade) is life-threatening without immediate intervention. When assessing clients with pericarditis, it is critical for the nurse to observe for signs of cardiac tamponade (eg, muffled or distant heart tones, hypotension, jugular venous distension). Development of cardiac tamponade requires emergency pericardiocentesis (ie, needle insertion into the pericardium to remove fluid) to prevent cardiac arrest. Incorrect Answers: [A. Client reports chest pain that is worse with deep inspiration] In acute pericarditis, the inflamed pericardium rubs against the heart, causing pain that often worsens with deep breathing or when positioned supine. The client should be placed in the Fowler position with a support (eg, bedside table) to lean on for comfort. [C. ECG showing ST-segment elevations in all leads] ST-segment elevation in almost all ECG leads is a characteristic of acute pericarditis that typically resolves as pericardial inflammation decreases. This is in contrast to acute myocardial infarction, in which ST-segment elevation is seen in only localized leads (depending on which vessel is occluded). [D. Pericardial friction rub auscultated at the left sternal border] Pericardial friction rub is an expected finding with acute pericarditis that occurs from the layers of the pericardium rubbing together to create a characteristic high-pitched, leathery, and grating sound. Educational objective:Nurses caring for clients with pericarditis should monitor for, and immediately report, signs of cardiac tamponade (eg, jugular venous distension, distant heart sounds, hypotension), a life-threatening complication occurring from increased pericardial fluid volume.

An elderly client reports shortness of breath with activity for the past 2 weeks. The nurse reviews the admission laboratory results and identifies which value as the most likely cause of the client's symptoms? A. Brain natriuretic peptide 70 pg/mL (70 pmol/L) B. Hematocrit 21% (0.21) C. Leukocytes 3,500/mm3 (3.5 x 109/L) D. Platelets 105,000/mm3 (105 x 109/L)

Correct Answers: B. Hematocrit 21% (0.21) Hematocrit (Hct) is the percentage of red blood cells (RBCs) in a volume of whole blood. Hct and hemoglobin (Hgb) values are related (approximately 3 x Hgb = Hct); when one value is decreased, the other is also. This client likely has hemoglobin of 7 g/dL (70 g/L) (normal, 13.2-17.3 g/dL [132-173 g/L] for males and 11.7-15.5 g/dL [117-155 g/L] for females). Hgb is a component of the RBC that carries oxygen to the body's tissues. A decrease in Hgb decreases oxygen-carrying capacity and transport to tissues. RBCs may be 100% saturated with oxygen at rest, but desaturation may occur with increased activity and oxygen demand in the presence of decreased Hct and Hgb. Manifestations associated with decreased oxygen transport include shortness of breath with activity, tachypnea, and tachycardia. Incorrect Answers: [A. Brain natriuretic peptide 70 pg/mL (70 pmol/L)] Brain natriuretic peptide (BNP) >100 pg/mL (100 pmol/L) is considered elevated and indicates ventricular stretch (heart failure) as the cause of the dyspnea. This client has normal BNP levels, making heart failure an unlikely cause. [C. Leukocytes 3,500/mm3 (3.5 x 109/L)] The leukocyte count is decreased (normal, 4,000-11,000/mm3 [4.0-11.0 x 109/L]). Leukocytes play a role in protecting the body from disease. [D. Platelets 105,000/mm3 (105 x 109/L)] The platelet count is decreased (normal, 150,000-400,000/mm3 [150-400 x 109/L]). Platelets play a role in blood clotting. Educational objective:Hemoglobin is a component of red blood cells that carries oxygen to the body's tissues. In the presence of decreased hematocrit and hemoglobin, decreased oxygen-carrying capacity and transport occur. Manifestations associated with decreased oxygen transport include shortness of breath with activity, tachypnea, and tachycardia.

When monitoring a client with cholelithiasis and acute cholecystitis, which findings might the nurse note during the health history and physical examination? Select all that apply. A. Flank pain radiating to the groin B. Ingestion of high-protein food before onset of pain C. Low-grade fever with chills D. Pain at the umbilicus E. Right upper-quadrant pain radiating to the right shoulder

Correct Answers: C and E Cardinal symptoms of acute cholecystitis from cholelithiasis include pain in the right upper quadrant (RUQ) with referred pain to the right shoulder and scapula (Option 5). Clients often report fatty food ingestion 1-3 hours before initial onset of pain. Associated symptoms include low-grade fever, chills, nausea, vomiting, and anorexia. During an acute attack of cholecystitis, inflammation of the mucous lining and gallbladder wall occurs as a result of gallstone(s) obstruction of the cystic bile duct. The inflammation and increased pressure in the gallbladder from the blocked bile duct result in Murphy's sign; palpation over the RUQ causes pain and inability to inhale deeply. Laboratory results show leukocytosis. Incorrect Answers: [A. Flank pain radiating to the groin] Flank pain radiating to the groin is seen with renal colic (ureteral stones). [B. Ingestion of high-protein food before onset of pain] Ingestion of high-fat foods (eg, cheese, avocado, fried foods, hamburger), not dietary protein, signals the gallbladder to contract and empty bile into the duodenum to help digestion. Gallstones, which normally float harmlessly around the gallbladder, are then squeezed into the bile duct, causing the pain of biliary colic. Gallstones stuck further down the bile duct (choledocholithiasis) may become colonized by bacteria. [D. Pain at the umbilicus] Initial onset of pain at the umbilicus is seen with acute appendicitis. Educational objective:Cardinal symptoms of acute calculous cholecystitis include pain in the right upper quadrant and referred pain to the right shoulder and scapula a few hours after eating high-fat foods. Associated symptoms include low-grade fever, chills, nausea, vomiting, and anorexia.

The nurse reviews the admission history of a 70-year-old client diagnosed with chronic obstructive pulmonary disease (COPD). Which of the following statements by the client does the nurse recognize as contributing to the development of COPD? Select all that apply. A. "I have been drinking alcohol almost daily since age 20." B. "I have been overweight for as long as I can remember." C. "I have smoked about a pack of cigarettes a day since I was 16 years old but quit last year." D. "I know I eat too much fast food." E. "I was a car mechanic for about 40 years and had my own garage."

Correct Answers: C and E Etiologies associated with the development of COPD Factors [Tobacco smoke (active & passive) — Carbon monoxide decreases O2 - carrying capacity & increases O2 demand — Causes hyperplasia & increased mucus production — Causes alveolar wall destruction — Decreases ciliary action — Related to 80% - 90$ of COPD deaths Occupational exposure to chemicals & dust (carbon monoxide, coal, biomass fuels) — Prolonged exposure causes inflammation of airways Air pollution — Inflammation of airways Genetics (a1-antitrypsin deficiency) — Premature emphysema — Affects <5% Chronic obstructive pulmonary disease (COPD) generally refers to two conditions: emphysema and chronic bronchitis. A combination of the two is common. COPD affects approximately 12 million people and is the third leading cause of death in the United States, occurring most commonly in the seventh decade of life. It is characterized by slowly progressive, persistent airflow obstruction, and its etiology is closely associated with chronic airway inflammation. The major risk factor for COPD is current or former tobacco smoking (eg, cigarette, pipe, cigar). An additional contributing factor is prolonged exposure to respiratory irritants (eg, chemical fumes, smoke, dust) related to the client's occupation (eg, car mechanic, firefighter, coal miner). Risk for COPD is even higher if the client both smokes tobacco and has occupational exposure to respiratory irritants. Chronic exposure to air pollution and genetic predisposition (eg, alpha1-antitrypsin deficiency) also contribute. Incorrect Answers: [A. "I have been drinking alcohol almost daily since age 20."] Alcohol use is not associated with the development of COPD. [B. "I have been overweight for as long as I can remember."] Although obesity can worsen COPD symptoms by contributing to dyspnea, obesity and poor nutrition are not factors that directly contribute to the development of COPD. [D. "I know I eat too much fast food."] Although obesity can worsen COPD symptoms by contributing to dyspnea, obesity and poor nutrition are not factors that directly contribute to the development of COPD. Educational objective:Chronic airway inflammation is closely associated with the development of chronic obstructive pulmonary disease. Specific etiologic factors include current or former tobacco smoking, prolonged exposure to occupational respiratory irritants, chronic exposure to air pollution, and genetic predisposition.

The nurse reinforces teaching to a client who was newly prescribed levothyroxine sodium after thyroid removal. Which instructions will the nurse include? Select all that apply. A. Drowsiness is a common side effect; taking the dose at bedtime will make this less noticeable B. Notify the health care provider if you become pregnant as the medication is harmful to the fetus C. Notify the health care provider if you feel a fluttering or rapid heartbeat D. Take the medication with a meal to prevent stomach upset E. You will need to take this medication for the rest of your life

Correct Answers: C and E Levothyroxine sodium (eg, Levoxyl, Levothroid, Synthroid) is used to replace thyroid hormone in clients with hypothyroidism (inadequate thyroid hormone) and for those who have had their thyroid removed. These clients must understand that this medication must be taken for the rest of their lives. A client's dose is adjusted based on serum thyroid-stimulating hormone (TSH) levels to prevent too much or too little hormone. Clients must be taught to report signs of excess thyroid hormone such as heart palpitations/tachycardia, weight loss, and insomnia. Incorrect Answers: [A. Drowsiness is a common side effect; taking the dose at bedtime will make this less noticeable] Clients with hypothyroidism experience lethargy and somnolence. Hormone replacement therapy will increase metabolic activity and alertness. [B. Notify the health care provider if you become pregnant as the medication is harmful to the fetus] This medication is a hormone that is normally present in the body, so it is safe to take during pregnancy. The dose may need to be altered due to the metabolic demands of pregnancy, but the drug will not harm the fetus. [D. Take the medication with a meal to prevent stomach upset] It is best to take this medication first thing in the morning as it is best absorbed on an empty stomach (1 hour before or 2 hours after a meal). Educational objective:Clients receiving thyroid hormone replacement therapy (levothyroxine sodium) should understand that treatment is lifelong and be taught the signs of excess hormone (eg, tachycardia/palpitations, weight loss, insomnia). The medication is best absorbed on an empty stomach and is safe to take during pregnancy.

A nurse is assisting with preventive health screenings at a community health event. Which of the following client statements should the nurse recognize as a warning sign of cancer? Select all that apply. A. "For the past few years, I get a productive cough in the winter that goes away in the spring." B. "I occasionally have heartburn an hour after I eat fried foods and sausage." C. "Last month when I was doing my breast self-examination, I noticed a marble-sized lump." D. "My mole is itchy, and the edges have become uneven with a blackish to bluish color." E. "Recently I have noticed that my bowel movements appear black."

Correct Answers: C, D, and E Cancer is a growth of abnormal cells in an organ system and may impair the organ's function and spread throughout the body. Many cancers are invasive and life threatening if allowed to reach late stages of development. However, cancer is often difficult to identify early because the client may be asymptomatic or have only vague symptoms. Nurses should screen clients for and immediately report warning signs of cancer, which can be remembered with the mnemonic CAUTION: — Change in bowel or bladder habits — A sore that does not heal — Unusual bleeding or discharge from a body orifice — Thickening or a lump in the breast or elsewhere — Indigestion or difficulty in swallowing that does not go away — Obvious change in a wart or mole — Nagging cough or hoarseness Incorrect Answers: [A. "For the past few years, I get a productive cough in the winter that goes away in the spring."] A productive cough that is annual and seasonal, particularly occurring in the winter, may indicate chronic bronchitis. The nagging cough found in clients with lung cancer is persistent, rather than seasonal. [B. "I occasionally have heartburn an hour after I eat fried foods and sausage."] A client report of occasional indigestion after specific triggers (eg, high-fat or spicy food, caffeine) may indicate gastroesophageal reflux disease. However, indigestion that is persistent or chronic may indicate cancer. Educational objective:Warning signs of cancer for nurses to monitor include change in bowel or bladder habits, a sore that does not heal, unusual bleeding or discharge, thickening or a lump in the breast or elsewhere, indigestion or difficulty swallowing, any obvious change in a wart or mole, and nagging cough or hoarseness (mnemonic: CAUTION).

The nurse is reinforcing general skin care guidelines to a client receiving teletherapy (external beam radiation therapy). Which statements does the client make that indicate proper understanding of the information? Select all that apply. A. "I may apply an ice pack to the treatment site if my skin begins to burn." B. "I will rub baby oil after each treatment to prevent dry skin." C. "I will use extra measures to protect my skin from sun exposure." D. "I will wash the treatment site with lukewarm water and mild soap." E. "I will wear soft, loose-fitting clothing."

Correct Answers: C, D, and E Clients receiving teletherapy (external beam radiation therapy) often experience significant effects to the skin of the treatment area. Teaching essential skin care standards to these clients is focused on preventing infection and promoting healing of the affected skin. Key measures of skin care that clients receiving teletherapy should implement include: — Protect the skin from infection by not rubbing, scratching, or scrubbing x Wear soft, loose-fitting clothing x Use soft, cotton bedsheets and towels x Pat skin dry after bathing x Avoid applying bandages or tape to the treatment area — Cleanse the skin daily by taking a lukewarm shower x Use mild soap without fragrance or deodorant x Do not wash off any radiation ink markings — Use only creams or lotions approved by the health care provider (HCP) x Avoid over-the-counter creams, oils, ointments, or powders unless specifically recommended by the HCP as they can worsen any irritation — Shield the skin from the effects of the sun during and after treatment x Avoid tanning beds and sunbathing x Wear a broad-brimmed hat, long sleeves, and long pants when outside x Use a sunscreen that is SPF 30 or higher — Avoid extremes in skin temperature x Avoid heating pads and ice packs x Maintain a cool, humid environment for comfort Educational objective:The client receiving teletherapy is taught measures to implement to protect the skin from infection and promote healing. Recommended skin care measures include taking a lukewarm shower daily, avoiding rubbing or scratching the skin, using only approved lotions, shielding the skin from the effects of the sun, and avoiding extremes in temperature.

The nurse is caring for a client diagnosed with ulcerative colitis and prescribed sulfasalazine. Which instructions should be reinforced at discharge? Select all that apply. A. Avoid small, frequent meals B. Consume a cup of coffee with each meal if desired C. Continue medication even after resolution of symptoms D. Eat a low-residue, high-protein, high-calorie diet E. Increase fluid intake to at least 2000 mL/day

Correct Answers: C, D, and E Ulcerative colitis is characterized by chronic inflammation and ulcerations in the large intestines, resulting in bloody diarrhea and decreased nutrient absorption. A low-residue, high-protein, high-calorie diet, along with daily vitamin and mineral supplements, is encouraged to meet the client's nutritional and metabolic needs. The low-residue diet limits trauma to the inflamed colon and may lessen symptoms. Easily digested foods such as enriched breads, rice, pastas, cooked vegetables, canned fruits, and tender meats are included in the diet. Raw fruits and vegetables, whole grains, highly seasoned foods, and fried foods are avoided. Clients should also drink at least 2000-3000 mL/day of fluid to maintain fluid and electrolyte balance and hydration. Sulfasalazine is a 5-aminosalicylate used to decrease inflammation in the intestines. To prevent relapse, the medication should be continued even when symptoms subside. Because sulfasalazine hinders the absorption of folate, folic acid supplements are encouraged. Incorrect Answers: [A. Avoid small, frequent meals] Small, frequent meals are encouraged to decrease the amount of fecal material in the gastrointestinal tract and to decrease stimulation. [B. Consume a cup of coffee with each meal if desired] Caffeine, alcohol, and tobacco are gastric irritants that stimulate the intestines and should be avoided. Educational objective:Dietary management of ulcerative colitis includes eating small, frequent meals; following a low-residue, high-protein, high-calorie diet; taking supplemental vitamins and minerals; avoiding caffeine, alcohol, and tobacco; and drinking at least 2000-3000 mL/day of fluid. Continued use of sulfasalazine prevents relapse and prolongs symptom remission.

A client with chronic kidney disease has a subcutaneous arteriovenous fistula (AVF) placed in the nondominant left wrist for hemodialysis. Which of the following statements indicate the client understands how to care for the fistula properly? Select all that apply. A. "I don't need to call my health care provider (HCP) if I have numbness or tingling in my left arm." B. "I don't need to call my health care provider (HCP) if I have numbness or tingling in my left arm." C. "I will squeeze a small sponge with my left hand several times a day." D. "I will touch the site and feel for a vibration several times a day." E. "I will try not to sleep on my left arm."

Correct Answers: C, D, and E. The creation of an AVF for hemodialysis access involves an anastomosis between an artery and a vein (usually the cephalic or basilic vein). The fistula permits the arterial blood to flow through the vein, causing the vein to become larger in diameter and the walls to thicken, enabling blood to flow at high pressures. After the AVF is placed, it takes 2-4 months for it to mature to accommodate the repeated venipunctures necessary for hemodialysis access. The major complications of an AVF are infection (especially in end-stage kidney disease and diabetes), stenosis, thrombosis, and hemorrhage. Clients are taught the following preventive interventions: — Report numbness or tingling of the extremity to the HCP to prevent neuromuscular damage — Do not allow anyone (other than dialysis personnel) to draw blood or take blood pressure measurements on the extremity to prevent thrombosis — Avoid wearing restrictive clothing or jewelry to prevent thrombosis — Do not use the arm with vascular access to carry heavy objects (more than 5 lb [2.26 kg]); however, exercises to increase strength could include squeezing a soft ball or sponge several times a day — Check the function of the vascular access several times a day by feeling for vibration to assess for patency, stenosis, and clotting — Do not sleep on the arm with vascular access or use creams or lotions on the site — Monitor for signs of infection and bleeding after dialysis and report immediately — Keep the site clean to help prevent infection Educational objective:The creation of an AVF for hemodialysis access involves an anastomosis between an artery and a vein. Clients are taught interventions to help prevent the major complications associated with an AVF (infection, stenosis, thrombosis, and hemorrhage).

The practical nurse is assisting the registered nurse in conducting client intake histories at a family practice clinic. Which client findings or histories indicate a need for heightened concern that the client may have cancer? Select all that apply. A. 60-year-old client was just diagnosed with benign prostatic hyperplasia B. Client reports a doughy-feeling, mobile, golf ball-size lesion under the skin over the right thigh C. Client reports a nagging cough with hoarseness for the past 3 months D. Female client weighed 150 lb (68.0 kg) and lost 15 lb (6.8 kg) in 3 months without dieting E. Male client reports a skin change on the breast that looks like an orange peel

Correct Answers: C, D, and E. The warning signs of cancer can be remembered with the acronym CAUTION: —Change in bowel or bladder habits —A sore that does not heal —Unusual bleeding or discharge from a body orifice —Thickening or a lump in the breast or elsewhere —Indigestion or difficulty in swallowing —Obvious change in a wart or mole —Nagging cough or hoarseness Unintentional weight loss of >10% of usual weight (in nonobese clients) requires evaluation, and it can indicate underlying cancer. Nausea, anorexia, and dysgeusia (altered taste sensation) are also clinical features of cancer that contribute to weight loss Although 99% of breast cancers are found in women, men can also develop breast cancer, especially if risk factors such as past chest radiation are present. Later signs of breast cancer include a newly retracted nipple or an orange-peel appearance of the breast tissue (peau d'orange) caused by plugging of dermal lymph drainage Incorrect Answers: [A. 60-year-old client was just diagnosed with benign prostatic hyperplasia] Benign prostatic hyperplasia is caused by hormonal changes related to aging. It is not an indication of cancer. [B. Client reports a doughy-feeling, mobile, golf ball-size lesion under the skin over the right thigh] Lipomas are benign, fatty masses and rarely become malignant. They are subcutaneous, have a soft doughy feel, and are mobile and asymptomatic. Masses that are hard and fixed, not soft and mobile, usually indicate malignancy. Educational objective:Signs of potential cancer include unplanned weight loss, nagging cough/hoarseness, and dimpled skin (orange peel) on the breast. Hard, fixed masses; nonhealing ulcers; and changing moles may also indicate malignancy and require further workup.

An elderly client is admitted with an acute exacerbation of chronic obstructive pulmonary disease. Pulse oximetry is 84% on room air. The client is restless, has expiratory wheezing and a productive cough, and is using the accessory muscles to breathe. Which prescription should the nurse question? A. Albuterol 2.5 mg by nebulizer B. IV methylprednisolone 125 mg now and every 6 hours C. IV morphine 2 mg now, may repeat every 2 hours D. Oxygen at 2 L/min by nasal cannula

Correct Answers: C. IV morphine 2 mg now, may repeat every 2 hours This client is experiencing an acute exacerbation of chronic obstructive pulmonary disease (COPD). Restlessness is an early, subtle sign of hypoxemia. Although morphine may be used to decrease restlessness and oxygen demand in some clients, it can depress respirations. Morphine and other medications (eg, benzodiazepines) that depress the respiratory center should not be used in clients with COPD exacerbation as they can further worsen CO2 retention. The primary treatment goal for clients with an acute COPD exacerbation is to increase oxygen saturation and decrease restlessness, pulse, and respiratory rate. Incorrect Answers: [A. Albuterol 2.5 mg by nebulizer] Albuterol is a beta agonist that is appropriate for the immediate relief of bronchoconstriction due to its rapid, short action. [B. IV methylprednisolone 125 mg now and every 6 hours] Methylprednisolone (Solu-Medrol) is a corticosteroid that is appropriate for decreasing inflammation of the lungs during an acute COPD exacerbation. [D. Oxygen at 2 L/min by nasal cannula] Initiation of low-flow, low-concentration oxygen at 2 L/min by nasal cannula is appropriate in a client with COPD. Many clients with this disease rely on their hypoxemic drive to breathe; therefore, it is best to start oxygen at a lower concentration and titrate upward if the saturation does not reach 90% within 20-30 minutes. Educational objective:Morphine and other medications (eg, benzodiazepines) that can depress the respiratory center should not be used in clients with an acute exacerbation of chronic obstructive pulmonary disease as these can further worsen CO2 retention.

The nurse is monitoring a client with suspected pulmonary tuberculosis. Which characteristic signs and symptoms does the nurse expect? Select all that apply. A. Dysuria B. Jaundice C. Low back pain D. Night sweats E. Purulent or blood-tinged sputum F. Weight loss

Correct Answers: D, E, and F Mycobacterium tuberculosis (TB) is a gram-positive, acid-fast bacillus that is transmitted through the airborne route. TB is usually (85%) pulmonary but can also be extrapulmonary (eg, meninges, genitourinary, bone and joints, gastrointestinal). Clinical manifestations of TB, regardless of location, include: — Low-grade fever — Night sweats — Anorexia and weight loss — Fatigue Additional symptoms depend on the location of the infection. Pulmonary TB typically includes: — Cough — Purulent or blood-tinged sputum (hemoptysis) — Shortness of breath — Dyspnea Dyspnea and hemoptysis are typically seen in the late stages. The classic manifestations of TB can be absent in immunocompromised clients and the elderly. Incorrect Answers: [A. Dysuria] Dysuria is a symptom of extrapulmonary genitourinary TB. [B. Jaundice] Jaundice can be present in disseminated TB with liver involvement. It can also be a side effect associated with drugs used to treat pulmonary TB (eg, isoniazid). [C. Low back pain] Back pain is a symptom of spinal TB. Educational objective:Characteristic signs and symptoms associated with tuberculosis (TB) infection, regardless of location, include low-grade fever, night sweats, anorexia, weight loss, and fatigue. Pulmonary TB will also include respiratory symptoms (eg, cough, hemoptysis, dyspnea).

The nurse caring for a client with an ileal conduit observes that the stoma appears bluish gray. What is the nurse's best action? A. Administer an antibacterial agent and assess for additional signs of infection B. Document the findings and continue to monitor for changes C. Measure the stoma and obtain a larger pouching device D. Report the findings to the health care provider immediately

Correct: D. Report the findings to the health care provider immediately An ileal conduit is a surgical technique that uses an excised piece of the client's ileum to create an incontinent urinary diversion. The client's ureters are connected to the ileal conduit, which is used to create an abdominal stoma that allows the passage of urine. A healthy stoma should be pink to brick red and moist, indicating vascularity and viability. If the stoma is dusky or any shade of blue, the nurse should suspect impaired perfusion and contact the health care provider immediately. This finding is considered a medical emergency. Incorrect Answers: [A. Administer an antibacterial agent and assess for additional signs of infection] Infection is a potential complication; signs and symptoms may include fever, elevated white blood cell count, odor, and delayed healing. A bluish gray color indicates impaired perfusion, not infection. [B. Document the findings and continue to monitor for changes] Although the nurse will document the findings and monitor for changes, lack of perfusion to the stoma is an emergency that must be reported immediately. [C. Measure the stoma and obtain a larger pouching device] Applying an appropriate-size pouching system (approximately 0.1 in [0.25 cm] larger than the stoma) prevents decreased perfusion and skin irritation. Using a larger drainage bag, especially at night, prevents urine backflow through the stoma and reduces the risk for infection. These are important concepts of stoma care but are not the priority at this time. Educational objective:Stoma care involves frequent nursing assessment for signs of potential complications such as impaired perfusion, infection, and wound dehiscence. The stoma should be pink to brick red and moist. Suspected impaired perfusion is considered a surgical emergency and should be reported to the health care provider immediately.

Romberg's sign

Falling to one side when standing with feet together and eyes closed, indicating abnormal cerebellar function or inner ear dysfunction

Heel-to-Shin (Test) (HTS)

The heel to shin test is a measure of coordination and may be abnormal if there is loss of motor strength, proprioception or a cerebellar lesion. If motor and sensory systems are intact, an abnormal, asymmetric heel to shin test is highly suggestive of an ipsilateral cerebellar lesion.

Phalen's test

The phalen's test is a provocative test used in the diagnosis of CTS. This occurs when the median nerve is compressed or squeezed at the wrist. The Phalen test is performed by asking the patient to place both elbows on a table while keeping both forearms vertical and flexing both wrists at 90 degrees for 60 seconds. A positive test is defined as the occurrence of pain or paresthesias in at least one finger innervated by the median nerve.

An 81-year-old client is admitted to a rehabilitation facility 3 days after total hip replacement. The next morning, the unlicensed assistive personnel (UAP) takes the client's vital signs, but when the UAP returns to assist the client with a shower, the client curses at and tries to hit the UAP. Which is the most appropriate response by the practical nurse? Temperature | 98.7 (37.05 C) Blood Pressure | 110/64 mm Hg Heart Rate | 92/min Respirations | 22/min O2 saturations | 90% on room air A. "I will walk to the room to observe the client's behavior." B. "It sounds like the client is not satisfied with the care provided. I'll see if we can make the client more comfortable." C. "Just leave the client alone now and try again later." D. "The client probably has dementia and is under a lot of stress with the change of environment."

This client is exhibiting behaviors that are concerning for delirium. Therefore, the nursing priority is to go to the client's room to observe the behavior and determine whether the client has delirium and its cause. Delirium is characterized by behavior changes and confusion that have an acute onset, and it is usually reversible. Common causes in older adults include infection, medications, and hypoxia. This client's vital signs (mildly elevated temperature, respiratory rate, and hypoxia) and recent surgery suggest pulmonary infection as the cause of the delirium. Although a temperature of 98.7 F (37 C) is normal for younger adults, it may indicate fever in an 81-year-old as mean body temperature decreases with age. Other signs of pulmonary infection include crackles in the lungs, productive cough, and pleuritic chest pain. Incorrect Answers: [B. "It sounds like the client is not satisfied with the care provided. I'll see if we can make the client more comfortable."] The nurse is making an assumption and there is not enough information to support dissatisfaction as the cause of this client's behavior. Further monitoring is needed. [C. "Just leave the client alone now and try again later."] This client is exhibiting signs of delirium, which is a medical emergency. Leaving the client alone without further observation and appropriate, timely intervention would constitute negligence. [D. "The client probably has dementia and is under a lot of stress with the change of environment."] The nurse is assuming that the client has dementia based on age. However, only 1 of every 8 older adults has dementia. Educational objective:Delirium is a common manifestation of a serious physiologic instability in older adults characterized by acute changes in cognition and behavior. When a client is suspected of having delirium, the nursing priority is evaluation of the cause to guide interventions.

Hirsutism

a condition in women that results in excessive growth of dark or coarse hair in a male-like pattern — face, chest and back. With hirsutism, extra hair growth often arises from excess male hormones (androgens), primarily testosterone.

Ventricular bigeminy occurs when:

every second complex is a PVC.

Acanthosis nigricans

thickening and darkening of skin near axillary region, A/w Diabetes Type II and gastric carcinoma


Kaugnay na mga set ng pag-aaral

A&P II: Lecture Exam 3: Notes 6: Pulmonary Ventilation

View Set

Clinical Medicine II Exam 6 - Cardiology Part 2

View Set

Title: The Electronic Health Record for the Physician's Office with SimChart for the Medical Office

View Set

American History Chapter 24 NWCC Weekly Questions

View Set

Academic and Interpersonal Skills: Mastery Test

View Set

Howard School - Plessy v. Ferguson

View Set

Фармакология,Антибиотици

View Set

AP Chemistry Comprehensive Review

View Set

RN Comprehensive Online Practice 2023 B

View Set